All Related Questions of Corporate Finance

Q: Your firm is considering a $150 million investment to launch a

Your firm is considering a $150 million investment to launch a new product line. The project is expected to generate a free cash flow of $20 million per year, and its unlevered cost of capital is 10%....

See Answer

Q: Consider Avco’s RFX project from Section 18.3. Suppose that

Consider Avco’s RFX project from Section 18.3. Suppose that Avco is receiving government loan guarantees that allow it to borrow at the 6% rate. Without these guarantees, Avco would pay 6.5% on its de...

See Answer

Q: Arden Corporation is considering an investment in a new project with an

Arden Corporation is considering an investment in a new project with an unlevered cost of capital of 9%. Arden’s marginal corporate tax rate is 40%, and its debt cost of capital is 5%. a. Suppose Arde...

See Answer

Q: XL Sports is expected to generate free cash flows of $10

XL Sports is expected to generate free cash flows of $10.9 million per year. XL has permanent debt of $40 million, a tax rate of 40%, and an unlevered cost of capital of 10%. a. What is the value of X...

See Answer

Q: Propel Corporation plans to make a $50 million investment, initially

Propel Corporation plans to make a $50 million investment, initially funded completely with debt. The free cash flows of the investment and Propel’s incremental debt from the project...

See Answer

Q: Gartner Systems has no debt and an equity cost of capital of

Gartner Systems has no debt and an equity cost of capital of 10%. Gartner’s current market capitalization is $100 million, and its free cash flows are expected to grow at 3% per year. Gartner’s corpor...

See Answer

Q: Revtek, Inc., has an equity cost of capital of 12

Revtek, Inc., has an equity cost of capital of 12% and a debt cost of capital of 6%. Revtek maintains a constant debt-equity ratio of 0.5, and its tax rate is 35%. a. What is Revtek’s WACC given its c...

See Answer

Q: In 2015, Intel Corporation had a market capitalization of $134

In 2015, Intel Corporation had a market capitalization of $134 billion, debt of $13.2 billion, cash of $13.8 billion, and EBIT of nearly $16 billion. If Intel were to increase its debt by $1 billion a...

See Answer

Q: Backcountry Adventures is a Colorado-based outdoor travel agent that operates

Backcountry Adventures is a Colorado-based outdoor travel agent that operates a series of winter backcountry huts. Currently, the value of the firm (debt + equity) is $3.5 million. But profits will de...

See Answer

Q: Find online the annual 10-K report for Costco Wholesale Corporation

Find online the annual 10-K report for Costco Wholesale Corporation (COST) for fiscal year 2015 (filed in October 2015). Answer the following questions from their income statement: a. What were Costco...

See Answer

Q: Suppose Goodyear Tire and Rubber Company is considering divesting one of its

Suppose Goodyear Tire and Rubber Company is considering divesting one of its manufacturing plants. The plant is expected to generate free cash flows of $1.5 million per year, growing at a rate of 2.5%...

See Answer

Q: Suppose Alcatel-Lucent has an equity cost of capital of 10

Suppose Alcatel-Lucent has an equity cost of capital of 10%, market capitalization of $10.8 billion, and an enterprise value of $14.4 billion. Suppose Alcatel-Lucent’s debt cost of c...

See Answer

Q: Acort Industries has 10 million shares outstanding and a current share price

Acort Industries has 10 million shares outstanding and a current share price of $40 per share. It also has long-term debt outstanding. This debt is risk free, is four years away from maturity, has ann...

See Answer

Q: Suppose Goodyear Tire and Rubber Company has an equity cost of capital

Suppose Goodyear Tire and Rubber Company has an equity cost of capital of 8.5%, a debt cost of capital of 7%, a marginal corporate tax rate of 35%, and a debt-equity ratio of 2.6. Suppose Goodyear mai...

See Answer

Q: You are a consultant who was hired to evaluate a new product

You are a consultant who was hired to evaluate a new product line for Markum Enterprises. The upfront investment required to launch the product line is $10 million. The product will generate free cash...

See Answer

Q: You would like to compare Ideko’s profitability to its competitors’ profitability using

You would like to compare Ideko’s profitability to its competitors’ profitability using the EBITDA/ sales multiple. Given Ideko’s current sales of...

See Answer

Q: Reproduce Ideko’s balance sheet and statement of cash flows, assuming Ideko’s

Reproduce Ideko’s balance sheet and statement of cash flows, assuming Ideko’s market share will increase by 0.5% per year; investment, financing, and depreciation will be adjusted accordingly; and the...

See Answer

Q: Calculate Ideko’s unlevered cost of capital when Ideko’s unlevered beta is 1

Calculate Ideko’s unlevered cost of capital when Ideko’s unlevered beta is 1.1 rather than 1.2, and all other required estimates are the same as in the chapter.

See Answer

Q: Calculate Ideko’s unlevered cost of capital when the market risk premium is

Calculate Ideko’s unlevered cost of capital when the market risk premium is 6% rather than 5%, the risk-free rate is 5% rather than 4%, and all other required estimates are the same as in the chapter....

See Answer

Q: Using the information produced in the income statement in Problem 4,

Using the information produced in the income statement in Problem 4, use EBITDA as a multiple to estimate the continuation value in 2010, assuming the current value remains unchanged (reproduce Table...

See Answer

Q: See Table 2.5 showing financial statement data and stock price

See Table 2.5 showing financial statement data and stock price data for Mydeco Corp. a. By what percentage did Mydeco’s revenues grow each year from 2013–2016? b. B...

See Answer

Q: How does the assumption on future improvements in working capital affect your

How does the assumption on future improvements in working capital affect your answer to Problem 13? Data from Problem 13: Using the information produced in the income statement in Problem 4, use EBI...

See Answer

Q: Approximately what expected future long-run growth rate would provide the

Approximately what expected future long-run growth rate would provide the same EBITDA multiple in 2010 as Ideko has today (i.e., 9.1)? Assume that the future debt-to-value ratio is held constant at 40...

See Answer

Q: Approximately what expected future long-run growth rate would provide the

Approximately what expected future long-run growth rate would provide the same EBITDA multiple in 2010 as Ideko has today (i.e., 9.1)? Assume that the future debt-to-value ratio is held constant at 40...

See Answer

Q: Using the APV method, estimate the value of Ideko and the

Using the APV method, estimate the value of Ideko and the NPV of the deal using the continuation value you calculated in Problem 13 and the unlevered cost of capital estimate in Section 19.4. Assume t...

See Answer

Q: Using the APV method, estimate the value of Ideko and the

Using the APV method, estimate the value of Ideko and the NPV of the deal using the continuation value you calculated in Problem 13 and the unlevered cost of capital estimate in Section 19.4. Assume t...

See Answer

Q: Use your answers from Problems 17 and 18 to infer the value

Use your answers from Problems 17 and 18 to infer the value today of the projected improvements in working capital under the assumptions that Ideko’s market share will increase by 0.5% per year and th...

See Answer

Q: Assume that Ideko’s market share will increase by 0.5%

Assume that Ideko’s market share will increase by 0.5% per year rather than the 1% used in the chapter. What production capacity will Ideko require each year? When will an expansion become necessary (...

See Answer

Q: Under the assumption that Ideko market share will increase by 0.

Under the assumption that Ideko market share will increase by 0.5% per year, you determine that the plant will require an expansion in 2010. The cost of this expansion will be $15 million. Assuming th...

See Answer

Q: Under the assumption that Ideko’s market share will increase by 0.

Under the assumption that Ideko’s market share will increase by 0.5% per year (and the investment and financing will be adjusted as described in Problem 3), you project the following...

See Answer

Q: Under the assumptions that Ideko’s market share will increase by 0.

Under the assumptions that Ideko’s market share will increase by 0.5% per year and that the forecasts in Table 19.8 remain the same, calculate Ideko’s working capit...

See Answer

Q: See Table 2.5 showing financial statement data and stock price

See Table 2.5 showing financial statement data and stock price data for Mydeco Corp. Suppose Mydeco repurchases 2 million shares each year from 2013 to 2016. What would its earnings per share be in ye...

See Answer

Q: Under the assumptions that Ideko’s market share will increase by 0.

Under the assumptions that Ideko’s market share will increase by 0.5% per year but that the projected improvements in net working capital do not transpire (so the numbers in Table 19...

See Answer

Q: Forecast Ideko’s free cash flow (reproduce Table 19.10),

Forecast Ideko’s free cash flow (reproduce Table 19.10), assuming Ideko’s market share will increase by 0.5% per year; investment, financing, and depreciation will...

See Answer

Q: Forecast Ideko’s free cash flow (reproduce Table 19.10),

Forecast Ideko’s free cash flow (reproduce Table 19.10), assuming Ideko’s market share will increase by 0.5% per year; investment, financing, and depreciation will...

See Answer

Q: Reproduce Ideko’s balance sheet and statement of cash flows, assuming Ideko’s

Reproduce Ideko’s balance sheet and statement of cash flows, assuming Ideko’s market share will increase by 0.5% per year; investment, financing, and depreciation will be adjusted accordingly; and the...

See Answer

Q: Explain the meanings of the following financial terms: a.

Explain the meanings of the following financial terms: a. Option b. Expiration date c. Strike price d. Call e. Put

See Answer

Q: What position has more downside exposure: a short position in a

What position has more downside exposure: a short position in a call or a short position in a put? That is, in the worst case, in which of these two positions would your losses be greater?

See Answer

Q: Consider the October 2015 IBM call and put options in Problem 3

Consider the October 2015 IBM call and put options in Problem 3. Ignoring any interest you might earn over the remaining few days’ life of the options: a. Compute the break-even IBM...

See Answer

Q: A forward contract is a contract to purchase an asset at a

A forward contract is a contract to purchase an asset at a fixed price on a particular date in the future. Both parties are obligated to fulfill the contract. Explain how to construct a forward contra...

See Answer

Q: You own a share of Costco stock. You are worried that

You own a share of Costco stock. You are worried that its price will fall and would like to insure yourself against this possibility. How can you purchase insurance against this possibility?

See Answer

Q: It is October 5, 2015, and you own IBM stock

It is October 5, 2015, and you own IBM stock. You would like to insure that the value of your holdings will not fall significantly. Using the data in Problem 3, and expressing your answer in terms of...

See Answer

Q: Explain the difference between an S corporation and a C corporation.

Explain the difference between an S corporation and a C corporation.

See Answer

Q: See Table 2.5 showing financial statement data and stock price

See Table 2.5 showing financial statement data and stock price data for Mydeco Corp. Suppose Mydeco had purchased additional equipment for $12 million at the end of 2013, and this equipment was deprec...

See Answer

Q: Consider the October 2015 IBM call and put options in Problem 3

Consider the October 2015 IBM call and put options in Problem 3. Ignoring the negligible interest you might earn on T-bills over the remaining few days’ life of the options, show that there is no arbi...

See Answer

Q: What is the difference between a European option and an American option

What is the difference between a European option and an American option? Are European options available exclusively in Europe and American options available exclusively in the United States?

See Answer

Q: In mid-February 2016, European-style options on the

In mid-February 2016, European-style options on the S&P 100 index (OEX) expiring in December 2017 were priced as follows: Given an interest rate of 0.40% for a December 2017 maturity (22 months...

See Answer

Q: Suppose Amazon stock is trading for $500 per share, and

Suppose Amazon stock is trading for $500 per share, and Amazon pays no dividends. a. What is the maximum possible price of a call option on Amazon? b. What is the maximum possible price of a put optio...

See Answer

Q: Consider the data for IBM options in Problem 3. Suppose a

Consider the data for IBM options in Problem 3. Suppose a new American-style put option on IBM is issued with a strike price of $155 and an expiration date of November 1st. a. What is the maximum poss...

See Answer

Q: You are watching the option quotes for your favorite stock, when

You are watching the option quotes for your favorite stock, when suddenly there is a news announcement. Explain what type of news would lead to the following effects: a. Call prices increase, and put...

See Answer

Q: Explain why an American call option on a non-dividend-

Explain why an American call option on a non-dividend-paying stock always has the same price as its European counterpart.

See Answer

Q: Consider an American put option on XAL stock with a strike price

Consider an American put option on XAL stock with a strike price of $55 and one year to expiration. Assume XAL pays no dividends, XAL is currently trading for $10 per share, and the one-year interest...

See Answer

Q: The stock of Harford Inc. is about to pay a $

The stock of Harford Inc. is about to pay a $0.30 dividend. It will pay no more dividends for the next month. Consider call options that expire in one month. If the interest rate is 6% APR (monthly co...

See Answer

Q: Suppose the S&P 500 is at 900, and a

Suppose the S&P 500 is at 900, and a one-year European call option with a strike price of $400 has a negative time value. If the interest rate is 5%, what can you conclude about the dividend yield of...

See Answer

Q: Suppose a firm’s tax rate is 35%. a. What

Suppose a firm’s tax rate is 35%. a. What effect would a $10 million operating expense have on this year’s earnings? What effect would it have on next year’s earnings? b. What effect would a $10 milli...

See Answer

Q: Suppose the S&P 500 is at 900, and it

Suppose the S&P 500 is at 900, and it will pay a dividend of $30 at the end of the year. Suppose also that the interest rate is 2%. If a one-year European put option has a negative time value, what is...

See Answer

Q: Wesley Corp. stock is trading for $25/share.

Wesley Corp. stock is trading for $25/share. Wesley has 20 million shares outstanding and a market debt-equity ratio of 0.5. Wesley’s debt is zero-coupon debt with a 5-year maturity and a yield to mat...

See Answer

Q: Below is an option quote on IBM from the CBOE Web site

Below is an option quote on IBM from the CBOE Web site showing options expiring in October and November 2015. a. Which option contract had the most trades on that day? b. Which option contract is bein...

See Answer

Q: Use the option data from July 13, 2009 in the following

Use the option data from July 13, 2009 in the following table to determine the rate Google would have paid if it had issued $128 billion in zero-coupon debt due in January 2011. Suppose Google current...

See Answer

Q: Suppose that in July 2009, Google were to issue $96

Suppose that in July 2009, Google were to issue $96 billion in zero-coupon senior debt, and another $32 billion in zero-coupon junior debt, both due in January 2011. Use the option data in the precedi...

See Answer

Q: Explain the difference between a long position in a put and a

Explain the difference between a long position in a put and a short position in a call.

See Answer

Q: Which of the following positions benefit if the stock price increases?

Which of the following positions benefit if the stock price increases? a. Long position in a call b. Short position in a call c. Long position in a put d. Short position in a put

See Answer

Q: The current price of Estelle Corporation stock is $25. In

The current price of Estelle Corporation stock is $25. In each of the next two years, this stock price will either go up by 20% or go down by 20%. The stock pays no dividends. The one-year risk-free i...

See Answer

Q: Consider the setting of Problem 9. Suppose that in the event

Consider the setting of Problem 9. Suppose that in the event Hema Corp. defaults, $90 million of its value will be lost to bankruptcy costs. Assume there are no other market imperfections. a. What is...

See Answer

Q: Roslin Robotics stock has a volatility of 30% and a current

Roslin Robotics stock has a volatility of 30% and a current stock price of $60 per share. Roslin pays no dividends. The risk-free interest is 5%. Determine the Black-Scholes value of a one year, at-th...

See Answer

Q: Find online the annual 10-K report for Costco Wholesale Corporation

Find online the annual 10-K report for Costco Wholesale Corporation (COST) for fiscal year 2015 (filed in October 2015). Answer the following questions from their cash flow statement: a. How much cash...

See Answer

Q: Rebecca is interested in purchasing a European call on a hot new

Rebecca is interested in purchasing a European call on a hot new stock, Up, Inc. The call has a strike price of $100 and expires in 90 days. The current price of Up stock is $120, and the stock has a...

See Answer

Q: Using the data in Table 21.1, compare the price

Using the data in Table 21.1, compare the price on July 24, 2009, of the following options on JetBlue stock to the price predicted by the Black-Scholes formula. Assume that the standard deviation of...

See Answer

Q: Using the market data in Figure 20.10 and a risk

Using the market data in Figure 20.10 and a risk-free rate of 0.25% per annum, calculate the implied volatility of Google stock in September 2012, using the bid price of the 700 January 2014 call opti...

See Answer

Q: Using the implied volatility you calculated in Problem 14, and the

Using the implied volatility you calculated in Problem 14, and the information in that problem, use the Black-Scholes option pricing formula to calculate the value of the 800 January 2014 call option....

See Answer

Q: Plot the value of a two-year European put option with

Plot the value of a two-year European put option with a strike price of $20 on World Wide Plants as a function of the stock price. Recall that World Wide Plants has a constant dividend yield of 5% per...

See Answer

Q: Consider the at-the-money call option on Roslin Robotics

Consider the at-the-money call option on Roslin Robotics evaluated in Problem 11. Suppose the call option is not available for trade in the market. You would like to replicate a long position in 1000...

See Answer

Q: Consider again the at-the-money call option on Roslin

Consider again the at-the-money call option on Roslin Robotics evaluated in Problem 11. What is the impact on the value of this call option of each of the following changes (evaluated separately)? a....

See Answer

Q: Harbin Manufacturing has 10 million shares outstanding with a current share price

Harbin Manufacturing has 10 million shares outstanding with a current share price of $20 per share. In one year, the share price is equally likely to be $30 or $18. The risk-free interest rate is 5%....

See Answer

Q: Using the information in Problem 1, use the Binomial Model to

Using the information in Problem 1, use the Binomial Model to calculate the price of a oneyear put option on Estelle stock with a strike price of $25. Data from Problem 1: The current price of Estel...

See Answer

Q: Using the information on Harbin Manufacturing in Problem 19, answer the

Using the information on Harbin Manufacturing in Problem 19, answer the following: a. Using the risk-neutral probabilities, what is the value of a one-year call option on Harbin stock with a strike pr...

See Answer

Q: Who reads financial statements? List at least three different categories of

Who reads financial statements? List at least three different categories of people. For each category, provide an example of the type of information they might be interested in and discuss why.

See Answer

Q: Using the information in Problem 1, calculate the risk-neutral

Using the information in Problem 1, calculate the risk-neutral probabilities. Then use them to price the option. Data from Problem 1: The current price of Estelle Corporation stock is $25. In each o...

See Answer

Q: Using the information in Problem 3, calculate the risk-neutral

Using the information in Problem 3, calculate the risk-neutral probabilities. Then use them to price the option. Data from Problem 3: The current price of Natasha Corporation stock is $6. In each of...

See Answer

Q: Explain the difference between the risk-neutral and actual probabilities.

Explain the difference between the risk-neutral and actual probabilities. In which states is one higher than the other? Why?

See Answer

Q: Explain why risk-neutral probabilities can be used to price derivative

Explain why risk-neutral probabilities can be used to price derivative securities in a world where investors are risk averse.

See Answer

Q: Calculate the beta of the January 2010 $9 call option on

Calculate the beta of the January 2010 $9 call option on JetBlue listed in Table 21.1. Assume that the volatility of JetBlue is 65% per year and its beta is 0.85. The short-term risk-free rate of inte...

See Answer

Q: Consider the March 2010 $5 put option on JetBlue listed in

Consider the March 2010 $5 put option on JetBlue listed in Table 21.1. Assume that the volatility of JetBlue is 65% per year and its beta is 0.85. The short-term risk-free rate of interest is 1% per y...

See Answer

Q: You would like to know the unlevered beta of Schwartz Industries (

You would like to know the unlevered beta of Schwartz Industries (SI). SI’s value of outstanding equity is $400 million, and you have estimated its beta to be 1.2. SI has four-year zero-coupon debt ou...

See Answer

Q: The J. Miles Corp. has 25 million shares outstanding with

The J. Miles Corp. has 25 million shares outstanding with a share price of $20 per share. Miles also has outstanding zero-coupon debt with a 5-year maturity, a face value of $900 million, and a yield...

See Answer

Q: The current price of Natasha Corporation stock is $6. In

The current price of Natasha Corporation stock is $6. In each of the next two years, this stock price can either go up by $2.50 or go down by $2. The stock pays no dividends. The one-year risk-free in...

See Answer

Q: Using the information in Problem 3, use the Binomial Model to

Using the information in Problem 3, use the Binomial Model to calculate the price of a twoyear European put option on Natasha stock with a strike price of $7. Data from Problem 3: The current price...

See Answer

Q: Find the most recent financial statements for Starbucks Corporation (SBUX)

Find the most recent financial statements for Starbucks Corporation (SBUX) using the following sources: a. From the company’s Web page www.starbucks.com. b. From the SEC Web site www.sec.gov. c. From...

See Answer

Q: Suppose the option in Example 21.1 actually sold in the

Suppose the option in Example 21.1 actually sold in the market for $8. Describe a trading strategy that yields arbitrage profits. Example 21.1: Problem Suppose a stock is currently trading for $60,...

See Answer

Q: Suppose the option in Example 21.2 actually sold today for

Suppose the option in Example 21.2 actually sold today for $5. You do not know what the option will trade for next period. Describe a trading strategy that will yield arbitrage profits. Example 21.2:...

See Answer

Q: Eagletron’s current stock price is $10. Suppose that over the

Eagletron’s current stock price is $10. Suppose that over the current year, the stock price will either increase by 100% or decrease by 50%. Also, the risk-free rate is 25% (EAR). a. What is the value...

See Answer

Q: What is the highest possible value for the delta of a call

What is the highest possible value for the delta of a call option? What is the lowest possible value?

See Answer

Q: Hema Corp. is an all equity firm with a current market

Hema Corp. is an all equity firm with a current market value of $1000 million (i.e., $1 billion), and will be worth $900 million or $1400 million in one year. The risk-free interest rate is 5%. Suppos...

See Answer

Q: Your company is planning on opening an office in Japan. Profits

Your company is planning on opening an office in Japan. Profits depend on how fast the economy in Japan recovers from its current recession. There is a 50% chance of recovery this year. You are trying...

See Answer

Q: Under the same assumptions as in Section 22.3, suppose

Under the same assumptions as in Section 22.3, suppose your corporation owns an operating electric car dealership, together with one-year options to open five more. a. What is the value and beta of yo...

See Answer

Q: The management of Southern Express Corporation is considering investing 10% of

The management of Southern Express Corporation is considering investing 10% of all future earnings in growth. The company has a single growth opportunity that it can take either now or in one period....

See Answer

Q: What decision should you make in Problem 2 if the one-

What decision should you make in Problem 2 if the one-year cost of capital is 15.44% and the profits last forever? Data from Problem 2: You are trying to decide whether to make an investment of $500...

See Answer

Q: Your R&D division has just synthesized a material that will

Your R&D division has just synthesized a material that will super conduct electricity at room temperature; you have given the go-ahead to try to produce this material commercially. It will take five y...

See Answer

Q: Consider the following potential events that might have taken place at Global

Consider the following potential events that might have taken place at Global Conglomerate on December 30, 2015. For each one, indicate which line items in Global’s balance sheet would be affected and...

See Answer

Q: You are an analyst working for Goldman Sachs, and you are

You are an analyst working for Goldman Sachs, and you are trying to value the growth potential of a large, established company, Big Industries. Big Industries has a thriving R&D division that has cons...

See Answer

Q: Repeat Problem 14, but this time assume that all the probabilities

Repeat Problem 14, but this time assume that all the probabilities are risk-neutral probabilities, which means the cost of capital is always the risk-free rate and risk-free rates are as follows: The...

See Answer

Q: You own a small networking startup. You have just received an

You own a small networking startup. You have just received an offer to buy your firm from a large, publicly traded firm, JCH Systems. Under the terms of the offer, you will receive 1 million shares of...

See Answer

Q: You own a wholesale plumbing supply store. The store currently generates

You own a wholesale plumbing supply store. The store currently generates revenues of $1 million per year. Next year, revenues will either decrease by 10% or increase by 5%, with equal probability, and...

See Answer

Q: You own a copper mine. The price of copper is currently

You own a copper mine. The price of copper is currently $1.50 per pound. The mine produces 1 million pounds of copper per year and costs $2 million per year to operate. It has enough copper to operate...

See Answer

Q: An original silver dollar from the late eighteenth century consists of approximately

An original silver dollar from the late eighteenth century consists of approximately 24 grams of silver. At a price of $0.19 per gram ($6 per troy ounce), the silver content of the coin is worth about...

See Answer

Q: You are trying to decide whether to make an investment of $

You are trying to decide whether to make an investment of $500 million in a new technology to produce Everlasting Gobstoppers. There is a 60% chance that the market for these candies will produce prof...

See Answer

Q: You own a piece of raw land in an up-and

You own a piece of raw land in an up-and-coming area in Gotham City. The costs to construct a building increase disproportionately with the size of the building. A building of q square feet costs 0.1...

See Answer

Q: What implicit assumption is made when managers use the equivalent annual benefit

What implicit assumption is made when managers use the equivalent annual benefit method to decide between two projects with different lives that use the same resource?

See Answer

Q: You own a cab company and are evaluating two options to replace

You own a cab company and are evaluating two options to replace your fleet. Either you can take out a five-year lease on the replacement cabs for $500 per month per cab, or you can purchase the cabs o...

See Answer

Q: What was the change in Global Conglomerate’s book value of equity from

What was the change in Global Conglomerate’s book value of equity from 2014 to 2015 according to Table 2.1? Does this imply that the market price of Global’s shares...

See Answer

Q: Genenco is developing a new drug that will slow the aging process

Genenco is developing a new drug that will slow the aging process. In order to succeed, two breakthroughs are needed: one to increase the potency of the drug, and the second to eliminate toxic side ef...

See Answer

Q: Your engineers are developing a new product to launch next year that

Your engineers are developing a new product to launch next year that will require both software and hardware innovations. The software team requests a budget of $5 million and forecasts an 80% chance...

See Answer

Q: Your firm is thinking of expanding. If you invest today,

Your firm is thinking of expanding. If you invest today, the expansion will generate $10 million in FCF at the end of the year, and will have a continuation value of either $150 million (if the econom...

See Answer

Q: Assume that the project in Example 22.5 pays an annual

Assume that the project in Example 22.5 pays an annual cash flow of $100,000 (instead of $90,000). a. What is the NPV of investing today? b. What is the NPV of waiting and investing tomorrow? c. Verif...

See Answer

Q: Assume that the project in Example 22.5 pays an annual

Assume that the project in Example 22.5 pays an annual cash flow of $80,000 (instead of $90,000). a. What is the NPV of investing today? b. What is the NPV of waiting and investing tomorrow? c. Verify...

See Answer

Q: Consider the United Studios example in Section 22.2. Suppose

Consider the United Studios example in Section 22.2. Suppose United has the rights to produce the first film, but has not yet purchased the sequel rights. a. How much are the sequel rights worth to Un...

See Answer

Q: Using the information in Problem 2, rework the problem assuming you

Using the information in Problem 2, rework the problem assuming you find out the size of the Everlasting Gobstopper market one year after you make the investment. That is, if you do not make the inves...

See Answer

Q: Describe the benefits and costs of delaying an investment opportunity.

Describe the benefits and costs of delaying an investment opportunity.

See Answer

Q: You are a financial analyst at Global Conglomerate and are considering entering

You are a financial analyst at Global Conglomerate and are considering entering the shoe business. You believe that you have a very narrow window for entering this market. Because of Christmas demand,...

See Answer

Q: It is the beginning of September and you have been offered the

It is the beginning of September and you have been offered the following deal to go heli-skiing. If you pick the first week in January and pay for your vacation now, you can get a week of heli-skiing...

See Answer

Q: In early 2012, General Electric (GE) had a book

In early 2012, General Electric (GE) had a book value of equity of $116 billion, 10.6 billion shares outstanding, and a market price of $17.00 per share. GE also had cash of $84 billion, and total deb...

See Answer

Q: A professor in the Computer Science department at United States Institute of

A professor in the Computer Science department at United States Institute of Technology has just patented a new search engine technology and would like to sell it to you, an interested venture capital...

See Answer

Q: Consider again the electric car dealership in Section 22.3.

Consider again the electric car dealership in Section 22.3. Suppose the current value of a dealership is $5 million because the first-year free cash flow is expected to be $500,000 rather than $600,00...

See Answer

Q: What are some of the alternative sources from which private companies can

What are some of the alternative sources from which private companies can raise equity capital?

See Answer

Q: Do underwriters face the most risk from a best-efforts IPO

Do underwriters face the most risk from a best-efforts IPO, a firm commitment IPO, or an auction IPO? Why?

See Answer

Q: Three years ago, you founded Outdoor Recreation, Inc., a

Three years ago, you founded Outdoor Recreation, Inc., a retailer specializing in the sale of equipment and clothing for recreational activities such as camping, skiing, and hiking. So far, your compa...

See Answer

Q: What is IPO underpricing? If you decide to try to buy

What is IPO underpricing? If you decide to try to buy shares in every IPO, will you necessarily make money from the underpricing?

See Answer

Q: Your firm has 10 million shares outstanding, and you are about

Your firm has 10 million shares outstanding, and you are about to issue 5 million new shares in an IPO. The IPO price has been set at $20 per share, and the underwriting spread is 7%. The IPO is a big...

See Answer

Q: You have an arrangement with your broker to request 1000 shares of

You have an arrangement with your broker to request 1000 shares of all available IPOs. Suppose that 10% of the time, the IPO is “very successful” and appreciates by 100% on the first day, 80% of the t...

See Answer

Q: What are the advantages to a company of selling stock in an

What are the advantages to a company of selling stock in an SEO using a cash offer? What are the advantages of a rights offer?

See Answer

Q: What are the advantages and the disadvantages to a private company of

What are the advantages and the disadvantages to a private company of raising money from a corporate investor?

See Answer

Q: In early-2015, Abercrombie & Fitch (ANF) had

In early-2015, Abercrombie & Fitch (ANF) had a book equity of $1390 million, a price per share of $25.52, and 69.35 million shares outstanding. At the same time, The Gap (GPS) had a book equity of $29...

See Answer

Q: Suppose venture capital firm GSB partners raised $100 million of committed

Suppose venture capital firm GSB partners raised $100 million of committed capital. Each year over the 10-year life of the fund, 2% of this committed capital will be used to pay GSB’s management fee....

See Answer

Q: Your robotic automation start-up, Kela Controls, has raised

Your robotic automation start-up, Kela Controls, has raised capital as follows: a. How much did Kela raise in each round? b. Assuming no other securities were issued, what fraction of the firmâ...

See Answer

Q: Beru.com recently raised $5 million with a pre-

Beru.com recently raised $5 million with a pre-money value of $9 million. They are seeking to raise another $6 million. What is the largest fraction of the firm they can offer and avoid a down round?...

See Answer

Q: Bit Box has raised $10 million in a Series A round

Bit Box has raised $10 million in a Series A round with $40 million post-money value and a 1.5x liquidation preference, and $25 million in a Series B round with a $75 million post-money value and a 3x...

See Answer

Q: What are the main advantages and disadvantages of going public?

What are the main advantages and disadvantages of going public?

See Answer

Q: Explain some of the differences between a public debt offering and a

Explain some of the differences between a public debt offering and a private debt offering.

See Answer

Q: Explain why bond issuers might voluntarily choose to put restrictive covenants into

Explain why bond issuers might voluntarily choose to put restrictive covenants into a new bond issue.

See Answer

Q: General Electric has just issued a callable 10-year, 6

General Electric has just issued a callable 10-year, 6% coupon bond with annual coupon payments. The bond can be called at par in one year or anytime thereafter on a coupon payment date. It has a pric...

See Answer

Q: Explain why the yield on a convertible bond is lower than the

Explain why the yield on a convertible bond is lower than the yield on an otherwise identical bond without a conversion feature.

See Answer

Q: Why do bonds with lower seniority have higher yields than equivalent bonds

Why do bonds with lower seniority have higher yields than equivalent bonds with higher seniority?

See Answer

Q: Your firm has identified three potential investment projects. The projects and

Your firm has identified three potential investment projects. The projects and their cash flows are shown here: Suppose all cash flows are certain and the risk-free interest rate is 10%. a. What is...

See Answer

Q: Explain the difference between a secured corporate bond and an unsecured corporate

Explain the difference between a secured corporate bond and an unsecured corporate bond.

See Answer

Q: What is the difference between a foreign bond and a Eurobond?

What is the difference between a foreign bond and a Eurobond?

See Answer

Q: Describe the kinds of securities the U.S. government uses

Describe the kinds of securities the U.S. government uses to finance the federal debt.

See Answer

Q: Suppose on January 15, 2013, the U.S.

Suppose on January 15, 2013, the U.S. Treasury issued a five-year inflation-indexed note with a coupon of 3%. On the date of issue, the consumer price index (CPI) was 250. By January 15, 2018, the CPI...

See Answer

Q: On January 15, 2020, the U.S. Treasury

On January 15, 2020, the U.S. Treasury issued a 10-year inflation-indexed note with a coupon of 6%. On the date of issue, the CPI was 400. By January 15, 2030, the CPI had decreased to 300. What princ...

See Answer

Q: Describe what prepayment risk in a GNMA is.

Describe what prepayment risk in a GNMA is.

See Answer

Q: What is the distinguishing feature of how municipal bonds are taxed?

What is the distinguishing feature of how municipal bonds are taxed?

See Answer

Q: Suppose an H1200 supercomputer has a cost of $200,000

Suppose an H1200 supercomputer has a cost of $200,000 and will have a residual market value of $60,000 in five years. The risk-free interest rate is 5% APR with monthly compounding. a. What is the ris...

See Answer

Q: Suppose Amazon is considering the purchase of computer servers and network infrastructure

Suppose Amazon is considering the purchase of computer servers and network infrastructure to expand its very successful business offering cloud-based computing. In total, it will purchase $48 million...

See Answer

Q: Western Airlines is considering a new route that will require adding an

Western Airlines is considering a new route that will require adding an additional Boeing 777 to its fleet. Western can purchase the airplane for $225 million or lease it for $25 million per year. If...

See Answer

Q: Use EDGAR to find Qualcomm’s 10K filing for 2015. From the

Use EDGAR to find Qualcomm’s 10K filing for 2015. From the balance sheet, answer the following questions: a. How much did Qualcomm have in cash, cash equivalents, and marketable securities (short- and...

See Answer

Q: Your computer manufacturing firm must purchase 10,000 keyboards from a

Your computer manufacturing firm must purchase 10,000 keyboards from a supplier. One supplier demands a payment of $100,000 today plus $10 per keyboard payable in one year. Another supplier will charg...

See Answer

Q: Suppose the risk-free interest rate is 5% APR with

Suppose the risk-free interest rate is 5% APR with monthly compounding. If a $2 million MRI machine can be leased for seven years for $22,000 per month, what residual value must the lessor recover to...

See Answer

Q: Consider a five-year lease for a $400,000

Consider a five-year lease for a $400,000 bottling machine, with a residual market value of $150,000 at the end of the five years. If the risk-free interest rate is 6% APR with monthly compounding, co...

See Answer

Q: Acme Distribution currently has the following items on its balance sheet:

Acme Distribution currently has the following items on its balance sheet: Under current FASB accounting standards (that is, prior to 2019), how will Acme’s balance sheet change if...

See Answer

Q: Your firm is considering leasing a $50,000 copier.

Your firm is considering leasing a $50,000 copier. The copier has an estimated economic life of eight years. Suppose the appropriate discount rate is 9% APR with monthly compounding. Classify each lea...

See Answer

Q: Craxton Engineering will either purchase or lease a new $756,

Craxton Engineering will either purchase or lease a new $756,000 fabricator. If purchased, the fabricator will be depreciated on a straight-line basis over seven years. Craxton can lease the fabricato...

See Answer

Q: Riverton Mining plans to purchase or lease $220,000 worth

Riverton Mining plans to purchase or lease $220,000 worth of excavation equipment. If purchased, the equipment will be depreciated on a straight-line basis over five years, after which it will be wort...

See Answer

Q: Suppose Clorox can lease a new computer data processing system for $

Suppose Clorox can lease a new computer data processing system for $975,000 per year for five years. Alternatively, it can purchase the system for $4.25 million. Assume Clorox has a borrowing cost of...

See Answer

Q: Suppose Procter and Gamble (P&G) is considering purchasing

Suppose Procter and Gamble (P&G) is considering purchasing $15 million in new manufacturing equipment. If it purchases the equipment, it will depreciate it on a straight-line basis over the five years...

See Answer

Q: Answer the following questions: a. What is the difference

Answer the following questions: a. What is the difference between a firm’s cash cycle and its operating cycle? b. How will a firm’s cash cycle be affected if a firm increases its inventory, all else...

See Answer

Q: What is meant by “stretching the accounts payable”?

What is meant by “stretching the accounts payable”?

See Answer

Q: Suppose Bank One offers a risk-free interest rate of 5

Suppose Bank One offers a risk-free interest rate of 5.5% on both savings and loans, and Bank Enn offers a risk-free interest rate of 6% on both savings and loans. a. What arbitrage opportunity is ava...

See Answer

Q: Use the financial statements supplied on the next page for International Motor

Use the financial statements supplied on the next page for International Motor Corporation (IMC) to answer the following questions. a. Calculate the cash conversion cycle for IMC for both 2015 and 201...

See Answer

Q: Ohio Valley Homecare Suppliers, Inc. (OVHS) had $

Ohio Valley Homecare Suppliers, Inc. (OVHS) had $20 million in sales in 2015. Its cost of goods sold was $8 million, and its average inventory balance was $2,000,000. a. Calculate the average number o...

See Answer

Q: Which of the following short-term securities would you expect to

Which of the following short-term securities would you expect to offer the highest before tax return: Treasury bills, certificates of deposit, short-term tax exempts, or commercial paper? Why?

See Answer

Q: Does an increase in a firm’s cash cycle necessarily mean that a

Does an increase in a firm’s cash cycle necessarily mean that a firm is managing its cash poorly?

See Answer

Q: The Greek Connection had sales of $32 million in 2015,

The Greek Connection had sales of $32 million in 2015, and a cost of goods sold of $20 million. A simplified balance sheet for the firm appears below: a. Calculate The Greek Connectionâ€&...

See Answer

Q: The Saban Corporation is trying to decide whether to switch to a

The Saban Corporation is trying to decide whether to switch to a bank that will accommodate electronic funds transfers from Saban’s customers. Saban’s financial manager believes the new system would d...

See Answer

Q: What are the three steps involved in establishing a credit policy?

What are the three steps involved in establishing a credit policy?

See Answer

Q: Which of the following companies are likely to have high short-

Which of the following companies are likely to have high short-term financing needs? Why? a. A clothing retailer b. A professional sports team c. An electric utility d. A company that operates toll ro...

See Answer

Q: What is the difference between direct paper and dealer paper?

What is the difference between direct paper and dealer paper?

See Answer

Q: Discuss the three different arrangements under which a firm may use inventory

Discuss the three different arrangements under which a firm may use inventory to secure a loan.

See Answer

Q: Throughout the 1990s, interest rates in Japan were lower than interest

Throughout the 1990s, interest rates in Japan were lower than interest rates in the United States. As a result, many Japanese investors were tempted to borrow in Japan and invest the proceeds in the U...

See Answer

Q: What is the difference between permanent working capital and temporary working capital

What is the difference between permanent working capital and temporary working capital?

See Answer

Q: Quarterly working capital levels for your firm for the next year are

Quarterly working capital levels for your firm for the next year are included in the following table. What are the permanent working capital needs of your company? What are the temporary needs?

See Answer

Q: Why might a company choose to finance permanent working capital with short

Why might a company choose to finance permanent working capital with short-term debt?

See Answer

Q: Hand-to-Mouth (H2M) is currently cash-

Hand-to-Mouth (H2M) is currently cash-constrained, and must make a decision about whether to delay paying one of its suppliers, or take out a loan. They owe the supplier $10,000 with terms of 2/10 Net...

See Answer

Q: Consider two loans with a 1-year maturity and identical face

Consider two loans with a 1-year maturity and identical face values: an 8% loan with a 1% loan origination fee and an 8% loan with a 5% (no-interest) compensating balance requirement. Which loan would...

See Answer

Q: What is the difference between evergreen credit and a revolving line of

What is the difference between evergreen credit and a revolving line of credit?

See Answer

Q: Which of the following one-year $1000 bank loans offers

Which of the following one-year $1000 bank loans offers the lowest effective annual rate? a. A loan with an APR of 6%, compounded monthly b. A loan with an APR of 6%, compounded annually, that also ha...

See Answer

Q: What are the two primary mechanisms under which ownership and control of

What are the two primary mechanisms under which ownership and control of a public corporation can change?

See Answer

Q: Loki, Inc., and Thor, Inc., have entered into

Loki, Inc., and Thor, Inc., have entered into a stock swap merger agreement whereby Loki will pay a 40% premium over Thor’s premerger price. If Thor’s premerger price per share was $40 and Loki’s was...

See Answer

Q: How does a toehold help overcome the free rider problem?

How does a toehold help overcome the free rider problem?

See Answer

Q: An American Depositary Receipt (ADR) is security issued by a

An American Depositary Receipt (ADR) is security issued by a U.S. bank and traded on a U.S. stock exchange that represents a specific number of shares of a foreign stock. For example, Nokia Corporatio...

See Answer

Q: Why do you think mergers cluster in time, causing merger waves

Why do you think mergers cluster in time, causing merger waves?

See Answer

Q: What are some reasons why a horizontal merger might create value for

What are some reasons why a horizontal merger might create value for shareholders?

See Answer

Q: Why do you think shareholders from target companies enjoy an average gain

Why do you think shareholders from target companies enjoy an average gain when acquired, while acquiring shareholders on average often do not gain anything?

See Answer

Q: If you are planning an acquisition that is motivated by trying to

If you are planning an acquisition that is motivated by trying to acquire expertise, you are basically seeking to gain intellectual capital. What concerns would you have in structuring the deal and th...

See Answer

Q: Do you agree that the European Union should be able to block

Do you agree that the European Union should be able to block mergers between two U.S.based firms? Why or why not?

See Answer

Q: How do the carry forward and carryback provisions of the U.

How do the carry forward and carryback provisions of the U.S. tax code affect the benefits of merging to capture operating losses?

See Answer

Q: Diversification is good for shareholders. So why shouldn’t managers acquire firms

Diversification is good for shareholders. So why shouldn’t managers acquire firms in different industries to diversify a company?

See Answer

Q: What inherent characteristic of corporations creates the need for a system of

What inherent characteristic of corporations creates the need for a system of checks on manager behavior?

See Answer

Q: Is it necessarily true that increasing managerial ownership stakes will improve firm

Is it necessarily true that increasing managerial ownership stakes will improve firm performance?

See Answer

Q: How can proxy contests be used to overcome a captured board?

How can proxy contests be used to overcome a captured board?

See Answer

Q: The promised cash flows of three securities are listed here. If

The promised cash flows of three securities are listed here. If the cash flows are risk-free, and the risk-free interest rate is 5%, determine the no-arbitrage price of each security before the first...

See Answer

Q: What is a say-on-pay vote?

What is a say-on-pay vote?

See Answer

Q: What are a board’s options when confronted with dissident shareholders?

What are a board’s options when confronted with dissident shareholders?

See Answer

Q: What is the essential trade-off faced by government in designing

What is the essential trade-off faced by government in designing regulation of public firms?

See Answer

Q: Many of the provisions of the Sarbanes-Oxley Act of 2002

Many of the provisions of the Sarbanes-Oxley Act of 2002 were aimed at auditors. How does this affect corporate governance?

See Answer

Q: The Dodd-Frank Act requires that firms disclose whether employees and

The Dodd-Frank Act requires that firms disclose whether employees and directors are permitted to hedge against declines in the firm’s stock price. Why might this matter for corporate governance?

See Answer

Q: What are the costs and benefits of prohibiting insider trading?

What are the costs and benefits of prohibiting insider trading?

See Answer

Q: How do the laws on insider trading differ for merger- versus

How do the laws on insider trading differ for merger- versus non-merger-related trading?

See Answer

Q: Are the rights of shareholders better protected in the United States or

Are the rights of shareholders better protected in the United States or in France?

See Answer

Q: What are some examples of agency problems?

What are some examples of agency problems?

See Answer

Q: How can a controlling family use a pyramidal control structure to benefit

How can a controlling family use a pyramidal control structure to benefit itself at the expense of other shareholders?

See Answer

Q: An Exchange-Traded Fund (ETF) is a security that

An Exchange-Traded Fund (ETF) is a security that represents a portfolio of individual stocks. Consider an ETF for which each share represents a portfolio of two shares of Hewlett-Packard (HPQ), one sh...

See Answer

Q: What are the advantages and disadvantages of the corporate organizational structure?

What are the advantages and disadvantages of the corporate organizational structure?

See Answer

Q: What is the role of the board of directors in corporate governance

What is the role of the board of directors in corporate governance?

See Answer

Q: How does a board become captured by a CEO?

How does a board become captured by a CEO?

See Answer

Q: What role do security analysts play in monitoring?

What role do security analysts play in monitoring?

See Answer

Q: How are lenders part of corporate governance?

How are lenders part of corporate governance?

See Answer

Q: What is a whistle-blower?

What is a whistle-blower?

See Answer

Q: What are the advantages and disadvantages of increasing the options granted to

What are the advantages and disadvantages of increasing the options granted to CEOs?

See Answer

Q: The William Companies (WMB) owns and operates natural gas pipelines

The William Companies (WMB) owns and operates natural gas pipelines that deliver 12% of the natural gas consumed in the United States. WMB is concerned that a major hurricane could disrupt its Gulfstr...

See Answer

Q: Suppose the current exchange rate is $1.80/£, the

Suppose the current exchange rate is $1.80/£, the interest rate in the United States is 5.25%, the interest rate in the United Kingdom is 4%, and the volatility of the $/£ exchange rate is 10%. Use th...

See Answer

Q: Assume each of the following securities has the same yield-to

Assume each of the following securities has the same yield-to-maturity: a five-year, zero-coupon bond; a nine-year, zero-coupon bond; a five-year annuity; and a nine-year annuity. Rank these securitie...

See Answer

Q: Consider two securities that pay risk-free cash flows over the

Consider two securities that pay risk-free cash flows over the next two years and that have the current market prices shown here: a. What is the no-arbitrage price of a security that pays cash flows...

See Answer

Q: You have been hired as a risk manager for Acorn Savings and

You have been hired as a risk manager for Acorn Savings and Loan. Currently, Acorn’s balance sheet is as follows (in millions of dollars): When you analyze the duration of loans, y...

See Answer

Q: The Citrix Fund has invested in a portfolio of government bonds that

The Citrix Fund has invested in a portfolio of government bonds that has a current market value of $44.8 million. The duration of this portfolio of bonds is 13.5 years. The fund has borrowed to purcha...

See Answer

Q: Your firm needs to raise $100 million in funds. You

Your firm needs to raise $100 million in funds. You can borrow short term at a spread of 1% over LIBOR. Alternatively, you can issue 10-year, fixed-rate bonds at a spread of 2.50% over 10-year Treasur...

See Answer

Q: Genentech’s main facility is located in South San Francisco. Suppose that

Genentech’s main facility is located in South San Francisco. Suppose that Genentech would experience a direct loss of $450 million in the event of a major earthquake that disrupted its operations. The...

See Answer

Q: Your firm imports manufactured goods from China. You are worried that

Your firm imports manufactured goods from China. You are worried that U.S.–China trade negotiations could break down next year, leading to a moratorium on imports. In the event of a moratorium, your f...

See Answer

Q: Your firm faces a 9% chance of a potential loss of

Your firm faces a 9% chance of a potential loss of $10 million next year. If your firm implements new policies, it can reduce the chance of this loss to 4%, but these new policies have an upfront cost...

See Answer

Q: BHP Billiton is the world’s largest mining firm. BHP expects to

BHP Billiton is the world’s largest mining firm. BHP expects to produce 2 billion pounds of copper next year, with a production cost of $0.90 per pound. a. What will be BHP’s operating profit from cop...

See Answer

Q: Your utility company will need to buy 100,000 barrels of

Your utility company will need to buy 100,000 barrels of oil in 10 days’ time, and it is worried about fuel costs. Suppose you go long 100 oil futures contracts, each for 1000 barrel...

See Answer

Q: Suppose Starbucks consumes 100 million pounds of coffee beans per year.

Suppose Starbucks consumes 100 million pounds of coffee beans per year. As the price of coffee rises, Starbucks expects to pass along 60% of the cost to its customers through higher prices per cup of...

See Answer

Q: Your start-up company has negotiated a contract to provide a

Your start-up company has negotiated a contract to provide a database installation for a manufacturing company in Poland. That firm has agreed to pay you $100,000 in three months’ ti...

See Answer

Q: Suppose a security with a risk-free cash flow of $

Suppose a security with a risk-free cash flow of $150 in one year trades for $140 today. If there are no arbitrage opportunities, what is the current risk-free interest rate?

See Answer

Q: Tailor Johnson, the menswear company with a subsidiary in Ethiopia described

Tailor Johnson, the menswear company with a subsidiary in Ethiopia described in Problem 9, is considering the tax benefits resulting from deferring repatriation of the earnings from the subsidiary. Un...

See Answer

Q: Assume that in the original Ityesi example in Table 31.1

Assume that in the original Ityesi example in Table 31.1, all sales actually occur in the United States and are projected to be $60 million per year for four years. Keeping the cost of sales, operatin...

See Answer

Q: Maryland Light, a U.S. light fixtures manufacturer,

Maryland Light, a U.S. light fixtures manufacturer, is considering an investment in Japan. The dollar cost of equity for Maryland Light is 11%. You are in the corporate treasury department, and you ne...

See Answer

Q: The dollar cost of debt for Coval Consulting, a U.

The dollar cost of debt for Coval Consulting, a U.S. research firm, is 7.5%. The firm faces a tax rate of 30% on all income, no matter where it is earned. Managers in the firm need to know its yen cos...

See Answer

Q: The table here shows the no-arbitrage prices of securities A

The table here shows the no-arbitrage prices of securities A and B that we calculated. a. What are the payoffs of a portfolio of one share of security A and one share of security B? b. What is the m...

See Answer

Q: Suppose security C has a payoff of $600 when the economy

Suppose security C has a payoff of $600 when the economy is weak and $1800 when the economy is strong. The risk-free interest rate is 4%. a. Security C has the same payoffs as which portfolio of the s...

See Answer

Q: You work for Innovation Partners and are considering creating a new security

You work for Innovation Partners and are considering creating a new security. This security would pay out $1000 in one year if the last digit in the closing value of the Dow Jones Industrial index in...

See Answer

Q: Suppose a risky security pays an expected cash flow of $80

Suppose a risky security pays an expected cash flow of $80 in one year. The risk-free rate is 4%, and the expected return on the market index is 10%. a. If the returns of this security are high when t...

See Answer

Q: Suppose Hewlett-Packard (HPQ) stock is currently trading on

Suppose Hewlett-Packard (HPQ) stock is currently trading on the NYSE with a bid price of $28.00 and an ask price of $28.10. At the same time, a NASDAQ dealer posts a bid price for HPQ of $27.85 and an...

See Answer

Q: Consider a portfolio of two securities: one share of Johnson and

Consider a portfolio of two securities: one share of Johnson and Johnson (JNJ) stock and a bond that pays $100 in one year. Suppose this portfolio is currently trading with a bid price of $141.65 and...

See Answer

Q: Xia Corporation is a company whose sole assets are $100,

Xia Corporation is a company whose sole assets are $100,000 in cash and three projects that it will undertake. The projects are risk-free and have the following cash flows: Xia plans to invest any u...

See Answer

Q: Your grandmother bought an annuity from Rock Solid Life Insurance Company for

Your grandmother bought an annuity from Rock Solid Life Insurance Company for $200,000 when she retired. In exchange for the $200,000, Rock Solid will pay her $25,000 per year until she dies. The inte...

See Answer

Q: You are thinking of making an investment in a new plant.

You are thinking of making an investment in a new plant. The plant will generate revenues of $1 million per year for as long as you maintain it. You expect that the maintenance cost will start at $50,...

See Answer

Q: What is the forward rate for year 2 (the forward rate

What is the forward rate for year 2 (the forward rate quoted today for an investment that begins in one year and matures in two years)?

See Answer

Q: What is the forward rate for year 3 (the forward rate

What is the forward rate for year 3 (the forward rate quoted today for an investment that begins in two years and matures in three years)? What can you conclude about forward rates when the yield curv...

See Answer

Q: What is the forward rate for year 5 (the forward rate

What is the forward rate for year 5 (the forward rate quoted today for an investment that begins in four years and matures in five years)?

See Answer

Q: Suppose you wanted to lock in an interest rate for an investment

Suppose you wanted to lock in an interest rate for an investment that begins in one year and matures in five years. What rate would you obtain if there are no arbitrage opportunities?

See Answer

Q: Suppose the yield on a one-year, zero-coupon

Suppose the yield on a one-year, zero-coupon bond is 5%. The forward rate for year 2 is 4%, and the forward rate for year 3 is 3%. What is the yield to maturity of a zero-coupon bond that matures in t...

See Answer

Q: Suppose the current market price of corn is $3.75

Suppose the current market price of corn is $3.75 per bushel. Your firm has a technology that can convert 1 bushel of corn to 3 gallons of ethanol. If the cost of conversion is $1.60 per bushel, at wh...

See Answer

Q: Find online the annual 10-K report for Costco Wholesale Corporation

Find online the annual 10-K report for Costco Wholesale Corporation (COST) for fiscal year 2015 (filed in October 2015). Answer the following questions from their balance sheet: a. How much cash did C...

See Answer

Q: You have decided to take your daughter skiing in Utah. The

You have decided to take your daughter skiing in Utah. The best price you have been able to find for a round trip air ticket is $359. You notice that you have 20,000 frequent flier miles that are abou...

See Answer

Q: Suppose the risk-free interest rate is 4%. a

Suppose the risk-free interest rate is 4%. a. Having $200 today is equivalent to having what amount in one year? b. Having $200 in one year is equivalent to having what amount today? c. Which would yo...

See Answer

Q: You have an investment opportunity in Japan. It requires an investment

You have an investment opportunity in Japan. It requires an investment of $1 million today and will produce a cash flow of ¥114 million in one year with no risk. Suppose the risk-free interest rate in...

See Answer

Q: Your firm has a risk-free investment opportunity where it can

Your firm has a risk-free investment opportunity where it can invest $160,000 today and receive $170,000 in one year. For what level of interest rates is this project attractive?

See Answer

Q: Your buddy in mechanical engineering has invented a money machine. The

Your buddy in mechanical engineering has invented a money machine. The main drawback of the machine is that it is slow. It takes one year to manufacture $100. However, once built, the machine will las...

See Answer

Q: How would your answer to Problem 16 change if the machine takes

How would your answer to Problem 16 change if the machine takes one year to build? Data from Problem 16: Your buddy in mechanical engineering has invented a money machine. The main drawback of the m...

See Answer

Q: You currently have a four-year-old mortgage outstanding on

You currently have a four-year-old mortgage outstanding on your house. You make monthly payments of $1500. You have just made a payment. The mortgage has 26 years to go (i.e., it had an original term...

See Answer

Q: You are head of the Schwartz Family Endowment for the Arts.

You are head of the Schwartz Family Endowment for the Arts. You have decided to fund an arts school in the San Francisco Bay area in perpetuity. Every five years, you will give the school $1 million....

See Answer

Q: When you purchased your house, you took out a 30-

When you purchased your house, you took out a 30-year annual-payment mortgage with an interest rate of 6% per year. The annual payment on the mortgage is $12,000. You have just made a payment and have...

See Answer

Q: You are 25 years old and decide to start saving for your

You are 25 years old and decide to start saving for your retirement. You plan to save $5000 at the end of each year (so the first deposit will be one year from now), and will make the last deposit whe...

See Answer

Q: You have decided to form a new start-up company developing

You have decided to form a new start-up company developing applications for the iPhone. Give examples of the three distinct types of financial decisions you will need to make.

See Answer

Q: Your oldest daughter is about to start kindergarten at a private school

Your oldest daughter is about to start kindergarten at a private school. Tuition is $10,000 per year, payable at the beginning of the school year. You expect to keep your daughter in private school th...

See Answer

Q: You are running a hot Internet company. Analysts predict that its

You are running a hot Internet company. Analysts predict that its earnings will grow at 30% per year for the next five years. After that, as competition increases, earnings growth is expected to slow...

See Answer

Q: Ten years ago Diana Torres wrote what has become the leading Tort

Ten years ago Diana Torres wrote what has become the leading Tort textbook. She has been receiving royalties based on revenues reported by the publisher. These revenues started at $1 million in the fi...

See Answer

Q: Your brother has offered to give you $100, starting next

Your brother has offered to give you $100, starting next year, and after that growing at 3% for the next 20 years. You would like to calculate the value of this offer by calculating how much money you...

See Answer

Q: Suppose you currently have $5000 in your savings account, and

Suppose you currently have $5000 in your savings account, and your bank pays interest at a rate of 0.5% per month. If you make no further deposits or withdrawals, how much will you have in the account...

See Answer

Q: Your firm spends $5000 every month on printing and mailing costs

Your firm spends $5000 every month on printing and mailing costs, sending statements to customers. If the interest rate is 0.5% per month, what is the present value of eliminating this cost by sending...

See Answer

Q: You have just entered an MBA program and have decided to pay

You have just entered an MBA program and have decided to pay for your living expenses using a credit card that has no minimum monthly payment. You intend to charge $1000 per month on the card for the...

See Answer

Q: Your credit card charges an interest rate of 2% per month

Your credit card charges an interest rate of 2% per month. You have a current balance of $1000, and want to pay it off. Suppose you can afford to pay off $100 per month. What will your balance be at t...

See Answer

Q: You have decided to buy a perpetuity. The bond makes one

You have decided to buy a perpetuity. The bond makes one payment at the end of every year forever and has an interest rate of 5%. If you initially put $1000 into the bond, what is the payment every ye...

See Answer

Q: You have just made an offer on a new home and are

You have just made an offer on a new home and are seeking a mortgage. You need to borrow $600,000. a. The bank offers a 30-year mortgage with fixed monthly payments and an interest rate of 0.5% per mo...

See Answer

Q: When a pharmaceutical company develops a new drug, it often receives

When a pharmaceutical company develops a new drug, it often receives patent protection for that medication, allowing it to charge a higher price. Explain how this public policy of providing patent pro...

See Answer

Q: Suppose you take the 30-year mortgage described in Problem 39

Suppose you take the 30-year mortgage described in Problem 39, part (a). How much will you still owe on the mortgage after 15 years? Data from Problem 39: You have just made an offer on a new home a...

See Answer

Q: You realize that the plan in Problem 42 has a flaw.

You realize that the plan in Problem 42 has a flaw. Because your income will increase over your lifetime, it would be more realistic to save less now and more later. Instead of putting the same amount...

See Answer

Q: You are 35 years old, and decide to save $5000

You are 35 years old, and decide to save $5000 each year (with the first deposit one year from now), in an account paying 8% interest per year. You will make your last deposit 30 years from now when y...

See Answer

Q: You have just turned 30 years old, have just received your

You have just turned 30 years old, have just received your MBA, and have accepted your first job. Now you must decide how much money to put into your retirement plan. The plan works as follows: Every...

See Answer

Q: Problem 45 is not very realistic because most retirement plans do not

Problem 45 is not very realistic because most retirement plans do not allow you to specify a fixed amount to contribute every year. Instead, you are required to specify a fixed percentage of your sala...

See Answer

Q: Suppose you invest $2000 today and receive $10,000

Suppose you invest $2000 today and receive $10,000 in five years. a. What is the IRR of this opportunity? b. Suppose another investment opportunity also requires $2000 upfront, but pays an equal amoun...

See Answer

Q: You are shopping for a car and read the following advertisement in

You are shopping for a car and read the following advertisement in the newspaper: “Own a new Spitfire! No money down. Four annual payments of just $10,000.” You have shopped around and know that you c...

See Answer

Q: You are considering purchasing a warehouse. The cost to purchase the

You are considering purchasing a warehouse. The cost to purchase the warehouse is $500,000. Renting the equivalent space costs $20,000 per year. If the annual interest rate is 6%, at what rate must re...

See Answer

Q: The Tillamook County Creamery Association manufactures Tillamook Cheddar Cheese. It markets

The Tillamook County Creamery Association manufactures Tillamook Cheddar Cheese. It markets this cheese in four varieties: aged 2 months, 9 months, 15 months, and 2 years. At the shop in the dairy, it...

See Answer

Q: Your daughter is currently eight years old. You anticipate that she

Your daughter is currently eight years old. You anticipate that she will be going to college in 10 years. You would like to have $100,000 in a savings account to fund her education at that time. If th...

See Answer

Q: Graff, Inc., has sales of $49,800,

Graff, Inc., has sales of $49,800, costs of $23,700, depreciation expense of $2,300, and interest expense of $1,800. If the tax rate is 22 percent, what is the operating cash flow, or OCF?

See Answer

Q: Prepare a 2021 balance sheet for Willis Corp. based on the

Prepare a 2021 balance sheet for Willis Corp. based on the following information: Cash = $165,000; Patents and copyrights = $858,000; Accounts payable = $273,000; Accounts receivable = $149,000; Tangi...

See Answer

Q: First National Bank charges 13.8 percent compounded monthly on its

First National Bank charges 13.8 percent compounded monthly on its business loans. First United Bank charges 14.1 percent compounded semiannually. As a potential borrower, which bank would you go to f...

See Answer

Q: Evergreen Credit Corp. wants to earn an effective annual return on

Evergreen Credit Corp. wants to earn an effective annual return on its consumer loans of 18.2 percent per year. The bank uses daily compounding on its loans. What interest rate is the bank required by...

See Answer

Q: What is the future value of $5,500 in 17

What is the future value of $5,500 in 17 years at an APR of 8.4 percent compounded semiannually?

See Answer

Q: Spartan Credit Bank is offering 8.3 percent compounded daily on

Spartan Credit Bank is offering 8.3 percent compounded daily on its savings accounts. If you deposit $7,500 today, how much will you have in the account in 5 years? In 10 years? In 20 years?

See Answer

Q: An investment will pay you $95,000 in 10 years

An investment will pay you $95,000 in 10 years. If the appropriate discount rate is 9 percent compounded daily, what is the present value?

See Answer

Q: Big Dom’s Pawn Shop charges an interest rate of 25.5

Big Dom’s Pawn Shop charges an interest rate of 25.5 percent per month on loans to its customers. Like all lenders, Big Dom must report an APR to consumers. What rate should the shop report? What is t...

See Answer

Q: Investment X offers to pay you $5,300 per year

Investment X offers to pay you $5,300 per year for eight years, whereas Investment Y offers to pay you $7,300 per year for five years. Which of these cash flow streams has the higher present value if...

See Answer

Q: You want to buy a new sports coupe for $84,

You want to buy a new sports coupe for $84,500, and the finance office at the dealership has quoted you an APR of 4.7 percent for a 60-month loan to buy the car. What will your monthly payments be? Wh...

See Answer

Q: One of your customers is delinquent on his accounts payable balance.

One of your customers is delinquent on his accounts payable balance. You’ve mutually agreed to a repayment schedule of $500 per month. You will charge 1.8 percent per month interest on the overdue bal...

See Answer

Q: Live Forever Life Insurance Co. is selling a perpetuity contract that

Live Forever Life Insurance Co. is selling a perpetuity contract that pays $1,500 monthly. The contract currently sells for $260,000. What is the monthly return on this investment vehicle? What is the...

See Answer

Q: Polska, Inc., is obligated to pay its creditors $10

Polska, Inc., is obligated to pay its creditors $10,300 during the year. a. What is the market value of the shareholders’ equity if assets have a market value of $11,600? b. What if assets equal $9,40...

See Answer

Q: You are planning to make monthly deposits of $475 into a

You are planning to make monthly deposits of $475 into a retirement account that pays 10 percent interest compounded monthly. If your first deposit will be made one month from now, how large will your...

See Answer

Q: In the previous problem, suppose you make $5,700

In the previous problem, suppose you make $5,700 annual deposits into the same retirement account. How large will your account balance be in 40 years?

See Answer

Q: Beginning three months from now, you want to be able to

Beginning three months from now, you want to be able to withdraw $3,000 each quarter from your bank account to cover college expenses over the next four years. If the account pays .57 percent interest...

See Answer

Q: If the appropriate discount rate for the following cash flows is 8

If the appropriate discount rate for the following cash flows is 8.5 percent compounded quarterly, what is the present value of the cash flows? Year ……………………………………… Cash Flow 1 ………………………………………………….....

See Answer

Q: If the appropriate discount rate for the following cash flows is 9

If the appropriate discount rate for the following cash flows is 9.32 percent per year, what is the present value of the cash flows? Year ……………………………………. Cash Flow 1 …………………………………………….. $2,480 2 ………...

See Answer

Q: Christie, Inc., has identified an investment project with the following

Christie, Inc., has identified an investment project with the following cash flows. If the discount rate is 6 percent, what is the future value of these cash flows in Year 4? What is the future value...

See Answer

Q: You are looking at an investment that has an effective annual rate

You are looking at an investment that has an effective annual rate of 14.3 percent. What is the effective semiannual return? The effective quarterly return? The effective monthly return?

See Answer

Q: You receive a credit card application from Shady Banks Savings and Loan

You receive a credit card application from Shady Banks Savings and Loan offering an introductory rate of 1.25 percent per year, compounded monthly for the first six months, increasing thereafter to 17...

See Answer

Q: You are planning to save for retirement over the next 30 years

You are planning to save for retirement over the next 30 years. To do this, you will invest $850 per month in a stock account and $250 per month in a bond account. The return of the stock account is e...

See Answer

Q: You have an investment that will pay you .83 percent per

You have an investment that will pay you .83 percent per month. How much will you have per dollar invested in one year? In two years?

See Answer

Q: During 2021, Raines Umbrella Corp. had sales of $865

During 2021, Raines Umbrella Corp. had sales of $865,000. Cost of goods sold, administrative and selling expenses, and depreciation expenses were $535,000, $125,000, and $170,000, respectively. In add...

See Answer

Q: You want to be a millionaire when you retire in 40 years

You want to be a millionaire when you retire in 40 years. How much do you have to save each month if you can earn an APR of 10.2 percent? How much do you have to save each month if you wait 10 years b...

See Answer

Q: You’ve just joined the investment banking firm of Dewey, Cheatum,

You’ve just joined the investment banking firm of Dewey, Cheatum, and Howe. They’ve offered you two different salary arrangements. You can have $90,000 per year for the next two years, or you can have...

See Answer

Q: You have just won the lottery and will receive $1.

You have just won the lottery and will receive $1.8 million in one year. You will receive payments for 30 years, and the payments will increase by 2.7 percent per year. If the appropriate discount rat...

See Answer

Q: Your job pays you only once a year for all the work

Your job pays you only once a year for all the work you did over the previous 12 months. Today, December 31, you just received your salary of $48,000 and you plan to spend all of it. However, you want...

See Answer

Q: What is the relationship between the value of an annuity and the

What is the relationship between the value of an annuity and the level of interest rates? Suppose you just bought an annuity with 13 annual payments of $10,000 per year at a discount rate of 10 percen...

See Answer

Q: An investment offers $3,850 per year for 15 years

An investment offers $3,850 per year for 15 years, with the first payment occurring one year from now. If the required return is 6 percent, what is the value of the investment? What would the value be...

See Answer

Q: You’re prepared to make monthly payments of $225, beginning at

You’re prepared to make monthly payments of $225, beginning at the end of this month, into an account that pays an APR of 6.5 percent compounded monthly. How many payments will you have made when your...

See Answer

Q: You want to borrow $95,000 from your local bank

You want to borrow $95,000 from your local bank to buy a new sailboat. You can afford to make monthly payments of $1,850, but no more. Assuming monthly compounding, what is the highest rate you can af...

See Answer

Q: You need a 30-year, fixed-rate mortgage to

You need a 30-year, fixed-rate mortgage to buy a new home for $245,000. Your mortgage bank will lend you the money at an APR of 4.8 percent for this 360-month loan. However, you can afford monthly pay...

See Answer

Q: The present value of the following cash flow stream is $8

The present value of the following cash flow stream is $8,200 when discounted at 9 percent annually. What is the value of the missing cash flow? Year ………………………….. Cash Flow 1 …………………………………... $2,100...

See Answer

Q: In Problem 17, suppose Raines Umbrella Corp. paid out $

In Problem 17, suppose Raines Umbrella Corp. paid out $128,000 in cash dividends. Is this possible? If net capital spending and net working capital were both zero, and if no new stock was issued durin...

See Answer

Q: You just won the TVM Lottery. You will receive $1

You just won the TVM Lottery. You will receive $1 million today plus another 10 annual payments that increase by $450,000 per year. Thus, in one year, you receive $1.45 million. In two years, you get...

See Answer

Q: You have just purchased a new warehouse. To finance the purchase

You have just purchased a new warehouse. To finance the purchase, you’ve arranged for a 30-year mortgage loan for 80 percent of the $2.6 million purchase price. The monthly payment on this loan will b...

See Answer

Q: Consider a firm with a contract to sell an asset for $

Consider a firm with a contract to sell an asset for $175,000 four years from now. The asset costs $104,600 to produce today. Given a relevant discount rate of 11 percent per year, will the firm make...

See Answer

Q: What is the value today of $5,100 per year

What is the value today of $5,100 per year, at a discount rate of 7.9 percent, if the first payment is received 6 years from today and the last payment is received 20 years from today?

See Answer

Q: A 15-year annuity pays $1,750 per month

A 15-year annuity pays $1,750 per month, and payments are made at the end of each month. If the APR is 9 percent compounded monthly for the first seven years, and APR of 6 percent compounded monthly t...

See Answer

Q: You have your choice of two investment accounts. Investment A is

You have your choice of two investment accounts. Investment A is a 13-year annuity that features end-of-month $1,600 payments and has an APR of 7.8 percent compounded monthly. Investment B is a 7 perc...

See Answer

Q: If you put up $45,000 today in exchange for

If you put up $45,000 today in exchange for a 6.4 percent, 15-year annuity, what will the annual cash flow be?

See Answer

Q: Given a discount rate of 4.6 percent per year,

Given a discount rate of 4.6 percent per year, what is the value at Date t = 7 of a perpetual stream of $7,300 payments with the first payment at Date t = 15?

See Answer

Q: A local finance company quotes an interest rate of 18.4

A local finance company quotes an interest rate of 18.4 percent on one-year loans. So, if you borrow $20,000, the interest for the year will be $3,680. Because you must repay a total of $23,680 in one...

See Answer

Q: A five-year annuity of 10 $5,500 semiannual

A five-year annuity of 10 $5,500 semiannual payments will begin 9 years from now, with the first payment coming 9.5 years from now. If the discount rate is 8 percent compounded monthly, what is the va...

See Answer

Q: Martinez Industries had the following operating results for 2021: Sales =

Martinez Industries had the following operating results for 2021: Sales = $38,072; Cost of goods sold = $27,168; Depreciation expense = $6,759; Interest expense = $3,050; Dividends paid = $2,170. At t...

See Answer

Q: Suppose you are going to receive $14,500 per year

Suppose you are going to receive $14,500 per year for five years. The appropriate interest rate is 7.1 percent. a. What is the present value of the payments if they are in the form of an ordinary ann...

See Answer

Q: You want to buy a new sports car from Muscle Motors for

You want to buy a new sports car from Muscle Motors for $64,500. The contract is in the form of a 60-month annuity due at an APR of 5.4 percent. What will your monthly payment be?

See Answer

Q: Prepare an amortization schedule for a five-year loan of $

Prepare an amortization schedule for a five-year loan of $58,500. The interest rate is 6 percent per year, and the loan calls for equal annual payments. How much interest is paid in the third year? Ho...

See Answer

Q: Rework Problem 55 assuming that the loan agreement calls for a principal

Rework Problem 55 assuming that the loan agreement calls for a principal reduction of $11,700 every year instead of equal annual payments. Problem 55: Prepare an amortization schedule for a five-year...

See Answer

Q: Bilbo Baggins wants to save money to meet three objectives. First

Bilbo Baggins wants to save money to meet three objectives. First, he would like to be able to retire 30 years from now with retirement income of $16,000 per month for 25 years, with the first payment...

See Answer

Q: After deciding to acquire a new car, you can either lease

After deciding to acquire a new car, you can either lease the car or purchase it on a three-year loan. The car you wish to buy costs $49,000. The dealer has a special leasing arrangement where you pay...

See Answer

Q: An All-Pro defensive lineman is in contract negotiations. The

An All-Pro defensive lineman is in contract negotiations. The team has offered the following salary structure: Time ……………………………………………… Salary 0 ………………………………………….. $7,900,000 1 ………………………………………….. $4,5...

See Answer

Q: Your company will generate $55,000 in annual revenue each

Your company will generate $55,000 in annual revenue each year for the next seven years from a new information database. If the appropriate discount rate is 8.2 percent, what is the present value of t...

See Answer

Q: This question illustrates what is known as discount interest. Imagine you

This question illustrates what is known as discount interest. Imagine you are discussing a loan with a somewhat unscrupulous lender. You want to borrow $20,000 for one year. The interest rate is 14.6...

See Answer

Q: You are serving on a jury. A plaintiff is suing the

You are serving on a jury. A plaintiff is suing the city for injuries sustained after a freak street sweeper accident. In the trial, doctors testified that it will be five years before the plaintiff i...

See Answer

Q: Consider the following abbreviated financial statements for Parrothead Enterprises:

Consider the following abbreviated financial statements for Parrothead Enterprises: a. What is owners’ equity for 2020 and 2021? b. What is the change in net working capital for 2...

See Answer

Q: You are looking at a one-year loan of $10

You are looking at a one-year loan of $10,000. The interest rate is quoted as 9.7 percent plus 2 points. A point on a loan is 1 percent (one percentage point) of the loan amount. Quotes similar to thi...

See Answer

Q: The interest rate on a one-year loan is quoted as

The interest rate on a one-year loan is quoted as 13 percent plus 3 points (see the previous problem). What is the EAR? Is your answer affected by the loan amount? Problem 62: You are looking at a on...

See Answer

Q: You are buying a house and will borrow $265,000

You are buying a house and will borrow $265,000 on a 30-year fixed-rate mortgage with monthly payments to finance the purchase. Your loan officer has offered you a mortgage with an APR of 4.6 percent....

See Answer

Q: In the previous problem, suppose that you believe that you will

In the previous problem, suppose that you believe that you will only live in the house for eight years before selling the house and buying another house. This means that in eight years, you will pay o...

See Answer

Q: Two banks in the area offer 30-year, $320

Two banks in the area offer 30-year, $320,000 mortgages at 4.9 percent and charge a $5,500 loan application fee. However, the application fee charged by Insecurity Bank and Trust is refundable if the...

See Answer

Q: This problem illustrates a deceptive way of quoting interest rates called add

This problem illustrates a deceptive way of quoting interest rates called add-on interest. Imagine that you see an advertisement for Crazy Judy’s Stereo City that reads something like this: “$1,500 In...

See Answer

Q: You have successfully started and operated a company for the past 10

You have successfully started and operated a company for the past 10 years. You have decided that it is time to sell your company and spend time on the beaches of Hawaii. A potential buyer is interest...

See Answer

Q: Your Christmas ski vacation was great, but it unfortunately ran a

Your Christmas ski vacation was great, but it unfortunately ran a bit over budget. All is not lost: You just received an offer in the mail to transfer your $15,000 balance from your current credit car...

See Answer

Q: If you deposit $4,900 at the end of each

If you deposit $4,900 at the end of each of the next 20 years into an account paying 10.3 percent interest, how much money will you have in the account in 20 years? How much will you have if you make...

See Answer

Q: An insurance company is offering a new policy to its customers.

An insurance company is offering a new policy to its customers. Typically, the policy is bought by a parent or grandparent for a child at the child’s birth. The purchaser (say, the parent) makes the f...

See Answer

Q: Graffiti Advertising, Inc., reported the following financial statements for the

Graffiti Advertising, Inc., reported the following financial statements for the last two years. Construct the cash flow identity for the company. Explain what each number means.

See Answer

Q: You have just arranged for a $2.55 million mortgage

You have just arranged for a $2.55 million mortgage to finance the purchase of a large tract of land. The mortgage has an APR of 5.3 percent, and it calls for monthly payments over the next 30 years....

See Answer

Q: What is the value of an investment that pays $75,

What is the value of an investment that pays $75,000 every other year forever, if the first payment occurs one year from today and the discount rate is 7 percent compounded daily? What is the value to...

See Answer

Q: You have 40 years left until retirement and want to retire with

You have 40 years left until retirement and want to retire with $4.5 million. Your salary is paid annually, and you will receive $60,000 at the end of the current year. Your salary will increase at 3...

See Answer

Q: A check-cashing store is in the business of making personal

A check-cashing store is in the business of making personal loans to walk-up customers. The store makes only one-week loans at 6.3 percent interest per week. a. What APR must the store report to its c...

See Answer

Q: You want to have $75,000 in your savings account

You want to have $75,000 in your savings account 12 years from now, and you’re prepared to make equal annual deposits into the account at the end of each year. If the account pays 6.4 percent interest...

See Answer

Q: Pursell Bank offers you a five-year loan for $100

Pursell Bank offers you a five-year loan for $100,000 at an annual interest rate of 6.8 percent. What will your annual loan payment be?

See Answer

Q: Timmy Tappan is single and had $189,000 in taxable

Timmy Tappan is single and had $189,000 in taxable income. Using the rates from Table 2.3 in the chapter, calculate his income taxes. What is the average tax rate? What is the marginal tax rate?

See Answer

Q: The 2020 balance sheet of Dugan, Inc., showed current assets

The 2020 balance sheet of Dugan, Inc., showed current assets of $5,320 and current liabilities of $2,510. The 2021 balance sheet showed current assets of $5,970 and current liabilities of $3,240. What...

See Answer

Q: Given the information for Osaka’s Tennis Shop, Inc., in Problems

Given the information for Osaka’s Tennis Shop, Inc., in Problems 9 and 10, suppose you also know that the firm’s net capital spending for 2021 was $1.5 million and that the firm reduced its net workin...

See Answer

Q: Bing, Inc., has current assets of $5,400

Bing, Inc., has current assets of $5,400, net fixed assets of $28,100, current liabilities of $4,100, and long-term debt of $10,600. What is the value of the shareholders’ equity account for this firm...

See Answer

Q: Firm A and Firm B have debt-total asset ratios of

Firm A and Firm B have debt-total asset ratios of 60 percent and 35 percent, respectively, and returns on total assets of 4.5 percent and 8 percent, respectively. Which firm has a greater return on eq...

See Answer

Q: Nataro, Inc., has sales of $742,000,

Nataro, Inc., has sales of $742,000, costs of $316,000, depreciation expense of $39,000, interest expense of $34,000, and a tax rate of 21 percent. What is the net income for this firm?

See Answer

Q: Suppose the firm in Problem 3 had 75,000 shares of

Suppose the firm in Problem 3 had 75,000 shares of common stock outstanding. What is the earnings per share, or EPS, figure? What is the dividends per share figure? Problem 3: Suppose the firm in Pro...

See Answer

Q: There are four pieces to an annuity present value. What are

There are four pieces to an annuity present value. What are they?

See Answer

Q: As you increase the length of time involved, what happens to

As you increase the length of time involved, what happens to the present value of an annuity? What happens to the future value?

See Answer

Q: Suppose the firm in Problem 2 paid out $125,000

Suppose the firm in Problem 2 paid out $125,000 in cash dividends. What is the addition to retained earnings? Problem 2: Nataro, Inc., has sales of $742,000, costs of $316,000, depreciation expense...

See Answer

Q: What do you think about the state lottery discussed in the chapter

What do you think about the state lottery discussed in the chapter advertising a $500,000 prize when the lump sum option is $250,000? Is it deceptive advertising?

See Answer

Q: If you were an athlete negotiating a contract, would you want

If you were an athlete negotiating a contract, would you want a big signing bonus payable immediately and smaller payments in the future, or vice versa? How about looking at it from the team’s perspec...

See Answer

Q: Use the following information for Taco Swell, Inc., (assume

Use the following information for Taco Swell, Inc., (assume the tax rate is 21 percent): Draw up an income statement and balance sheet for this company for 2020 and 2021.

See Answer

Q: Use the following information for Taco Swell, Inc., (assume

Use the following information for Taco Swell, Inc., (assume the tax rate is 21 percent): For 2021, calculate the cash flow from assets, cash flow to creditors, and cash flow to stockholders.

See Answer

Q: Wallace Driving School’s 2020 balance sheet showed net fixed assets of $

Wallace Driving School’s 2020 balance sheet showed net fixed assets of $2.3 million, and the 2021 balance sheet showed net fixed assets of $3.1 million. The company’s 2021 income statement showed a de...

See Answer

Q: Prince Albert Canning PLC had a net loss of £26,

Prince Albert Canning PLC had a net loss of £26,382 on sales of £315,650. What was the company’s profit margin? Does the fact that these figures are quoted in a foreign currency make any difference? W...

See Answer

Q: Some recent financial statements for Smolira Golf Corp. follow. Use

Some recent financial statements for Smolira Golf Corp. follow. Use this information to work. Find the following financial ratios for Smolira Golf Corp. (use year-end figures rather than average va...

See Answer

Q: Some recent financial statements for Smolira Golf Corp. follow. Use

Some recent financial statements for Smolira Golf Corp. follow. Use this information to work. Construct the DuPont identity for Smolira Golf Corp.

See Answer

Q: Some recent financial statements for Smolira Golf Corp. follow. Use

Some recent financial statements for Smolira Golf Corp. follow. Use this information to work. Prepare the 2021 statement of cash flows for Smolira Golf Corp.

See Answer

Q: Some recent financial statements for Smolira Golf Corp. follow. Use

Some recent financial statements for Smolira Golf Corp. follow. Use this information to work. Smolira Golf Corp. has 15,000 shares of common stock outstanding, and the market price for a share of s...

See Answer

Q: Some recent financial statements for Smolira Golf Corp. follow. Use

Some recent financial statements for Smolira Golf Corp. follow. Use this information to work. What is Tobin’s Q for Smolira Golf? What assumptions are you making about the book va...

See Answer

Q: SDJ, Inc., has net working capital of $2,

SDJ, Inc., has net working capital of $2,630, current liabilities of $5,970, and inventory of $3,860. What is the current ratio? What is the quick ratio?

See Answer

Q: Dexter, Inc., had a cost of goods sold of $

Dexter, Inc., had a cost of goods sold of $75,318. At the end of the year, the accounts payable balance was $18,452. How long on average did it take the company to pay off its suppliers during the yea...

See Answer

Q: The market value of the equity of Nina, Inc., is

The market value of the equity of Nina, Inc., is $755,000. The balance sheet shows $63,000 in cash and $220,000 in debt, while the income statement has EBIT of $95,000 and a total of $178,000 in depre...

See Answer

Q: Kodi Company has a debt-equity ratio of .63.

Kodi Company has a debt-equity ratio of .63. Return on assets is 8.4 percent, and total equity is $645,000. What is the equity multiplier? Return on equity? Net income?

See Answer

Q: Specialized ratios are sometimes used in specific industries. For example,

Specialized ratios are sometimes used in specific industries. For example, the book-to-bill ratio is closely watched for semiconductor manufacturers. A ratio of .93 indicates that for every $100 worth...

See Answer

Q: Just Dew It Corporation reports the following balance sheet information for 2020

Just Dew It Corporation reports the following balance sheet information for 2020 and 2021. Use this information to work. Prepare the 2020 and 2021 common-size balance sheets for Just Dew It.

See Answer

Q: Just Dew It Corporation reports the following balance sheet information for 2020

Just Dew It Corporation reports the following balance sheet information for 2020 and 2021. Use this information to work. Prepare the 2021 common-base year balance sheet for Just Dew It.

See Answer

Q: Just Dew It Corporation reports the following balance sheet information for 2020

Just Dew It Corporation reports the following balance sheet information for 2020 and 2021. Use this information to work. Prepare the 2021 combined common-size, common-base year balance sheet for Jus...

See Answer

Q: Just Dew It Corporation reports the following balance sheet information for 2020

Just Dew It Corporation reports the following balance sheet information for 2020 and 2021. Use this information to work. For each account on this company’s balance sheet, show the...

See Answer

Q: Just Dew It Corporation reports the following balance sheet information for 2020

Just Dew It Corporation reports the following balance sheet information for 2020 and 2021. Use this information to work. Based on the balance sheets given for Just Dew It, calculate the following fi...

See Answer

Q: Y3K, Inc., has sales of $5,987,

Y3K, Inc., has sales of $5,987, total assets of $2,532, and a debt-equity ratio of .57. If its return on equity is 11 percent, what is its net income?

See Answer

Q: A company has net income of $213,700, a

A company has net income of $213,700, a profit margin of 7.1 percent, and an accounts receivable balance of $126,385. Assuming 65 percent of sales are on credit, what is the company’s days’ sales in r...

See Answer

Q: Denver, Inc., has sales of $14.2 million

Denver, Inc., has sales of $14.2 million, total assets of $11.3 million, and total debt of $4.9 million. If the profit margin is 5 percent, what is net income? What is ROA? What is ROE?

See Answer

Q: The Mikado Company has a long-term debt ratio of .

The Mikado Company has a long-term debt ratio of .35 and a current ratio of 1.45. Current liabilities are $1,140, sales are $8,370, profit margin is 8.3 percent, and ROE is 16.5 percent. What is the a...

See Answer

Q: In response to complaints about high prices, a grocery chain runs

In response to complaints about high prices, a grocery chain runs the following advertising campaign: “If you pay your child $1.25 to go buy $50 worth of groceries, then your child m...

See Answer

Q: The 2020 balance sheet of Osaka’s Tennis Shop, Inc., showed

The 2020 balance sheet of Osaka’s Tennis Shop, Inc., showed long-term debt of $2.25 million, and the 2021 balance sheet showed long-term debt of $2.66 million. The 2021 income statement showed an inte...

See Answer

Q: Whipporwill, Inc.’s, net income for the most recent

Whipporwill, Inc.’s, net income for the most recent year was $19,382. The tax rate was 21 percent. The firm paid $3,681 in total interest expense and deducted $4,738 in depreciation expense. What was...

See Answer

Q: Cage Corp. has current liabilities of $415,000,

Cage Corp. has current liabilities of $415,000, a quick ratio of .86, inventory turnover of 10.3, and a current ratio of 1.34. What is the cost of goods sold for the company?

See Answer

Q: Dahlia Corp. has a current accounts receivable balance of $513

Dahlia Corp. has a current accounts receivable balance of $513,260. Credit sales for the year just ended were $4,986,340. What is the receivables turnover? The days’ sales in receivables? How long did...

See Answer

Q: The Top Corporation has ending inventory of $426,287,

The Top Corporation has ending inventory of $426,287, and cost of goods sold for the year just ended was $4,738,216. What is the inventory turnover? The days’ sales in inventory? How long on average d...

See Answer

Q: Mobius, Inc., has a total debt ratio of .57

Mobius, Inc., has a total debt ratio of .57. What is its debt-equity ratio? What is its equity multiplier?

See Answer

Q: Bolton Corp. had additions to retained earnings for the year just

Bolton Corp. had additions to retained earnings for the year just ended of $435,000. The firm paid out $245,000 in cash dividends, and it has ending total equity of $5.7 million. If the company curren...

See Answer

Q: If Rogers, Inc., has an equity multiplier of 1.

If Rogers, Inc., has an equity multiplier of 1.43, total asset turnover of 1.87, and a profit margin of 6.05 percent, what is its ROE?

See Answer

Q: Jackson Corp. has a profit margin of 5.8 percent

Jackson Corp. has a profit margin of 5.8 percent, total asset turnover of 1.75, and ROE of 13.85 percent. What is this firm’s debt-equity ratio?

See Answer

Q: Based only on the following information for Ortiz Corp., did cash

Based only on the following information for Ortiz Corp., did cash go up or down? By how much? Classify each event as a source or use of cash. Decrease in inventory ……………………………….. $425 Decrease in acc...

See Answer

Q: Consider the following simplified financial statements for the Wesney Corporation (assuming

Consider the following simplified financial statements for the Wesney Corporation (assuming no income taxes): The company has predicted a sales increase of 15 percent. It has predicted that every it...

See Answer

Q: The 2020 balance sheet of Osaka’s Tennis Shop, Inc., showed

The 2020 balance sheet of Osaka’s Tennis Shop, Inc., showed $780,000 in the common stock account and $4.78 million in the additional paid-in surplus account. The 2021 balance sheet showed $965,000 and...

See Answer

Q: The balance sheet for the Heir Jordan Corporation follows. Based on

The balance sheet for the Heir Jordan Corporation follows. Based on this information and the income statement in the previous problem, supply the missing information using the percentage of sales appr...

See Answer

Q: From the previous two questions, prepare a pro forma balance sheet

From the previous two questions, prepare a pro forma balance sheet showing EFN, assuming an increase in sales of 15 percent, no new external debt or equity financing, and a constant payout ratio. Pro...

See Answer

Q: If Fairlane Co. has an ROA of 8.3 percent

If Fairlane Co. has an ROA of 8.3 percent and a payout ratio of 35 percent, what is its internal growth rate?

See Answer

Q: If Premier Corp. has an ROE of 14.1 percent

If Premier Corp. has an ROE of 14.1 percent and a payout ratio of 25 percent, what is its sustainable growth rate?

See Answer

Q: Based on the following information, calculate the sustainable growth rate for

Based on the following information, calculate the sustainable growth rate for Kayla’s Heavy Equipment: Profit margin = 7.3% Capital intensity ratio = .95 Debt-equity ratio = 1.05 Net income = $84,000...

See Answer

Q: Assuming the following ratios are constant, what is the sustainable growth

Assuming the following ratios are constant, what is the sustainable growth rate? Total asset turnover = 3.20 Profit margin = 5.2% Equity multiplier = .95 Payout ratio = 35%

See Answer

Q: Pinnacle Mfg., Inc., is currently operating at only 94 percent

Pinnacle Mfg., Inc., is currently operating at only 94 percent of fixed asset capacity. Current sales are $830,000. How fast can sales grow before any new fixed assets are needed?

See Answer

Q: For the company in Problem 16, suppose fixed assets are $

For the company in Problem 16, suppose fixed assets are $590,000 and sales are projected to grow to $910,000. How much in new fixed assets are required to support this growth in sales? Assume the comp...

See Answer

Q: Gamgee Co. wishes to maintain a growth rate of 11 percent

Gamgee Co. wishes to maintain a growth rate of 11 percent per year, a debt-equity ratio of .75, and a dividend payout ratio of 25 percent. The ratio of total assets to sales is constant at .65. What p...

See Answer

Q: A firm wishes to maintain an internal growth rate of 6.

A firm wishes to maintain an internal growth rate of 6.4 percent and a dividend payout ratio of 25 percent. The current profit margin is 5.7 percent, and the firm uses no external financing sources. W...

See Answer

Q: What happens to the future value of a perpetuity if interest rates

What happens to the future value of a perpetuity if interest rates increase? What if interest rates decrease?

See Answer

Q: In Question 1, assume the company pays out half of net

In Question 1, assume the company pays out half of net income in the form of a cash dividend. Costs and assets vary with sales, but debt and equity do not. Prepare the pro forma statements and determi...

See Answer

Q: Based on the following information, calculate the sustainable growth rate for

Based on the following information, calculate the sustainable growth rate for Hendrix Guitars, Inc.: Profit margin = 5.9% Total asset turnover = 1.15 Total debt ratio = .45 Payout ratio = 40%

See Answer

Q: You’ve collected the following information about Caccamisse, Inc.: Sales

You’ve collected the following information about Caccamisse, Inc.: Sales = $255,000 Net income = $19,200 Dividends = $7,500 Total debt = $67,000 Total equity = $77,000 What is the sustainable growth r...

See Answer

Q: Deprey, Inc., had equity of $153,000 at

Deprey, Inc., had equity of $153,000 at the beginning of the year. At the end of the year, the company had total assets of $215,000. During the year, the company sold no new equity. Net income for the...

See Answer

Q: Calculate the internal growth rate for the company in Problem 22.

Calculate the internal growth rate for the company in Problem 22. Now calculate the internal growth rate using ROA × b for both beginning of period and end of period total assets. What do you observe?...

See Answer

Q: The most recent financial statements for Crosby, Inc., follow.

The most recent financial statements for Crosby, Inc., follow. Sales for 2021 are projected to grow by 20 percent. Interest expense will remain constant; the tax rate and the dividend payout rate will...

See Answer

Q: In Problem 24, suppose the firm was operating at only 80

In Problem 24, suppose the firm was operating at only 80 percent capacity in 2020. What is EFN now? Problem 24: The most recent financial statements for Crosby, Inc., follow. Sales for 2021 are proje...

See Answer

Q: In Problem 24, suppose the firm wishes to keep its debt

In Problem 24, suppose the firm wishes to keep its debt-equity ratio constant. What is EFN now? Problem 24: The most recent financial statements for Crosby, Inc., follow. Sales for 2021 are projected...

See Answer

Q: Redo Problem 24 using sales growth rates of 15 and 25 percent

Redo Problem 24 using sales growth rates of 15 and 25 percent in addition to 20 percent. Illustrate graphically the relationship between EFN and the growth rate, and use this graph to determine the re...

See Answer

Q: Redo Problem 26 using sales growth rates of 30 and 35 percent

Redo Problem 26 using sales growth rates of 30 and 35 percent in addition to 20 percent. Assume the firm wishes to maintain its debt-equity ratio. Illustrate graphically the relationship between EFN a...

See Answer

Q: Klingon Widgets, Inc., purchased new cloaking machinery three years ago

Klingon Widgets, Inc., purchased new cloaking machinery three years ago for $6 million. The machinery can be sold to the Romulans today for $5.4 million. Klingon’s current balance sheet shows net fixe...

See Answer

Q: Tinsley, Inc., wishes to maintain a growth rate of 12

Tinsley, Inc., wishes to maintain a growth rate of 12 percent per year and a debt-equity ratio of .55. The profit margin is 6.2 percent, and the ratio of total assets to sales is constant at 1.05. Is...

See Answer

Q: The most recent financial statements for Camryn, Inc., are shown

The most recent financial statements for Camryn, Inc., are shown here (assuming no income taxes): Assets and costs are proportional to sales. Debt and equity are not. No dividends are paid. Next yea...

See Answer

Q: Based on the result in Problem 30, show that the internal

Based on the result in Problem 30, show that the internal and sustainable growth rates are as given in the chapter. Problem 30: Define the following: S = Previous year’s sales A = Total assets E = T...

See Answer

Q: The most recent financial statements for Mixton, Inc., are shown

The most recent financial statements for Mixton, Inc., are shown here: Assets and costs are proportional to sales. Debt and equity are not. A dividend of $1,400 was paid, and the company wishes to m...

See Answer

Q: The most recent financial statements for Assouad, Inc., are shown

The most recent financial statements for Assouad, Inc., are shown here: Assets, costs, and current liabilities are proportional to sales. Long-term debt and equity are not. The company maintains a c...

See Answer

Q: The most recent financial statements for Mandy Co. are shown here

The most recent financial statements for Mandy Co. are shown here: Assets and costs are proportional to sales. Debt and equity are not. The company maintains a constant 30 percent dividend payout ra...

See Answer

Q: For the company in Problem 6, what is the sustainable growth

For the company in Problem 6, what is the sustainable growth rate? Problem 6: The most recent financial statements for Mandy Co. are shown here: Assets and costs are proportional to sales. Debt and...

See Answer

Q: The most recent financial statements for Tran Co. are shown here

The most recent financial statements for Tran Co. are shown here: Assets and costs are proportional to sales. The company maintains a constant 40 percent dividend payout ratio and a constant debt-eq...

See Answer

Q: Consider the following income statement for the Heir Jordan Corporation:

Consider the following income statement for the Heir Jordan Corporation: A 20 percent growth rate in sales is projected. Prepare a pro forma income statement assuming costs vary with sales and the d...

See Answer

Q: First City Bank pays 7.5 percent simple interest on its

First City Bank pays 7.5 percent simple interest on its savings account balances, whereas Second City Bank pays 7.5 percent interest compounded annually. If you made a deposit of $8,600 in each bank,...

See Answer

Q: Nightwish Corp. shows the following information on its 2021 income statement

Nightwish Corp. shows the following information on its 2021 income statement: Sales = $336,000; Costs = $194,700; Other expenses = $9,800; Depreciation expense = $20,600; Interest expense = $14,200; T...

See Answer

Q: Imprudential, Inc., has an unfunded pension liability of $450

Imprudential, Inc., has an unfunded pension liability of $450 million that must be paid in 20 years. To assess the value of the firm’s stock, financial analysts want to discount this liability back to...

See Answer

Q: You have just received notification that you have won the $2

You have just received notification that you have won the $2 million first prize in the Centennial Lottery. However, the prize will be awarded on your 100th birthday (assuming you’re around to collect...

See Answer

Q: In 1895, the first U.S. Open Golf Championship

In 1895, the first U.S. Open Golf Championship was held. The winner’s prize money was $150. In 2019, the winner’s check was $2.25 million. What was the percentage increase per year in the winner’s che...

See Answer

Q: The “Brasher doubloon,” which was featured in the plot of

The “Brasher doubloon,” which was featured in the plot of the Raymond Chandler novel, The High Window, was sold at auction in 2018 for a reported $5.5 million. The coin had a face value of $15 when it...

See Answer

Q: Refer back to the Series EE savings bonds we discussed at the

Refer back to the Series EE savings bonds we discussed at the very beginning of the chapter. a. Assuming you purchased a $50 face value bond, what is the exact rate of return you would earn if you hel...

See Answer

Q: Suppose you are still committed to owning a $275,000

Suppose you are still committed to owning a $275,000 Ferrari (see Problem 9). If you believe your mutual fund can achieve an annual rate of return of 11.2 percent and you want to buy the car in 9 year...

See Answer

Q: For each of the following, compute the future value:

For each of the following, compute the future value:

See Answer

Q: You expect to receive $10,000 at graduation in two

You expect to receive $10,000 at graduation in two years. You plan on investing it at 11 percent until you have $60,000. How long will you wait from now?

See Answer

Q: For each of the following, compute the present value:

For each of the following, compute the present value:

See Answer

Q: Solve for the unknown interest rate in each of the following:

Solve for the unknown interest rate in each of the following:

See Answer

Q: Given the following information for Troiano Pizza Co., calculate the depreciation

Given the following information for Troiano Pizza Co., calculate the depreciation expense: Sales = $76,800; Costs = $36,900; Addition to retained earnings = $6,800; Dividends paid = $2,370; Interest e...

See Answer

Q: Solve for the unknown number of years in each of the following

Solve for the unknown number of years in each of the following:

See Answer

Q: Assume the total cost of a college education will be $325

Assume the total cost of a college education will be $325,000 when your child enters college in 18 years. You presently have $85,000 to invest. What annual rate of interest must you earn on your inves...

See Answer

Q: At 4.3 percent interest, how long does it take

At 4.3 percent interest, how long does it take to double your money? To quadruple it?

See Answer

Q: According to the Census Bureau, in October 2019, the average

According to the Census Bureau, in October 2019, the average house price in the United States was $368,600. In October 2000, the average price was $197,700. What was the annual increase in the price o...

See Answer

Q: You’re trying to save to buy a new $275,000

You’re trying to save to buy a new $275,000 Ferrari. You have $50,000 today that can be invested at your bank. The bank pays 4.8 percent annual interest on its accounts. How long will it be before you...

See Answer

Q: Mendez Co. has identified an investment project with the following cash

Mendez Co. has identified an investment project with the following cash flows. If the discount rate is 10 percent, what is the present value of these cash flows? What is the present value at 18 percen...

See Answer

Q: The Maybe Pay Life Insurance Co. is trying to sell you

The Maybe Pay Life Insurance Co. is trying to sell you an investment policy that will pay you and your heirs $30,000 per year forever. If the required return on this investment is 5.6 percent, how muc...

See Answer

Q: In the previous problem, suppose a sales associate told you the

In the previous problem, suppose a sales associate told you the policy costs $525,000. At what interest rate would this be a fair deal? Problem 10: The Maybe Pay Life Insurance Co. is trying to sell...

See Answer

Q: Find the EAR in each of the following cases:

Find the EAR in each of the following cases:

See Answer

Q: Find the APR, or stated rate, in each of the

Find the APR, or stated rate, in each of the following cases:

See Answer

Q: Given that Zoom was up by about 250 percent for in the

Given that Zoom was up by about 250 percent for in the first half of 2020, why didn’t all investors hold this stock?

See Answer

Q: In the chapter opener, we mentioned Toyota’s decision to invest $

In the chapter opener, we mentioned Toyota’s decision to invest $13 billion to increase production at five U.S. plants. Toyota apparently felt that it would be better able to compete and create value...

See Answer

Q: In the previous problem, suppose your required return on the project

In the previous problem, suppose your required return on the project is 11 percent and your pretax cost savings are $150,000 per year. Will you accept the project? What if the pretax cost savings are...

See Answer

Q: A five-year project has an initial fixed asset investment of

A five-year project has an initial fixed asset investment of $345,000, an initial NWC investment of $25,000, and an annual OCF of −$41,000. The fixed asset is fully depreciated over the life of the pr...

See Answer

Q: You are evaluating two different silicon wafer milling machines. The Techron

You are evaluating two different silicon wafer milling machines. The Techron I costs $265,000, has a three-year life, and has pretax operating costs of $74,000 per year. The Techron II costs $445,000,...

See Answer

Q: Winnebagel Corp. currently sells 20,000 motor homes per year

Winnebagel Corp. currently sells 20,000 motor homes per year at $103,000 each and 14,000 luxury motor coaches per year at $155,000 each. The company wants to introduce a new portable camper to fill ou...

See Answer

Q: Martin Enterprises needs someone to supply it with 110,000 cartons

Martin Enterprises needs someone to supply it with 110,000 cartons of machine screws per year to support its manufacturing needs over the next five years, and you’ve decided to bid on the contract. It...

See Answer

Q: Tanaka Machine Shop is considering a fouryear project to improve its production

Tanaka Machine Shop is considering a fouryear project to improve its production efficiency. Buying a new machine press for $445,000 is estimated to result in $160,000 in annual pretax cost savings. Th...

See Answer

Q: Eggz, Inc., is considering the purchase of new equipment that

Eggz, Inc., is considering the purchase of new equipment that will allow the company to collect loose hen feathers for sale. The equipment will cost $525,000 and will be eligible for 100 percent bonus...

See Answer

Q: Rust Industrial Systems Company is trying to decide between two different conveyor

Rust Industrial Systems Company is trying to decide between two different conveyor belt systems. System A costs $295,000, has a four-year life, and requires $77,000 in pretax annual operating costs. S...

See Answer

Q: Suppose in the previous problem that the company always needs a conveyor

Suppose in the previous problem that the company always needs a conveyor belt system; when one wears out, it must be replaced. Which project should the firm choose now? Problem 23: Rust Industrial S...

See Answer

Q: Consider a project to supply 100 million postage stamps per year to

Consider a project to supply 100 million postage stamps per year to the U.S. Postal Service for the next five years. You have an idle parcel of land available that cost $750,000 five years ago; if the...

See Answer

Q: Union Local School District has a bond outstanding with a coupon rate

Union Local School District has a bond outstanding with a coupon rate of 2.9 percent paid semiannually and 16 years to maturity. The yield to maturity on this bond is 2.7 percent, and the bond has a p...

See Answer

Q: In the previous problem, suppose you were going to use a

In the previous problem, suppose you were going to use a three-year MACRS depreciation schedule for your manufacturing equipment and you could keep working capital investments down to only $25,000 per...

See Answer

Q: Vandelay Industries is considering the purchase of a new machine for the

Vandelay Industries is considering the purchase of a new machine for the production of latex. Machine A costs $2.1 million and will last for six years. Variable costs are 35 percent of sales, and fixe...

See Answer

Q: The previous problem suggests that using LEDs instead of incandescent bulbs is

The previous problem suggests that using LEDs instead of incandescent bulbs is a no-brainer. However, electricity costs actually vary quite a bit depending on location and user type (you can get infor...

See Answer

Q: A proposed new investment has projected sales of $515,000

A proposed new investment has projected sales of $515,000. Variable costs are 36 percent of sales, and fixed costs are $173,000; depreciation is $46,000. Prepare a pro forma income statement assuming...

See Answer

Q: The previous two problems suggest that using LEDs is a good idea

The previous two problems suggest that using LEDs is a good idea from a purely financial perspective unless you live in an area where power is relatively inexpensive, but there is another wrinkle. Sup...

See Answer

Q: Before LEDs became a popular replacement for incandescent light bulbs, compact

Before LEDs became a popular replacement for incandescent light bulbs, compact fluorescent lamps (CFLs) were hailed as the new generation of lighting. However, CFLs had even more wrinkles. In no parti...

See Answer

Q: Your small remodeling business has two work vehicles. One is a

Your small remodeling business has two work vehicles. One is a small passenger car used for job site visits and for other general business purposes. The other is a heavy truck used to haul equipment....

See Answer

Q: In the previous problem, suppose you drive the truck x miles

In the previous problem, suppose you drive the truck x miles per year. How many miles would you have to drive the car before upgrading the car would be the better choice? Problem 32: Your small remod...

See Answer

Q: You have been hired as a consultant for Pristine Urban-Tech

You have been hired as a consultant for Pristine Urban-Tech Zither, Inc. (PUTZ), manufacturers of fine zithers. The market for zithers is growing quickly. The company bought some land three years ago...

See Answer

Q: In the previous problem, suppose the fixed asset actually qualifies for

In the previous problem, suppose the fixed asset actually qualifies for 100 percent bonus depreciation in the first year. What is the new NPV? Problem 34: You have been hired as a consultant for Pri...

See Answer

Q: If Treasury bills are currently paying 4.6 percent and the

If Treasury bills are currently paying 4.6 percent and the inflation rate is 1.9 percent, what is the approximate real rate of interest? The exact real rate?

See Answer

Q: Aria Acoustics, Inc. (AAI), projects unit sales for

Aria Acoustics, Inc. (AAI), projects unit sales for a new seven-octave voice emulation implant as follows: Year ………………………….. Unit Sales 1 …………………………………… 71,000 2 …………………………………. 84,000 3 …………………………………...

See Answer

Q: A proposed cost-saving device has an installed cost of $

A proposed cost-saving device has an installed cost of $905,000. The device will be used in a five-year project but is classified as three-year MACRS property for tax purposes. The required initial ne...

See Answer

Q: To solve the bid price problem presented in the text, we

To solve the bid price problem presented in the text, we set the project NPV equal to zero and found the required price using the definition of OCF. Thus the bid price represents a financial break-eve...

See Answer

Q: Your company has been approached to bid on a contract to sell

Your company has been approached to bid on a contract to sell 5,000 voice recognition (VR) computer keyboards per year for four years. Due to technological improvements, beyond that time they will be...

See Answer

Q: Consider the following income statement: Sales ………………….…………………….. $704

Consider the following income statement: Sales ………………….…………………….. $704,600 Costs …………….…..………………………… 527,300 Depreciation ……..………………………….. 82,100 EBIT …………..…..……………………………………. ? Taxes (22%) …..……...

See Answer

Q: Suppose we are thinking about replacing an old computer with a new

Suppose we are thinking about replacing an old computer with a new one. The old one cost us $1.4 million; the new one will cost $1.7 million. The new machine will be depreciated straight-line to zero...

See Answer

Q: A proposed new project has projected sales of $215,000

A proposed new project has projected sales of $215,000, costs of $104,000, and depreciation of $25,300. The tax rate is 23 percent. Calculate operating cash flow using the four different approaches de...

See Answer

Q: A piece of newly purchased industrial equipment costs $1.475

A piece of newly purchased industrial equipment costs $1.475 million and is classified as seven-year property under MACRS. Calculate the annual depreciation allowances and end-of-the-year book values...

See Answer

Q: Consider an asset that costs $745,000 and is depreciated

Consider an asset that costs $745,000 and is depreciated straight-line to zero over its eight-year tax life. The asset is to be used in a five-year project; at the end of the project, the asset can be...

See Answer

Q: An asset used in a four-year project falls in the

An asset used in a four-year project falls in the five-year MACRS class for tax purposes. The asset has an acquisition cost of $5.7 million and will be sold for $1.8 million at the end of the project....

See Answer

Q: Suppose the real rate is 1.8 percent and the inflation

Suppose the real rate is 1.8 percent and the inflation rate is 2.7 percent. What rate would you expect to see on a Treasury bill?

See Answer

Q: Esfandairi Enterprises is considering a new three-year expansion project that

Esfandairi Enterprises is considering a new three-year expansion project that requires an initial fixed asset investment of $2.18 million. The fixed asset will be depreciated straight-line to zero ove...

See Answer

Q: Night Shades, Inc., manufactures biotech sunglasses. The variable materials

Night Shades, Inc., manufactures biotech sunglasses. The variable materials cost is $12.14 per unit, and the variable labor cost is $6.89 per unit. a. What is the variable cost per unit? b. Suppose th...

See Answer

Q: Consider a project with the following data: Accounting break-even

Consider a project with the following data: Accounting break-even quantity = 14,300 units; cash break-even quantity = 9,700 units; life = 5 years; fixed costs = $205,000; variable costs = $19 per unit...

See Answer

Q: At an output level of 40,000 units, you calculate

At an output level of 40,000 units, you calculate that the degree of operating leverage is 3.19. If output rises to 44,000 units, what will the percentage change in operating cash flow be? Will the ne...

See Answer

Q: In the previous problem, suppose fixed costs are $183,

In the previous problem, suppose fixed costs are $183,000. What is the operating cash flow at 38,000 units? The degree of operating leverage? Problem 11: At an output level of 40,000 units, you calcu...

See Answer

Q: A proposed project has fixed costs of $76,000 per

A proposed project has fixed costs of $76,000 per year. The operating cash flow at 9,200 units is $108,700. Ignoring the effect of taxes, what is the degree of operating leverage? If units sold rise f...

See Answer

Q: At an output level of 14,500 units, you have

At an output level of 14,500 units, you have calculated that the degree of operating leverage is 3.41. The operating cash flow is $81,000 in this case. Ignoring the effect of taxes, what are fixed cos...

See Answer

Q: In the previous problem, what will be the new degree of

In the previous problem, what will be the new degree of operating leverage in each case? Problem 14: At an output level of 14,500 units, you have calculated that the degree of operating leverage is 3...

See Answer

Q: Consider a four-year project with the following information: Initial

Consider a four-year project with the following information: Initial fixed asset investment = $655,000; straight-line depreciation to zero over the four-year life; zero salvage value; price = $28; va...

See Answer

Q: In the previous problem, what is the degree of operating leverage

In the previous problem, what is the degree of operating leverage at the given level of output? What is the degree of operating leverage at the accounting break-even level of output? Problem 17: Cons...

See Answer

Q: An investment offers a total return of 11.7 percent over

An investment offers a total return of 11.7 percent over the coming year. Janice Yellen thinks the total real return on this investment will be only 9 percent. What does Janice believe the inflation r...

See Answer

Q: You are considering a new product launch. The project will cost

You are considering a new product launch. The project will cost $1,675,000, have a four-year life, and have no salvage value; depreciation is straight-line to zero. Sales are projected at 195 units pe...

See Answer

Q: Ojo Outerwear Corporation can manufacture mountain climbing shoes for $45.

Ojo Outerwear Corporation can manufacture mountain climbing shoes for $45.17 per pair in variable raw material costs and $29.73 per pair in variable labor expense. The shoes sell for $210 per pair. La...

See Answer

Q: McGilla Golf has decided to sell a new line of golf clubs

McGilla Golf has decided to sell a new line of golf clubs. The clubs will sell for $815 per set and have a variable cost of $365 per set. The company has spent $150,000 for a marketing study that dete...

See Answer

Q: In the previous problem, you feel that the values are accurate

In the previous problem, you feel that the values are accurate to within only ± 10 percent. What are the best-case and worst-case NPVs? Problem 20: McGilla Golf has decided to sell a new line of golf...

See Answer

Q: In Problem 20, McGilla Golf would like to know the sensitivity

In Problem 20, McGilla Golf would like to know the sensitivity of NPV to changes in the price of the new clubs and the quantity of new clubs sold. What is the sensitivity of the NPV to each of these v...

See Answer

Q: Hybrid cars are touted as a “green” alternative; however

Hybrid cars are touted as a “green” alternative; however, the financial aspects of hybrid ownership are not as clear. Consider the 2019 Toyota RAV4 Hybrid, which had a list price of $5,900 (including...

See Answer

Q: This problem concerns the effect of taxes on the various break-

This problem concerns the effect of taxes on the various break-even measures. a. Show that, when we consider taxes, the general relationship between operating cash flow, OCF, and sales volume, Q, can...

See Answer

Q: Consider a project to supply Detroit with 20,000 tons of

Consider a project to supply Detroit with 20,000 tons of machine screws annually for automobile production. You will need an initial $3.1 million investment in threading equipment to get the project s...

See Answer

Q: In Problem 27, suppose you’re confident about your own projections,

In Problem 27, suppose you’re confident about your own projections, but you’re a little unsure about Detroit’s actual machine screw requirement. What is the sensitivity of the project OCF to changes i...

See Answer

Q: Use the results of Problem 25 to find the accounting, cash

Use the results of Problem 25 to find the accounting, cash, and financial break-even quantities for the company in Problem 27. Problem 25: This problem concerns the effect of taxes on the various bre...

See Answer

Q: Say you own an asset that had a total return last year

Say you own an asset that had a total return last year of 14.1 percent. If the inflation rate last year was 2.83 percent, what was your real return?

See Answer

Q: Stinnett Transmissions, Inc., has the following estimates for its new

Stinnett Transmissions, Inc., has the following estimates for its new gear assembly project: Price = $1,220 per unit; variable costs = $380 per unit; fixed costs = $3.75 million; quantity = 90,000 uni...

See Answer

Q: Use the results of Problem 26 to find the degree of operating

Use the results of Problem 26 to find the degree of operating leverage for the company in Problem 27 at the base-case output level of 20,000 tons. How does this number compare to the sensitivity figur...

See Answer

Q: We are evaluating a project that costs $845,000,

We are evaluating a project that costs $845,000, has an eight-year life, and has no salvage value. Assume that depreciation is straight-line to zero over the life of the project. Sales are projected a...

See Answer

Q: In the previous problem, suppose the projections given for price,

In the previous problem, suppose the projections given for price, quantity, variable costs, and fixed costs are all accurate to within ±10 percent. Calculate the best-case and worst-case NPV figures....

See Answer

Q: In each of the following cases, calculate the accounting break-

In each of the following cases, calculate the accounting break-even and the cash break-even points. Ignore any tax effects in calculating the cash break-even.

See Answer

Q: In each of the following cases, find the unknown variable:

In each of the following cases, find the unknown variable:

See Answer

Q: A project has the following estimated data: Price = $53

A project has the following estimated data: Price = $53 per unit; variable costs = $22 per unit; fixed costs = $31,460; required return = 12 percent; initial investment = $46,200; life = four years. I...

See Answer

Q: Suppose a stock had an initial price of $74 per share

Suppose a stock had an initial price of $74 per share, paid a dividend of $1.65 per share during the year, and had an ending share price of $83. Compute the percentage total return.

See Answer

Q: For Problem 9, suppose the average inflation rate over this period

For Problem 9, suppose the average inflation rate over this period was 3.1 percent and the average T-bill rate over the period was 3.9 percent. a. What was the average real return on the company’s sto...

See Answer

Q: Given the information in Problem 10, what was the average real

Given the information in Problem 10, what was the average real risk-free rate over this time period? What was the average real risk premium? Problem 10: For Problem 9, suppose the average inflation r...

See Answer

Q: Locate the Treasury issue in Figure 7.5 maturing in February

Locate the Treasury issue in Figure 7.5 maturing in February 2029. What is its coupon rate? What is its bid price? What was the previous day’s asked price? Assume a par value of $10,...

See Answer

Q: Look at Table 12.1 and Figure 12.7 in

Look at Table 12.1 and Figure 12.7 in the text. When were T-bill rates at their highest over the period from 1926 through 2019? Why do you think they were so high during this period? What relationship...

See Answer

Q: You bought one of Great White Shark Repellant Co.’s 5

You bought one of Great White Shark Repellant Co.’s 5.1 percent coupon bonds one year ago for $1,010. These bonds have a par value of $1,000, make annual payments, and mature 14 years from now. Suppos...

See Answer

Q: You find a certain stock that had returns of 18 percent,

You find a certain stock that had returns of 18 percent, −23 percent, 16 percent, and 9 percent for four of the last five years. If the average return of the stock over this period was 10.3 percent, w...

See Answer

Q: A stock has had returns of 6 percent, 29 percent,

A stock has had returns of 6 percent, 29 percent, 13 percent, −19 percent, 34 percent, and −2 percent over the last six years. What are the arithmetic and geometric average returns for the stock?

See Answer

Q: A stock has had the following year-end prices and dividends

A stock has had the following year-end prices and dividends: What are the arithmetic and geometric average returns for the stock?

See Answer

Q: In Problem 1, what was the dividend yield? The capital

In Problem 1, what was the dividend yield? The capital gains yield? Problem 1: Suppose a stock had an initial price of $74 per share, paid a dividend of $1.65 per share during the year, and had an en...

See Answer

Q: Over a 40-year period, an asset had an arithmetic

Over a 40-year period, an asset had an arithmetic return of 12.8 percent and a geometric return of 9.7 percent. Using Blume’s formula, what is your best estimate of the future annual returns over 5 ye...

See Answer

Q: Suppose the returns on long-term corporate bonds and T-

Suppose the returns on long-term corporate bonds and T-bills are normally distributed. Based on the historical record, use the NORMDIST function in Excel® to answer the following questions: a. What is...

See Answer

Q: Rework Problems 1 and 2 assuming the ending share price is $

Rework Problems 1 and 2 assuming the ending share price is $61. Problem 2: In Problem 1, what was the dividend yield? The capital gains yield? Problem 1: Suppose a stock had an initial price of $74...

See Answer

Q: Suppose you bought a bond with an annual coupon of 6 percent

Suppose you bought a bond with an annual coupon of 6 percent one year ago for $1,010. The bond sells for $1,025 today. a. Assuming a $1,000 face value, what was your total dollar return on this invest...

See Answer

Q: Locate the Treasury bond in Figure 7.5 maturing in November

Locate the Treasury bond in Figure 7.5 maturing in November 2026. Is this a premium or a discount bond? What is its current yield? What is its yield to maturity? What is the bid-ask spread in dollars?...

See Answer

Q: What was the average annual return on large-company stocks from

What was the average annual return on large-company stocks from 1926 through 2019: a. In nominal terms? b. In real terms?

See Answer

Q: Using the following returns, calculate the arithmetic average returns, the

Using the following returns, calculate the arithmetic average returns, the variances, and the standard deviations for X and Y.

See Answer

Q: You’ve observed the following returns on Pine Computer’s stock over the past

You’ve observed the following returns on Pine Computer’s stock over the past five years: 8 percent, −12 percent, 14 percent, 21 percent, and 16 percent. a. What was the arithmetic average return on th...

See Answer

Q: Ashburn Corp. issued 25-year bonds two years ago at

Ashburn Corp. issued 25-year bonds two years ago at a coupon rate of 5.6 percent. The bonds make semiannual payments. If these bonds currently sell for 97 percent of par value, what is the YTM?

See Answer

Q: Yan Yan Corp. has a $2,000 par value

Yan Yan Corp. has a $2,000 par value bond outstanding with a coupon rate of 4.7 percent paid semiannually and 13 years to maturity. The yield to maturity of the bond is 5.05 percent. What is the dolla...

See Answer

Q: The RLX Co. just paid a dividend of $3.

The RLX Co. just paid a dividend of $3.20 per share on its stock. The dividends are expected to grow at a constant rate of 4 percent per year indefinitely. If investors require a return of 10.5 percen...

See Answer

Q: The next dividend payment by Im, Inc., will be $

The next dividend payment by Im, Inc., will be $1.87 per share. The dividends are anticipated to maintain a growth rate of 4.3 percent forever. If the stock currently sells for $37 per share, what is...

See Answer

Q: Bond X is a premium bond making semiannual payments. The bond

Bond X is a premium bond making semiannual payments. The bond pays a coupon rate of 6.8 percent, has a YTM of 6.2 percent, and has 13 years to maturity. Bond Y is a discount bond making semiannual pay...

See Answer

Q: Both Bond Sam and Bond Dave have 7.1 percent coupons

Both Bond Sam and Bond Dave have 7.1 percent coupons, make semiannual payments, and are priced at par value. Bond Sam has 3 years to maturity, whereas Bond Dave has 20 years to maturity. If interest r...

See Answer

Q: Given that Hertz was down by 88 percent in the first half

Given that Hertz was down by 88 percent in the first half of 2020, why did some investors hold the stock? Why didn’t they sell out before the price declined so sharply?

See Answer

Q: Bond J has a coupon rate of 3 percent. Bond K

Bond J has a coupon rate of 3 percent. Bond K has a coupon rate of 9 percent. Both bonds have 18 years to maturity, make semiannual payments, and have a YTM of 6 percent. If interest rates suddenly ri...

See Answer

Q: Williams Software has 6.4 percent coupon bonds on the market

Williams Software has 6.4 percent coupon bonds on the market with 18 years to maturity. The bonds make semiannual payments and currently sell for 106.32 percent of par. What is the current yield on th...

See Answer

Q: Uliana Co. wants to issue new 20-year bonds for

Uliana Co. wants to issue new 20-year bonds for some muchneeded expansion projects. The company currently has 6 percent coupon bonds on the market with a par value of $1,000 that sell for $967, make s...

See Answer

Q: You purchase a bond with an invoice price of $1,

You purchase a bond with an invoice price of $1,053 and a par value of $1,000. The bond has a coupon rate of 5.3 percent, and there are four months to the next semiannual coupon date. What is the clea...

See Answer

Q: You purchase a bond with a coupon rate of 6.4

You purchase a bond with a coupon rate of 6.4 percent, a par value of $1,000, and a clean price of $1,027. If the next semiannual coupon payment is due in two months, what is the invoice price?

See Answer

Q: Milton Corp. has 8 percent coupon bonds making annual payments with

Milton Corp. has 8 percent coupon bonds making annual payments with a YTM of 7.2 percent. The current yield on these bonds is 7.55 percent. How many years do these bonds have left until they mature?...

See Answer

Q: Suppose the following bond quotes for IOU Corporation appear in the financial

Suppose the following bond quotes for IOU Corporation appear in the financial page of today’s newspaper. Assume the bond has a face value of $2,000 and the current date is April 19,...

See Answer

Q: You have found the following historical information for the Daniela Company over

You have found the following historical information for the Daniela Company over the past four years: Earnings are expected to grow at 11 percent for the next year. Using the companyâ€...

See Answer

Q: Imagination Dragons Corporation needs to raise funds to finance a plant expansion

Imagination Dragons Corporation needs to raise funds to finance a plant expansion, and it has decided to issue 25-year zero coupon bonds with a par value of $1,000 each to raise the money. The require...

See Answer

Q: Suppose your company needs to raise $65 million and you want

Suppose your company needs to raise $65 million and you want to issue 20-year bonds for this purpose. Assume the required return on your bond issue will be 4.9 percent, and you’re evaluating two issue...

See Answer

Q: Even though most corporate bonds in the United States make coupon payments

Even though most corporate bonds in the United States make coupon payments semiannually, bonds issued elsewhere often have annual coupon payments. Suppose a German company issues a bond with a par val...

See Answer

Q: You’ve just found a 10 percent coupon bond on the market that

You’ve just found a 10 percent coupon bond on the market that sells for par value. What is the maturity on this bond?

See Answer

Q: You want to have $2.5 million in real dollars

You want to have $2.5 million in real dollars in an account when you retire in 40 years. The nominal return on your investment is 10.1 percent and the inflation rate is 3.4 percent. What real amount m...

See Answer

Q: Bond P is a premium bond with a coupon rate of 9

Bond P is a premium bond with a coupon rate of 9 percent. Bond D has a coupon rate of 5 percent and is currently selling at a discount. Both bonds make annual payments, have a par value of $1,000, a Y...

See Answer

Q: The YTM on a bond is the interest rate you earn on

The YTM on a bond is the interest rate you earn on your investment if interest rates don’t change. If you actually sell the bond before it matures, your realized return is known as the holding period...

See Answer

Q: Cookie Dough Corporation has two different bonds currently outstanding. Bond M

Cookie Dough Corporation has two different bonds currently outstanding. Bond M has a face value of $20,000 and matures in 20 years. The bond makes no payments for the first six years, then pays $900 e...

See Answer

Q: At one point, certain U.S. Treasury bonds were

At one point, certain U.S. Treasury bonds were callable. Consider the prices in the following three Treasury issues as of May 15, 2021: The bond in the middle is callable in February 2022. What is t...

See Answer

Q: The following Treasury bond quote appeared in The Wall Street Journal on

The following Treasury bond quote appeared in The Wall Street Journal on May 11, 2004: Why would anyone buy this Treasury bond with a negative yield to maturity? How is this possible?

See Answer

Q: When Marilyn Monroe died, ex-husband Joe DiMaggio vowed to

When Marilyn Monroe died, ex-husband Joe DiMaggio vowed to place fresh flowers on her grave every Sunday as long as he lived. The week after she died in 1962, a bunch of fresh flowers that the former...

See Answer

Q: You are planning to save for retirement over the next 30 years

You are planning to save for retirement over the next 30 years. To save for retirement, you will invest $700 per month in a stock account in real dollars and $300 per month in a bond account in real d...

See Answer

Q: After successfully completing your corporate finance class, you feel the next

After successfully completing your corporate finance class, you feel the next challenge ahead is to serve on the board of directors of Cornwall Enterprises. Unfortunately, you will be the only person...

See Answer

Q: A Japanese company has a bond outstanding that sells for 96.

A Japanese company has a bond outstanding that sells for 96.318 percent of its ¥100,000 par value. The bond has a coupon rate of 3.4 percent paid annually and matures in 16 years. What is the yield to...

See Answer

Q: The Dahlia Flower Co. has earnings of $3.64

The Dahlia Flower Co. has earnings of $3.64 per share. The benchmark PE for the company is 18. What stock price would you consider appropriate? What if the benchmark PE were 21?

See Answer

Q: Z Space, Inc., is a new company and currently has

Z Space, Inc., is a new company and currently has negative earnings. The company’s sales are $2.7 million and there are 175,000 shares outstanding. If the benchmark price-sales ratio is 4.3, what is y...

See Answer

Q: Quinoa Farms just paid a dividend of $2.95 on

Quinoa Farms just paid a dividend of $2.95 on its stock. The growth rate in dividends is expected to be a constant 3.4 percent per year indefinitely. Investors require a return of 15 percent for the f...

See Answer

Q: Metallica Bearings, Inc., is a young start-up company

Metallica Bearings, Inc., is a young start-up company. No dividends will be paid on the stock over the next 9 years because the firm needs to plow back its earnings to fuel growth. The company will pa...

See Answer

Q: Premier, Inc., has an odd dividend policy. The company

Premier, Inc., has an odd dividend policy. The company has just paid a dividend of $3.75 per share and has announced that it will increase the dividend by $5 per share for each of the next five years...

See Answer

Q: McCabe Corporation is expected to pay the following dividends over the next

McCabe Corporation is expected to pay the following dividends over the next four years: $15, $11, $9, and $2.95. Afterward, the company pledges to maintain a constant 4 percent growth rate in dividend...

See Answer

Q: Synovec Co. is growing quickly. Dividends are expected to grow

Synovec Co. is growing quickly. Dividends are expected to grow at a rate of 30 percent for the next three years, with the growth rate falling off to a constant 4 percent thereafter. If the required re...

See Answer

Q: Orkazana Corp. is experiencing rapid growth. Dividends are expected to

Orkazana Corp. is experiencing rapid growth. Dividends are expected to grow at 25 percent per year during the next three years, 15 percent over the following year, and then 6 percent per year indefini...

See Answer

Q: Antiques R Us is a mature manufacturing firm. The company just

Antiques R Us is a mature manufacturing firm. The company just paid a dividend of $12.40, but management expects to reduce the payout by 4 percent per year indefinitely. If you require a return of 9.5...

See Answer

Q: Matterhorn Corporation stock currently sells for $49 per share. The

Matterhorn Corporation stock currently sells for $49 per share. The market requires a return of 11 percent on the firm’s stock. If the company maintains a constant 3.5 percent growth rate in dividends...

See Answer

Q: Nikita Enterprises has bonds on the market making annual payments, with

Nikita Enterprises has bonds on the market making annual payments, with eight years to maturity, a par value of $1,000, and selling for $962. At this price, the bonds yield 5.1 percent. What must the...

See Answer

Q: E-Eyes.com just issued some new 20/20

E-Eyes.com just issued some new 20/20 preferred stock. The issue will pay an annual dividend of $20 in perpetuity, beginning 20 years from now. If the market requires a return of 5.4 percent on this i...

See Answer

Q: You have found the following stock quote for RJW Enterprises, Inc

You have found the following stock quote for RJW Enterprises, Inc., in the financial pages of today’s newspaper. What was the closing price for this stock that appeared in yesterday&...

See Answer

Q: Impossible Corp. just paid a dividend of $1.93

Impossible Corp. just paid a dividend of $1.93 per share. The dividends are expected to grow at 24 percent for the next eight years and then level off to a growth rate of 3.5 percent indefinitely. If...

See Answer

Q: Navel County Choppers, Inc., is experiencing rapid growth. The

Navel County Choppers, Inc., is experiencing rapid growth. The company expects dividends to grow at 18 percent per year for the next 11 years before leveling off at 4 percent into perpetuity. The requ...

See Answer

Q: Meadow Dew Corp. currently has an EPS of $4.

Meadow Dew Corp. currently has an EPS of $4.05, and the benchmark PE for the company is 21. Earnings are expected to grow at 4.9 percent per year. a. What is your estimate of the current stock price?...

See Answer

Q: In the previous problem, we assumed that the stock had a

In the previous problem, we assumed that the stock had a single stock price for the year. However, if you look at stock prices over any year, you will find a high and low stock price for the year. Ins...

See Answer

Q: Regal, Inc., currently has an EPS of $3.

Regal, Inc., currently has an EPS of $3.25 and an earnings growth rate of 8 percent. If the benchmark PE ratio is 23, what is the target share price five years from now?

See Answer

Q: In practice, a common way to value a share of stock

In practice, a common way to value a share of stock when a company pays dividends is to value the dividends over the next five years or so, then find the terminal stock price using a benchmark PE rati...

See Answer

Q: Penguin, Inc., has balance sheet equity of $7.

Penguin, Inc., has balance sheet equity of $7.9 million. At the same time, the income statement shows net income of $832,000. The company paid dividends of $285,000 and has 245,000 shares of stock out...

See Answer

Q: Consider four different stocks, all of which have a required return

Consider four different stocks, all of which have a required return of 12 percent and a most recent dividend of $3.45 per share. Stocks W, X, and Y are expected to maintain constant growth rates in di...

See Answer

Q: Westco Co. issued 15-year bonds a year ago at

Westco Co. issued 15-year bonds a year ago at a coupon rate of 5.4 percent. The bonds make semiannual payments and have a par value of $1,000. If the YTM on these bonds is 4.5 percent, what is the cur...

See Answer

Q: Most corporations pay quarterly dividends on their common stock rather than annual

Most corporations pay quarterly dividends on their common stock rather than annual dividends. Barring any unusual circumstances during the year, the board raises, lowers, or maintains the current divi...

See Answer

Q: Storico Co. just paid a dividend of $3.65

Storico Co. just paid a dividend of $3.65 per share. The company will increase its dividend by 20 percent next year and then reduce its dividend growth rate by 5 percentage points per year until it re...

See Answer

Q: This one’s a little harder. Suppose the current share price for

This one’s a little harder. Suppose the current share price for the firm in the previous problem is $67.25 and all the dividend information remains the same. What required return must investors be dem...

See Answer

Q: Five Star Corporation will pay a dividend of $3.04

Five Star Corporation will pay a dividend of $3.04 per share next year. The company pledges to increase its dividend by 3.75 percent per year indefinitely. If you require a return of 11 percent on you...

See Answer

Q: Caccamise Co. is expected to maintain a constant 3.4

Caccamise Co. is expected to maintain a constant 3.4 percent growth rate in its dividends indefinitely. If the company has a dividend yield of 5.3 percent, what is the required return on the company’s...

See Answer

Q: Suppose you know that a company’s stock currently sells for $78

Suppose you know that a company’s stock currently sells for $78 per share and the required return on the stock is 10.9 percent. You also know that the total return on the stock is evenly divided betwe...

See Answer

Q: Hailey Corp. pays a constant $9.45 dividend on

Hailey Corp. pays a constant $9.45 dividend on its stock. The company will maintain this dividend for the next 13 years and will then cease paying dividends forever. If the required return on this sto...

See Answer

Q: Fegley, Inc., has an issue of preferred stock outstanding that

Fegley, Inc., has an issue of preferred stock outstanding that pays a $3.80 dividend every year in perpetuity. If this issue currently sells for $93 per share, what is the required return?

See Answer

Q: Red, Inc., Yellow Corp., and Blue Company each will

Red, Inc., Yellow Corp., and Blue Company each will pay a dividend of $4.15 next year. The growth rate in dividends for all three companies is 4 percent. The required return for each company’s stock i...

See Answer

Q: What is the payback period for the following set of cash flows

What is the payback period for the following set of cash flows? Year …………………….. Cash Flow 0 …………………………… −$7,700 1 ……………………………….. 1,900 2 ……………………………….. 3,000 3 ……………………………….. 2,300 4 ……………………………….. 1...

See Answer

Q: A major college textbook publisher has an existing finance textbook. The

A major college textbook publisher has an existing finance textbook. The publisher is debating whether to produce an “essentialized” version, meaning a shorter (and lower-priced) book. What are some o...

See Answer

Q: What is the IRR of the following set of cash flows?

What is the IRR of the following set of cash flows? Year ………………………….. Cash Flow 0 ……………………………….. −$18,700 1 ……………………………………… 9,400 2 …………………………………… 10,400 3 ……………………………………… 6,500

See Answer

Q: Bruin, Inc., has identified the following two mutually exclusive projects

Bruin, Inc., has identified the following two mutually exclusive projects: a. What is the IRR for each of these projects? Using the IRR decision rule, which project should the company accept? Is thi...

See Answer

Q: Consider the following two mutually exclusive projects: /

Consider the following two mutually exclusive projects: Sketch the NPV profiles for X and Y over a range of discount rates from zero to 25 percent. What is the crossover rate for these two projects?...

See Answer

Q: Howell Petroleum, Inc., is trying to evaluate a generation project

Howell Petroleum, Inc., is trying to evaluate a generation project with the following cash flows: Year ……………………… Cash Flow 0 ………………….. −$52,000,000 1 ………………………. 74,000,000 2 …………………… − 12,000,000 a....

See Answer

Q: What is the profitability index for the following set of cash flows

What is the profitability index for the following set of cash flows if the relevant discount rate is 10 percent? What if the discount rate is 15 percent? If it is 22 percent? Year ………………………………….. Cas...

See Answer

Q: The Michner Corporation is trying to choose between the following two mutually

The Michner Corporation is trying to choose between the following two mutually exclusive design projects: a. If the required return is 10 percent and the company applies the profitability index deci...

See Answer

Q: Consider the following two mutually exclusive projects: /

Consider the following two mutually exclusive projects: Whichever project you choose, if any, you require a return of 11 percent on your investment. a. If you apply the payback criterion, which inve...

See Answer

Q: An investment has an installed cost of $574,380.

An investment has an installed cost of $574,380. The cash flows over the four-year life of the investment are projected to be $216,700, $259,300, $214,600, and $167,410, respectively. If the discount...

See Answer

Q: Duo Corp. is evaluating a project with the following cash flows

Duo Corp. is evaluating a project with the following cash flows: Year …………………………………………… Cash Flow 0 ………………………………………………. −$53,000 1 ……………………………………………………. 16,700 2 ……………………………………………………. 21,900 3 ………………...

See Answer

Q: An investment project provides cash inflows of $835 per year for

An investment project provides cash inflows of $835 per year for eight years. What is the project payback period if the initial cost is $1,900? What if the initial cost is $3,600? What if it is $7,400...

See Answer

Q: Draiman Corporation has bonds on the market with 14.5 years

Draiman Corporation has bonds on the market with 14.5 years to maturity, a YTM of 5.3 percent, a par value of $1,000, and a current price of $987. The bonds make semiannual payments. What must the cou...

See Answer

Q: Suppose the company in the previous problem uses a discount rate of

Suppose the company in the previous problem uses a discount rate of 11 percent and a reinvestment rate of 8 percent on all of its projects. Calculate the MIRR of the project using all three methods us...

See Answer

Q: An investment under consideration has a payback of seven years and a

An investment under consideration has a payback of seven years and a cost of $745,000. If the required return is 11 percent, what is the worst-case NPV? The best-case NPV? Explain. Assume the cash flo...

See Answer

Q: The Yurdone Corporation wants to set up a private cemetery business.

The Yurdone Corporation wants to set up a private cemetery business. According to the CFO, Barry M. Deep, business is “looking up.” As a result, the cemetery project will provide a net cash inflow of...

See Answer

Q: A project has the following cash flows: Year ………………………

A project has the following cash flows: Year ……………………… Cash Flow 0 …………………………… $74,000 1 …………………………… − 49,000 2 …………………………… − 41,000 What is the IRR for this project? If the required return is 12 pe...

See Answer

Q: Mako Corp. has a project with the following cash flows:

Mako Corp. has a project with the following cash flows: Year ……………………………… Cash Flow 0 …………………………………… $25,000 1 …………………………….……… − 11,000 2 ………………………………………… 7,000 What is the IRR of the project? What...

See Answer

Q: Anderson International Limited is evaluating a project in Erewhon. The project

Anderson International Limited is evaluating a project in Erewhon. The project will create the following cash flows: Year ………………………………… Cash Flow 0 ………………………………… −$1,785,000 1 ………………………………………. 610,00...

See Answer

Q: Kara, Inc., imposes a payback cutoff of three years for

Kara, Inc., imposes a payback cutoff of three years for its international investment projects. If the company has the following two projects available, should it accept either of them?

See Answer

Q: An investment project has annual cash inflows of $2,800

An investment project has annual cash inflows of $2,800, $3,700, $5,100, and $4,300, for the next four years, respectively. The discount rate is 9 percent. What is the discounted payback period for th...

See Answer

Q: Calculating Discounted Payback [LO3] An investment project costs $19

Calculating Discounted Payback [LO3] An investment project costs $19,000 and has annual cash flows of $5,100 for six years. What is the discounted payback period if the discount rate is zero percent?...

See Answer

Q: You’re trying to determine whether to expand your business by building a

You’re trying to determine whether to expand your business by building a new manufacturing plant. The plant has an installation cost of $12.6 million, which will be depreciated straight-line to zero o...

See Answer

Q: You find a zero coupon bond with a par value of $

You find a zero coupon bond with a par value of $10,000 and 24 years to maturity. If the yield to maturity on this bond is 4.2 percent, what is the dollar price of the bond? Assume semiannual compound...

See Answer

Q: A firm evaluates all of its projects by applying the IRR rule

A firm evaluates all of its projects by applying the IRR rule. If the required return is 14 percent, should the firm accept the following project? Year ……………………… Cash Flow 0 ………………………….. −$41,000 1 …...

See Answer

Q: A project that provides annual cash flows of $15,300

A project that provides annual cash flows of $15,300 for nine years costs $74,000 today. Is this a good project if the required return is 8 percent? What if it’s 20 percent? At what discount rate woul...

See Answer

Q: Parker & Stone, Inc., is looking at setting up a

Parker & Stone, Inc., is looking at setting up a new manufacturing plant in South Park to produce garden tools. The company bought some land six years ago for $2.8 million in anticipation of using it...

See Answer

Q: In the previous problem, suppose the required return on the project

In the previous problem, suppose the required return on the project is 12 percent. What is the project’s NPV? Problem 9: Esfandairi Enterprises is considering a new three-year expansion project that...

See Answer

Q: In the previous problem, suppose the project requires an initial investment

In the previous problem, suppose the project requires an initial investment in net working capital of $250,000, and the fixed asset will have a market value of $180,000 at the end of the project. What...

See Answer

Q: In the previous problem, suppose the fixed asset actually falls into

In the previous problem, suppose the fixed asset actually falls into the three-year MACRS class. All the other facts are the same. What is the project’s Year 1 net cash flow now? Year 2? Year 3? What...

See Answer

Q: In the previous problem, suppose the fixed asset actually qualifies for

In the previous problem, suppose the fixed asset actually qualifies for 100 percent bonus depreciation in the first year. All the other facts are the same. What is the project’s Year 1 net cash flow n...

See Answer

Q: Dog Up! Franks is looking at a new sausage system with

Dog Up! Franks is looking at a new sausage system with an installed cost of $385,000. This cost will be depreciated straight-line to zero over the project’s five-year life, at the end of which the sau...

See Answer

Q: In the previous problem, suppose the fixed asset actually qualifies for

In the previous problem, suppose the fixed asset actually qualifies for 100 percent bonus depreciation in the first year. What is the new NPV? Problem 14: Dog Up! Franks is looking at a new sausage s...

See Answer

Q: Your firm is contemplating the purchase of a new $535,

Your firm is contemplating the purchase of a new $535,000 computer-based order entry system. The system will be depreciated straight-line to zero over its five-year life. It will be worth $30,000 at t...

See Answer

Q: Asset W has an expected return of 8.8 percent and

Asset W has an expected return of 8.8 percent and a beta of .90. If the risk-free rate is 2.6 percent, complete the following table for portfolios of Asset W and a risk-free asset. Illustrate the rela...

See Answer

Q: You have $100,000 to invest in a portfolio containing

You have $100,000 to invest in a portfolio containing Stock X and Stock Y. Your goal is to create a portfolio that has an expected return of 12.1 percent. If Stock X has an expected return of 10.28 pe...

See Answer

Q: Your firm has an average collection period of 38 days. Current

Your firm has an average collection period of 38 days. Current practice is to factor all receivables immediately at a discount of 1.75 percent. What is the effective cost of borrowing in this case? As...

See Answer

Q: Sexton Corp. has projected the following sales for the coming year

Sexton Corp. has projected the following sales for the coming year: Sales in the year following this one are projected to be 15 percent greater in each quarter. a. Calculate payments to suppliers as...

See Answer

Q: The Bandon Pine Corporation’s purchases from suppliers in a quarter are equal

The Bandon Pine Corporation’s purchases from suppliers in a quarter are equal to 75 percent of the next quarter’s forecast sales. The payables period is 60 days. Wa...

See Answer

Q: In a typical month, the Montrose Corporation receives 90 checks totaling

In a typical month, the Montrose Corporation receives 90 checks totaling $85,000. These are delayed four days on average. What is the average daily float? Assume 30 days in a month.

See Answer

Q: No More Books Corporation has an agreement with Floyd Bank whereby the

No More Books Corporation has an agreement with Floyd Bank whereby the bank handles $3.4 million in collections per day and requires a $320,000 compensating balance. No More Books is contemplating can...

See Answer

Q: Bird’s Eye Treehouses, Inc., a Kentucky company, has determined

Bird’s Eye Treehouses, Inc., a Kentucky company, has determined that a majority of its customers are located in the Pennsylvania area. It therefore is considering using a lockbox system offered by a b...

See Answer

Q: Each business day, on average, a company writes checks totaling

Each business day, on average, a company writes checks totaling $19,500 to pay its suppliers. The usual clearing time for the checks is four days. Meanwhile, the company is receiving payments from its...

See Answer

Q: Purple Feet Wine, Inc., receives an average of $18

Purple Feet Wine, Inc., receives an average of $18,600 in checks per day. The delay in clearing is typically three days. The current interest rate is .017 percent per day. a. What is the company’s flo...

See Answer

Q: Your neighbor goes to the post office once a month and picks

Your neighbor goes to the post office once a month and picks up two checks, one for $8,450 and one for $4,200. The larger check takes four days to clear after it is deposited; the smaller one takes th...

See Answer

Q: Your firm has an average receipt size of $135. A

Your firm has an average receipt size of $135. A bank has approached you concerning a lockbox service that will decrease your total collection time by two days. You typically receive 4,900 checks per...

See Answer

Q: Consider the following information about Stocks I and II:

Consider the following information about Stocks I and II: The market risk premium is 7 percent, and the risk-free rate is 3.5 percent. Which stock has the most systematic risk? Which one has the mos...

See Answer

Q: A mail-order firm processes 5,800 checks per month

A mail-order firm processes 5,800 checks per month. Of these, 60 percent are for $49 and 40 percent are for $84. The $49 checks are delayed two days on average; the $84 checks are delayed three days o...

See Answer

Q: Paper Submarine Manufacturing is investigating a lockbox system to reduce its collection

Paper Submarine Manufacturing is investigating a lockbox system to reduce its collection time. It has determined the following: Average number of payments per day …………………………………………. 435 Average value...

See Answer

Q: It takes Cookie Cutter Modular Homes, Inc., about six days

It takes Cookie Cutter Modular Homes, Inc., about six days to receive and deposit checks from customers. The company’s management is considering a lockbox system to reduce the firm’s collection times....

See Answer

Q: Every two weeks, No More Pencils, Inc., disburses checks

Every two weeks, No More Pencils, Inc., disburses checks that average $124,000 and take seven days to clear. How much interest can the company earn annually if it delays transfer of funds from an inte...

See Answer

Q: You place an order for 250 units of inventory at a unit

You place an order for 250 units of inventory at a unit price of $190. The supplier offers terms of 1/10, net 30. a. How long do you have to pay before the account is overdue? If you take the full pe...

See Answer

Q: Red Hawk, Inc., is considering a change in its cash

Red Hawk, Inc., is considering a change in its cash-only sales policy. The new terms of sale would be net one month. Based on the following information, determine if the company should proceed or not....

See Answer

Q: Queen Manufacturing uses 3,900 switch assemblies per week and then

Queen Manufacturing uses 3,900 switch assemblies per week and then reorders another 3,900. If the relevant carrying cost per switch assembly is $6.25, and the fixed order cost is $1,400, is the compan...

See Answer

Q: The Trektronics store begins each week with 380 phasers in stock.

The Trektronics store begins each week with 380 phasers in stock. This stock is depleted each week and reordered. If the carrying cost per phaser is $31 per year and the fixed order cost is $325, what...

See Answer

Q: The Branson Corporation is considering a change in its cash-only

The Branson Corporation is considering a change in its cash-only policy. The new terms would be net one period. Based on the following information, determine if the company should proceed or not. The...

See Answer

Q: Fitzgerald, Inc., currently has an all-cash credit policy

Fitzgerald, Inc., currently has an all-cash credit policy. It is considering making a change in the credit policy by going to terms of net 30 days. Based on the following information, what do you reco...

See Answer

Q: Suppose you observe the following situation: /

Suppose you observe the following situation: Assume these securities are correctly priced. Based on the CAPM, what is the expected return on the market? What is the risk-free rate?

See Answer

Q: In Problem 14, what is the break-even quantity for

In Problem 14, what is the break-even quantity for the new credit policy? Problem 14: The Branson Corporation is considering a change in its cash-only policy. The new terms would be net one period. B...

See Answer

Q: In Problem 15, what is the break-even price per

In Problem 15, what is the break-even price per unit under the new credit policy? Assume all other values remain the same. Problem 15: Fitzgerald, Inc., currently has an all-cash credit policy. It is...

See Answer

Q: The Caccamisse Corporation has annual sales of $29 million. The

The Caccamisse Corporation has annual sales of $29 million. The average collection period is 34 days. What is the average investment in accounts receivable as shown on the balance sheet? Assume 365 da...

See Answer

Q: Solar Engines manufactures solar engines for tractor-trailers. Given the

Solar Engines manufactures solar engines for tractor-trailers. Given the fuel savings available, new orders for 125 units have been made by customers requesting credit. The variable cost is $14,800 pe...

See Answer

Q: In Problem 21, assume that the probability of default is 15

In Problem 21, assume that the probability of default is 15 percent. Should the orders be filled now? Assume the number of repeat customers is affected by the defaults. In other words, 30 percent of t...

See Answer

Q: Kyoto Joe, Inc., sells earnings forecasts for Japanese securities.

Kyoto Joe, Inc., sells earnings forecasts for Japanese securities. Its credit terms are 2/10, net 30. Based on experience, 65 percent of all customers will take the discount. a. What is the average c...

See Answer

Q: Sweetens Cove, Inc., has weekly credit sales of $23

Sweetens Cove, Inc., has weekly credit sales of $23,400, and the average collection period is 31 days. What is the average accounts receivable figure?

See Answer

Q: Benton, Inc., has an average collection period of 24 days

Benton, Inc., has an average collection period of 24 days. Its average daily investment in receivables is $49,300. What are annual credit sales? What is the receivables turnover? Assume 365 days per y...

See Answer

Q: Essence of Skunk Fragrances, Ltd., sells 9,300 units

Essence of Skunk Fragrances, Ltd., sells 9,300 units of its perfume collection each year at a price per unit of $415. All sales are on credit with terms of 1/10, net 40. The discount is taken by 45 pe...

See Answer

Q: The Paa Ko Corporation sells on credit terms of net 30.

The Paa Ko Corporation sells on credit terms of net 30. Its accounts are, on average, four days past due. If annual credit sales are $7.2 million, what is the company’s balance sheet amount in account...

See Answer

Q: Suppose you observe the following situation: /

Suppose you observe the following situation: a. Calculate the expected return on each stock. b. Assuming the capital asset pricing model holds and Stock A’s beta is greater than St...

See Answer

Q: Air Spares is a wholesaler that stocks engine components and test equipment

Air Spares is a wholesaler that stocks engine components and test equipment for the commercial aircraft industry. A new customer has placed an order for eight high-bypass turbine engines, which increa...

See Answer

Q: Suppose the spot and six-month forward rates on the Norwegian

Suppose the spot and six-month forward rates on the Norwegian krone are Kr 9.14 and Kr 9.27, respectively. The annual riskfree rate in the United States is 3.8 percent, and the annual risk-free rate i...

See Answer

Q: You observe that the inflation rate in the United States is 2

You observe that the inflation rate in the United States is 2.6 percent per year and that T-bills currently yield 3.2 percent annually. Using the approximate international Fisher effect, what do you e...

See Answer

Q: Suppose the spot and three-month forward rates for the yen

Suppose the spot and three-month forward rates for the yen are ¥114.37 and ¥113.89, respectively. a. Is the yen expected to get stronger or weaker? b. What would you estimate is the difference between...

See Answer

Q: Suppose the spot exchange rate for the Hungarian forint is HUF 308

Suppose the spot exchange rate for the Hungarian forint is HUF 308.27. The inflation rate in the United States will be 2.6 percent per year. It will be 4.5 percent in Hungary. What do you predict the...

See Answer

Q: Lakonishok Equipment has an investment opportunity in Europe. The project costs

Lakonishok Equipment has an investment opportunity in Europe. The project costs €9.5 million and is expected to produce cash flows of €1.6 million in Year 1, €2.1 million in Year 2, and €3.2 million i...

See Answer

Q: You are evaluating a proposed expansion of an existing subsidiary located in

You are evaluating a proposed expansion of an existing subsidiary located in Switzerland. The cost of the expansion would be SF 13.8 million. The cash flows from the project would be SF 4.1 million pe...

See Answer

Q: Atreides International has operations in Arrakis. The balance sheet for this

Atreides International has operations in Arrakis. The balance sheet for this division in Arrakeen solaris shows assets of 39,000 solaris, debt in the amount of 11,000 solaris, and equity of 28,000 sol...

See Answer

Q: In Problem 16, assume the equity increases by 1,250

In Problem 16, assume the equity increases by 1,250 solaris due to retained earnings. If the exchange rate at the end of the year is 1.54 solaris per dollar, what does the balance sheet look like? Pr...

See Answer

Q: Suppose the spot exchange rate for the Canadian dollar is Can$

Suppose the spot exchange rate for the Canadian dollar is Can$1.34 and the six-month forward rate is Can$1.41. a. Which is worth more, a U.S. dollar or a Canadian dollar? b. Assuming absolute PPP hold...

See Answer

Q: Chauhan Corp. has a debt-equity ratio of .65

Chauhan Corp. has a debt-equity ratio of .65. The company is considering a new plant that will cost $55 million to build. When the company issues new equity, it incurs a flotation cost of 6 percent. T...

See Answer

Q: Suppose the Japanese yen exchange rate is ¥116 = $1

Suppose the Japanese yen exchange rate is ¥116 = $1, and the British pound exchange rate is £1 = $1.27. a. What is the cross-rate in terms of yen per pound? b. Suppose the cross-rate is ¥156 = £1. Is...

See Answer

Q: The treasurer of a major U.S. firm has $

The treasurer of a major U.S. firm has $30 million to invest for three months. The interest rate in the United States is .15 percent per month. The interest rate in Great Britain is .26 percent per mo...

See Answer

Q: Suppose the current exchange rate for the Polish zloty is Z 3

Suppose the current exchange rate for the Polish zloty is Z 3.91. The expected exchange rate in three years is Z 3.98. What is the difference in the annual inflation rates for the United States and Po...

See Answer

Q: Suppose your company imports computer motherboards from Singapore. The exchange rate

Suppose your company imports computer motherboards from Singapore. The exchange rate is given in Figure 21.1. You have just placed an order for 30,000 motherboards at a cost to you of 218.50 Singapore...

See Answer

Q: Refer to Table 23.1 in the text to answer this

Refer to Table 23.1 in the text to answer this question. Suppose you purchase a September 2020 cocoa futures contract this day at the last price of the day. What will your profit or loss be if cocoa p...

See Answer

Q: Refer to Table 23.1 in the text to answer this

Refer to Table 23.1 in the text to answer this question. Suppose you sell five July 2020 silver futures contracts this day at the last price of the day. What will your profit or loss be if silver pric...

See Answer

Q: Refer to Table 23.2 in the text to answer this

Refer to Table 23.2 in the text to answer this question. Suppose you purchase the July 2020 call option on corn futures with a strike price of $3.25. Assume you purchased the option at the last price....

See Answer

Q: Refer to Table 23.2 in the text to answer this

Refer to Table 23.2 in the text to answer this question. Suppose you purchase the July 2020 put option on corn futures with a strike price of $3.25. Assume your purchase was at the last price. What is...

See Answer

Q: Suppose your company has a building worth $125 million. Because

Suppose your company has a building worth $125 million. Because it is located in a high-risk area for natural disasters, the probability of a total loss in any particular year is 1.05 percent. What is...

See Answer

Q: Refer to Table 23.1 in the text to answer this

Refer to Table 23.1 in the text to answer this question. Suppose your firm produces breakfast cereal and needs 165,000 bushels of corn in December 2020 for an upcoming promotion. You would like to loc...

See Answer

Q: This is a comprehensive project evaluation problem bringing together much of what

This is a comprehensive project evaluation problem bringing together much of what you have learned in this and previous chapters. Suppose you have been hired as a financial consultant to Defense Elect...

See Answer

Q: T-bills currently yield 3.4 percent. Stock in

T-bills currently yield 3.4 percent. Stock in Deadwood Manufacturing is currently selling for $67 per share. There is no possibility that the stock will be worth less than $60 per share in one year. a...

See Answer

Q: You have been hired to value a new 30-year callable

You have been hired to value a new 30-year callable, convertible bond. The bond has a coupon rate of 2.3 percent, payable semiannually, and its face value is $1,000. The conversion price is $49, and t...

See Answer

Q: A bond with 20 detachable warrants has just been offered for sale

A bond with 20 detachable warrants has just been offered for sale at $1,000. The bond matures in 20 years and has an annual coupon of $18. Each warrant gives the owner the right to purchase two shares...

See Answer

Q: We are examining a new project. We expect to sell 7

We are examining a new project. We expect to sell 7,400 units per year at $59 net cash flow apiece for the next 10 years. In other words, the annual cash flow is projected to be $59 × 7,400 = $436,600...

See Answer

Q: In Problem 14, suppose you think it is likely that expected

In Problem 14, suppose you think it is likely that expected sales will be revised upward to 11,400 units if the first year is a success and revised downward to 3,500 units if the first year is not a s...

See Answer

Q: In Problem 15, suppose the scale of the project can be

In Problem 15, suppose the scale of the project can be doubled in one year in the sense that twice as many units can be produced and sold. Naturally, expansion would be desirable only if the project i...

See Answer

Q: Suppose a share of stock sells for $54. The risk

Suppose a share of stock sells for $54. The risk-free rate is 5 percent, and the stock price in one year will be either $60 or $70. a. What is the value of a call option with an exercise price of $60...

See Answer

Q: Which of the following two sets of relationships, at time of

Which of the following two sets of relationships, at time of issuance for convertible bonds, is more typical? Why?

See Answer

Q: Campbell, Inc., has a $1,000 face value

Campbell, Inc., has a $1,000 face value convertible bond issue that is currently selling in the market for $960. Each bond is exchangeable at any time for 17 shares of the company’s stock. The convert...

See Answer

Q: The Tribiani Co. just issued a dividend of $2.

The Tribiani Co. just issued a dividend of $2.90 per share on its common stock. The company is expected to maintain a constant 4.5 percent growth rate in its dividends indefinitely. If the stock sells...

See Answer

Q: Liberty Products, Inc., is considering a new product launch.

Liberty Products, Inc., is considering a new product launch. The firm expects to have annual operating cash flow of $4.9 million for the next eight years. The company uses a discount rate of 11 percen...

See Answer

Q: You have been hired to value a new 25-year callable

You have been hired to value a new 25-year callable, convertible bond. The bond has a coupon rate of 2.1 percent, payable annually. The conversion price is $54, and the stock currently sells for $26.4...

See Answer

Q: Consider the following project of Hand Clapper, Inc. The company

Consider the following project of Hand Clapper, Inc. The company is considering a four-year project to manufacture clap-command garage door openers. This project requires an initial investment of $12...

See Answer

Q: Use the option quote information shown here to answer the questions that

Use the option quote information shown here to answer the questions that follow. The stock is currently selling for $40. a. Suppose you buy 10 contracts of the February 38 call option. How much will...

See Answer

Q: The price of Chive Corp. stock will be either $57

The price of Chive Corp. stock will be either $57 or $84 at the end of the year. Call options are available with one year to expiration. T-bills currently yield 4 percent. a. Suppose the current price...

See Answer

Q: The price of Cilantro, Inc., stock will be either $

The price of Cilantro, Inc., stock will be either $60 or $80 at the end of the year. Call options are available with one year to expiration. T-bills currently yield 6 percent. a. Suppose the current p...

See Answer

Q: A one-year call option contract on Cheesy Poofs Co.

A one-year call option contract on Cheesy Poofs Co. stock sells for $725. In one year, the stock will be worth $64 or $81 per share. The exercise price on the call option is $70. What is the current v...

See Answer

Q: Rackin Pinion Corporation’s assets are currently worth $1,030.

Rackin Pinion Corporation’s assets are currently worth $1,030. In one year, they will be worth either $1,000 or $1,270. The risk-free interest rate is 3.7 percent. Suppose the company has an outstandi...

See Answer

Q: Buckeye Industries has a bond issue with a face value of $

Buckeye Industries has a bond issue with a face value of $1,000 that is coming due in one year. The value of the company’s assets is currently $1,060. The CEO believes that the assets in the company w...

See Answer

Q: A $1,000 par convertible debenture has a conversion price

A $1,000 par convertible debenture has a conversion price for common stock of $18 per share. With the common stock selling at $25, what is the conversion value of the bond?

See Answer

Q: Brannan Manufacturing has a target debt-equity ratio of .35

Brannan Manufacturing has a target debt-equity ratio of .35. Its cost of equity is 11 percent, and its pretax cost of debt is 6 percent. If the tax rate is 21 percent, what is the company’s WACC?

See Answer

Q: If you have $1,490 today, how much will

If you have $1,490 today, how much will it be worth in six years at 9 percent per year compounded continuously?

See Answer

Q: In Problem 9, suppose you wanted the option to sell the

In Problem 9, suppose you wanted the option to sell the land to the buyer in one year. Assuming all the facts are the same, describe the transaction that would occur today. What is the price of the tr...

See Answer

Q: You are given the following information concerning options on a particular stock

You are given the following information concerning options on a particular stock: Stock price = $64 Exercise price = $60 Risk-free rate = 2% per year, compounded continuously Maturity = 6 months Stand...

See Answer

Q: A call option with an exercise price of $25 and four

A call option with an exercise price of $25 and four months to expiration has a price of $3.10. The stock is currently priced at $25.19, and the risk-free rate is 2.5 percent per year, compounded cont...

See Answer

Q: A call option has an exercise price of $70 and matures

A call option has an exercise price of $70 and matures in six months. The current stock price is $73, and the risk-free rate is 5 percent per year, compounded continuously. What is the price of the ca...

See Answer

Q: Sunburn Sunscreen has a zero coupon bond issue outstanding with a $

Sunburn Sunscreen has a zero coupon bond issue outstanding with a $10,000 face value that matures in one year. The current market value of the firm’s assets is $10,900. The standard deviation of the r...

See Answer

Q: Suppose the firm in Problem 16 is considering two mutually exclusive investments

Suppose the firm in Problem 16 is considering two mutually exclusive investments. Project A has an NPV of $2,400, and Project B has an NPV of $2,800. As the result of taking Project A, the standard de...

See Answer

Q: Frostbite Thermalwear has a zero coupon bond issue outstanding with a face

Frostbite Thermalwear has a zero coupon bond issue outstanding with a face value of $20,000 that matures in one year. The current market value of the firm’s assets is $23,100. The standard deviation o...

See Answer

Q: If you need $25,000 in 12 years, how

If you need $25,000 in 12 years, how much will you need to deposit today if you can earn 7 percent per year compounded continuously?

See Answer

Q: A company has a single zero coupon bond outstanding that matures in

A company has a single zero coupon bond outstanding that matures in five years with a face value of $16.5 million. The current value of the company’s assets is $15.1 million, and the standard deviatio...

See Answer

Q: Fama’s Llamas has a weighted average cost of capital of 8.

Fama’s Llamas has a weighted average cost of capital of 8.4 percent. The company’s cost of equity is 11 percent, and its pretax cost of debt is 5.8 percent. The tax rate is 25 percent. What is the com...

See Answer

Q: Zoso Industries has a zero coupon bond issue that matures in two

Zoso Industries has a zero coupon bond issue that matures in two years with a face value of $50,000. The current value of the company’s assets is $34,600, and the standard deviation of the return on a...

See Answer

Q: Marshall Corp. has a zero coupon bond that matures in five

Marshall Corp. has a zero coupon bond that matures in five years with a face value of $75,000. The current value of the company’s assets is $71,000, and the standard deviation of its return on assets...

See Answer

Q: In addition to the five factors discussed in the chapter, dividends

In addition to the five factors discussed in the chapter, dividends also affect the price of an option. The Black-Scholes option pricing model with dividends is: All of the variables are the same as...

See Answer

Q: The put-call parity condition is altered when dividends are paid

The put-call parity condition is altered when dividends are paid. The dividend-adjusted put-call parity formula is: S × e–dt + P = E × e–Rt + C where d is again the continuously compounded dividend yi...

See Answer

Q: A stock is currently selling for $73 per share. A

A stock is currently selling for $73 per share. A call option with an exercise price of $70 sells for $5.27 and expires in three months. If the riskfree rate of interest is 2.6 percent per year, compo...

See Answer

Q: A put option that expires in six months with an exercise price

A put option that expires in six months with an exercise price of $45 sells for $4.84. The stock is currently priced at $43, and the risk-free rate is 3.5 percent per year, compounded continuously. Wh...

See Answer

Q: A put option and a call option with an exercise price of

A put option and a call option with an exercise price of $65 and three months to expiration sell for $5.27 and $1.04, respectively. If the risk-free rate is 3.1 percent per year, compounded continuous...

See Answer

Q: What are the prices of a call option and a put option

What are the prices of a call option and a put option with the following characteristics? Stock price = $58 Exercise price = $60 Risk-free rate = 2.7% per year, compounded continuously Maturity = 4 mo...

See Answer

Q: What are the deltas of a call option and a put option

What are the deltas of a call option and a put option with the following characteristics? What does the delta of the option tell you?

See Answer

Q: You own a lot in Key West, Florida, that is

You own a lot in Key West, Florida, that is currently unused. Similar lots have recently sold for $1.35 million. Over the past five years, the price of land in the area has increased 7 percent per yea...

See Answer

Q: Dani Corp. has 5.5 million shares of common stock

Dani Corp. has 5.5 million shares of common stock outstanding. The current share price is $83, and the book value per share is $5. The company also has two bond issues outstanding. The first bond issu...

See Answer

Q: Pearl, Inc., has offered $197 million cash for all

Pearl, Inc., has offered $197 million cash for all of the common stock in Jam Corporation. Based on recent market information, Jam is worth $178 million as an independent operation. If the merger make...

See Answer

Q: Consider the following premerger information about Firm A and Firm B:

Consider the following premerger information about Firm A and Firm B: Assume that Firm A acquires Firm B via an exchange of stock at a price of $61 for each share of B’s stock. Bot...

See Answer

Q: Fly-By-Night Couriers is analyzing the possible acquisition of

Fly-By-Night Couriers is analyzing the possible acquisition of Flash-in-the-Pan Delivery. Neither firm has debt. The forecasts of Fly-By-Night show that the purchase would increase its annual aftertax...

See Answer

Q: Harrods PLC has a market value of £85 million and 4

Harrods PLC has a market value of £85 million and 4.5 million shares outstanding. Selfridge Department Store has a market value of £30 million and 1.8 million shares outstanding. Harrods is contemplat...

See Answer

Q: Consider the following premerger information about Firm X and Firm Y:

Consider the following premerger information about Firm X and Firm Y: Assume that Firm X acquires Firm Y by issuing new long-term debt for all the shares outstanding at a merger premium of $5 per sh...

See Answer

Q: Assume that the following balance sheets are stated at book value

Assume that the following balance sheets are stated at book value. Suppose that Meat Co. purchases Loaf, Inc. The fair market value of Loaf’s fixed assets is $11,500 versus the $8...

See Answer

Q: Silver Enterprises has acquired All Gold Mining in a merger transaction.

Silver Enterprises has acquired All Gold Mining in a merger transaction. Construct the balance sheet for the new corporation if the merger is treated as a purchase of interests for accounting purposes...

See Answer

Q: Penn Corp. is analyzing the possible acquisition of Teller Company.

Penn Corp. is analyzing the possible acquisition of Teller Company. Both firms have no debt. Penn believes the acquisition will increase its total aftertax annual cash flows by $1.45 million indefinit...

See Answer

Q: Three Guys Burgers, Inc., has offered $15.3

Three Guys Burgers, Inc., has offered $15.3 million for all of the common stock in Two Guys Fries Corp. The current market capitalization of Two Guys as an independent company is $12.1 million. Assume...

See Answer

Q: The shareholders of Bread Company have voted in favor of a buyout

The shareholders of Bread Company have voted in favor of a buyout offer from Butter Corporation. Information about each firm is given here: Bread’s shareholders will receive one sh...

See Answer

Q: Stock Y has a beta of 1.2 and an expected

Stock Y has a beta of 1.2 and an expected return of 11.5 percent. Stock Z has a beta of .80 and an expected return of 8.5 percent. If the risk-free rate is 3.2 percent and the market risk premium is 6...

See Answer

Q: In Problem 12, suppose the most recent dividend was $3

In Problem 12, suppose the most recent dividend was $3.85 and the dividend growth rate is 5 percent. Assume that the overall cost of debt is the weighted average of that implied by the two outstanding...

See Answer

Q: Consider the following premerger information about a bidding firm (Firm B

Consider the following premerger information about a bidding firm (Firm B) and a target firm (Firm T). Assume that both firms have no debt outstanding. Firm B has estimated that the value of the syn...

See Answer

Q: In Problem 8, are the shareholders of Firm T better off

In Problem 8, are the shareholders of Firm T better off with the cash offer or the stock offer? At what exchange ratio of B shares to T shares would the shareholders in T be indifferent between the tw...

See Answer

Q: You work for a nuclear research laboratory that is contemplating leasing a

You work for a nuclear research laboratory that is contemplating leasing a diagnostic scanner (leasing is a very common practice with expensive, high-tech equipment). The scanner costs $4.3 million, a...

See Answer

Q: The Wildcat Oil Company is trying to decide whether to lease or

The Wildcat Oil Company is trying to decide whether to lease or buy a new computerassisted drilling system for its oil exploration business. Management has decided that it must use the system to stay...

See Answer

Q: The Wildcat Oil Company is trying to decide whether to lease or

The Wildcat Oil Company is trying to decide whether to lease or buy a new computerassisted drilling system for its oil exploration business. Management has decided that it must use the system to stay...

See Answer

Q: The Wildcat Oil Company is trying to decide whether to lease or

The Wildcat Oil Company is trying to decide whether to lease or buy a new computerassisted drilling system for its oil exploration business. Management has decided that it must use the system to stay...

See Answer

Q: You work for a nuclear research laboratory that is contemplating leasing a

You work for a nuclear research laboratory that is contemplating leasing a diagnostic scanner (leasing is a very common practice with expensive, high-tech equipment). The scanner costs $4.3 million, a...

See Answer

Q: You work for a nuclear research laboratory that is contemplating leasing a

You work for a nuclear research laboratory that is contemplating leasing a diagnostic scanner (leasing is a very common practice with expensive, high-tech equipment). The scanner costs $4.3 million, a...

See Answer

Q: You work for a nuclear research laboratory that is contemplating leasing a

You work for a nuclear research laboratory that is contemplating leasing a diagnostic scanner (leasing is a very common practice with expensive, high-tech equipment). The scanner costs $4.3 million, a...

See Answer

Q: You work for a nuclear research laboratory that is contemplating leasing a

You work for a nuclear research laboratory that is contemplating leasing a diagnostic scanner (leasing is a very common practice with expensive, high-tech equipment). The scanner costs $4.3 million, a...

See Answer

Q: Ursala, Inc., has a target debt-equity ratio of

Ursala, Inc., has a target debt-equity ratio of .65. Its WACC is 10.4 percent, and the tax rate is 23 percent. a. If the company’s cost of equity is 14 percent, what is its pretax cost of debt? b. If...

See Answer

Q: You work for a nuclear research laboratory that is contemplating leasing a

You work for a nuclear research laboratory that is contemplating leasing a diagnostic scanner (leasing is a very common practice with expensive, high-tech equipment). The scanner costs $4.3 million, a...

See Answer

Q: The Wildcat Oil Company is trying to decide whether to lease or

The Wildcat Oil Company is trying to decide whether to lease or buy a new computerassisted drilling system for its oil exploration business. Management has decided that it must use the system to stay...

See Answer

Q: The Wildcat Oil Company is trying to decide whether to lease or

The Wildcat Oil Company is trying to decide whether to lease or buy a new computerassisted drilling system for its oil exploration business. Management has decided that it must use the system to stay...

See Answer

Q: The Wildcat Oil Company is trying to decide whether to lease or

The Wildcat Oil Company is trying to decide whether to lease or buy a new computerassisted drilling system for its oil exploration business. Management has decided that it must use the system to stay...

See Answer

Q: Based on the following information, calculate the expected return:

Based on the following information, calculate the expected return:

See Answer

Q: Based on the following information, calculate the expected return:

Based on the following information, calculate the expected return:

See Answer

Q: Based on the following information, calculate the expected return and standard

Based on the following information, calculate the expected return and standard deviation for Stock A and Stock B:

See Answer

Q: A portfolio is invested 45 percent in Stock G, 40 percent

A portfolio is invested 45 percent in Stock G, 40 percent in Stock J, and 15 percent in Stock K. The expected returns on these stocks are 11 percent, 9 percent, and 15 percent, respectively. What is t...

See Answer

Q: Consider the following information: / a.

Consider the following information: a. What is the expected return on an equally weighted portfolio of these three stocks? b. What is the variance of a portfolio invested 20 percent each in A and B...

See Answer

Q: Consider the following information: / a.

Consider the following information: a. Your portfolio is invested 30 percent each in A and C, and 40 percent in B. What is the expected return of the portfolio? b. What is the variance of this portf...

See Answer

Q: Given the following information for Lightning Power Co., find the WACC

Given the following information for Lightning Power Co., find the WACC. Assume the company’s tax rate is 21 percent. Debt: 12,000 bonds with a 4.6 percent coupon outstanding, $1,000 par value, 25 year...

See Answer

Q: You own a stock portfolio invested 15 percent in Stock Q,

You own a stock portfolio invested 15 percent in Stock Q, 20 percent in Stock R, 30 percent in Stock S, and 35 percent in Stock T. The betas for these four stocks are .79, 1.23, 1.13, and 1.36, respec...

See Answer

Q: You own a portfolio equally invested in a risk-free asset

You own a portfolio equally invested in a risk-free asset and two stocks. If one of the stocks has a beta of 1.34 and the total portfolio is equally as risky as the market, what must the beta be for t...

See Answer

Q: A stock has a beta of 1.15, the expected

A stock has a beta of 1.15, the expected return on the market is 11.3 percent, and the risk-free rate is 3.6 percent. What must the expected return on this stock be?

See Answer

Q: A stock has an expected return of 11.4 percent,

A stock has an expected return of 11.4 percent, the risk-free rate is 3.9 percent, and the market risk premium is 6.8 percent. What must the beta of this stock be?

See Answer

Q: A stock has an expected return of 11.85 percent,

A stock has an expected return of 11.85 percent, its beta is 1.08, and the risk-free rate is 3.9 percent. What must the expected return on the market be?

See Answer

Q: A stock has an expected return of 10.45 percent,

A stock has an expected return of 10.45 percent, its beta is .85, and the expected return on the market is 11.8 percent. What must the risk-free rate be?

See Answer

Q: What are the portfolio weights for a portfolio that has 145 shares

What are the portfolio weights for a portfolio that has 145 shares of Stock A that sell for $47 per share and 200 shares of Stock B that sell for $21 per share?

See Answer

Q: You own a portfolio that has $4,450 invested in

You own a portfolio that has $4,450 invested in Stock A and $9,680 invested in Stock B. If the expected returns on these stocks are 8 percent and 11 percent, respectively, what is the expected return...

See Answer

Q: You own a portfolio that is invested 15 percent in Stock X

You own a portfolio that is invested 15 percent in Stock X, 35 percent in Stock Y, and 50 percent in Stock Z. The expected returns on these three stocks are 9 percent, 15 percent, and 12 percent, resp...

See Answer

Q: You have $10,000 to invest in a stock portfolio

You have $10,000 to invest in a stock portfolio. Your choices are Stock X with an expected return of 12.4 percent and Stock Y with an expected return of 10.1 percent. If your goal is to create a portf...

See Answer

Q: Lingenburger Cheese Corporation has 6.4 million shares of common stock

Lingenburger Cheese Corporation has 6.4 million shares of common stock outstanding, 200,000 shares of 3.8 percent preferred stock outstanding, par value of $100, and 120,000 bonds with a semiannual co...

See Answer

Q: In the aggregate, debt offerings are much more common than equity

In the aggregate, debt offerings are much more common than equity offerings and typically much larger as well. Why?

See Answer

Q: Why are the costs of selling equity so much larger than the

Why are the costs of selling equity so much larger than the costs of selling debt? Answer: From the previous question, economies of scale are part of the answer. Beyond this, debt issues are ea...

See Answer

Q: Why do noninvestment-grade bonds have much higher direct costs than

Why do noninvestment-grade bonds have much higher direct costs than investment-grade issues?

See Answer

Q: Why is underpricing not a great concern with bond offerings? Use

Why is underpricing not a great concern with bond offerings? Use the following information to answer the next three questions. Zipcar, the car-sharing company, went public in April 2011. Assisted by t...

See Answer

Q: The Zipcar IPO was underpriced by about 56 percent. Should Zipcar

The Zipcar IPO was underpriced by about 56 percent. Should Zipcar be upset at Goldman over the underpricing?

See Answer

Q: In the previous question, how would it affect your thinking to

In the previous question, how would it affect your thinking to know that the company was incorporated about 10 years earlier, had only $186 million in revenues in 2010, and had never earned a profit?...

See Answer

Q: In the previous two questions, how would it affect your thinking

In the previous two questions, how would it affect your thinking to know that in addition to the 9.68 million shares offered in the IPO, Zipcar had an additional 30 million shares outstanding? Of thos...

See Answer

Q: For initial public offerings of common stock, 2017 was a slow

For initial public offerings of common stock, 2017 was a slow year, with about $24.53 billion raised by the process. Relatively few of the 108 firms involved paid cash dividends. Why do you think that...

See Answer

Q: How do you think this tax law change affected ex-dividend

How do you think this tax law change affected ex-dividend stock prices?

See Answer

Q: The following material represents the cover page and summary of the prospectus

The following material represents the cover page and summary of the prospectus for the initial public offering of the Pest Investigation Control Corporation (PICC), which is going public tomorrow with...

See Answer

Q: An all-equity firm is considering the following projects:

An all-equity firm is considering the following projects: The T-bill rate is 4 percent, and the expected return on the market is 12 percent. a. Which projects have a higher expected return than the...

See Answer

Q: How do you think this tax law change affected the relative attractiveness

How do you think this tax law change affected the relative attractiveness of stock repurchases compared to dividend payments?

See Answer

Q: On Tuesday, December 5, Hometown Power Co.’s board

On Tuesday, December 5, Hometown Power Co.’s board of directors declares a dividend of 75 cents per share payable on Wednesday, January 17, to shareholders of record as of Wednesday, January 3. When i...

See Answer

Q: If a U.S. company exports its goods to Japan

If a U.S. company exports its goods to Japan, how would it use a futures contract on Japanese yen to hedge its exchange rate risk? Would it buy or sell yen futures? In answering, assume that the excha...

See Answer

Q: In 2019, Japan-based Mitsubishi Corporation acquired Dutch power provider

In 2019, Japan-based Mitsubishi Corporation acquired Dutch power provider Eneco for $4.52 billion. Is this a horizontal or vertical acquisition? How do you suppose Eneco’s nationality affected Mitsubi...

See Answer

Q: In 2019, activist investor Elliott Management was pressuring eBay to sell

In 2019, activist investor Elliott Management was pressuring eBay to sell StubHub. The sale was completed in February 2020, when Ebay sold StubHub to Viagogo. Why might investors prefer that a company...

See Answer

Q: Explain why the aftertax borrowing rate is the appropriate discount rate to

Explain why the aftertax borrowing rate is the appropriate discount rate to use in lease evaluation.

See Answer

Q: Suppose your company needs $43 million to build a new assembly

Suppose your company needs $43 million to build a new assembly line. Your target debt-equity ratio is .75. The flotation cost for new equity is 6 percent, but the flotation cost for debt is only 2 per...

See Answer

Q: Shinedown Company needs to raise $95 million to start a new

Shinedown Company needs to raise $95 million to start a new project and will raise the money by selling new bonds. The company will generate no internal equity for the foreseeable future. The company...

See Answer

Q: The Swanson Corporation’s common stock has a beta of 1.07

The Swanson Corporation’s common stock has a beta of 1.07. If the risk-free rate is 3.4 percent and the expected return on the market is 11 percent, what is the company’s cost of equity capital?

See Answer

Q: Lebleu, Inc., is considering a project that will result in

Lebleu, Inc., is considering a project that will result in initial aftertax cash savings of $2.9 million at the end of the first year, and these savings will grow at a rate of 2 percent per year indef...

See Answer

Q: Ginger Industries stock has a beta of 1.08. The

Ginger Industries stock has a beta of 1.08. The company just paid a dividend of $.85, and the dividends are expected to grow at 4 percent. The expected return on the market is 11.3 percent, and Treasu...

See Answer

Q: In Problem 14, what is the break-even price per

In Problem 14, what is the break-even price per unit that should be charged under the new credit policy? Assume that the sales figure under the new policy is 3,310 units and all other values remain th...

See Answer

Q: Derry Corp. is expected to have an EBIT of $2

Derry Corp. is expected to have an EBIT of $2.1 million next year. Increases in depreciation, the increase in net working capital, and capital spending are expected to be $165,000, $80,000, and $120,0...

See Answer

Q: In the previous problem, instead of a perpetual growth rate in

In the previous problem, instead of a perpetual growth rate in adjusted cash flow from assets, you decide to calculate the terminal value of the company with the price-sales ratio. You believe that Ye...

See Answer

Q: You have looked at the current financial statements for J&R

You have looked at the current financial statements for J&R Homes, Co. The company has an EBIT of $3.35 million this year. Depreciation, the increase in net working capital, and capital spending were...

See Answer

Q: In the previous problem, suppose you believe that sales in five

In the previous problem, suppose you believe that sales in five years will be $45.5 million and the price-sales ratio will be 2.15. What is the share price now? Problem 26: You have looked at the cur...

See Answer

Q: Landman Corporation (LC) manufactures time series photographic equipment. It

Landman Corporation (LC) manufactures time series photographic equipment. It is currently at its target debt-equity ratio of .60. It’s considering building a new $73 million manufacturing facility. Th...

See Answer

Q: Stock in Jansen Industries has a beta of 1.05.

Stock in Jansen Industries has a beta of 1.05. The market risk premium is 7 percent, and T-bills are currently yielding 3.5 percent. The company’s most recent dividend was $2.45 per share, and dividen...

See Answer

Q: Suppose Wacken, Ltd., just issued a dividend of $2

Suppose Wacken, Ltd., just issued a dividend of $2.73 per share on its common stock. The company paid dividends of $2.31, $2.39, $2.48, and $2.58 per share in the last four years. If the stock current...

See Answer

Q: Savers has an issue of preferred stock with a stated dividend of

Savers has an issue of preferred stock with a stated dividend of $3.85 that just sold for $87 per share. What is the bank’s cost of preferred stock?

See Answer

Q: Sunrise, Inc., is trying to determine its cost of debt

Sunrise, Inc., is trying to determine its cost of debt. The firm has a debt issue outstanding with 23 years to maturity that is quoted at 96 percent of face value. The issue makes semiannual payments...

See Answer

Q: Jiminy’s Cricket Farm issued a 30-year, 4.5

Jiminy’s Cricket Farm issued a 30-year, 4.5 percent semiannual bond three years ago. The bond currently sells for 104 percent of its face value. The company’s tax rate is 22 percent. a. What is the p...

See Answer

Q: In the previous problem, what would the risk-free rate

In the previous problem, what would the risk-free rate have to be for the two stocks to be correctly priced? Problem 18: Stock Y has a beta of 1.2 and an expected return of 11.5 percent. Stock Z has...

See Answer

Q: For the firm in Problem 7, suppose the book value of

For the firm in Problem 7, suppose the book value of the debt issue is $75 million. In addition, the company has a second debt issue on the market, a zero coupon bond with eight years left to maturity...

See Answer

Q: Ninecent Corporation has a target capital structure of 70 percent common stock

Ninecent Corporation has a target capital structure of 70 percent common stock, 5 percent preferred stock, and 25 percent debt. Its cost of equity is 11 percent, the cost of preferred stock is 5 perce...

See Answer

Q: Hassinah, Inc., is proposing a rights offering. Presently there

Hassinah, Inc., is proposing a rights offering. Presently there are 435,000 shares outstanding at $71 each. There will be 50,000 new shares offered at $64 each. a. What is the new market value of the...

See Answer

Q: In Problem 10, what would the ROE on the investment have

In Problem 10, what would the ROE on the investment have to be if we wanted the price after the offering to be $75 per share? (Assume the PE ratio remains constant.) What is the NPV of this investment...

See Answer

Q: Bell Buckle Mfg. is considering a rights offer. The company

Bell Buckle Mfg. is considering a rights offer. The company has determined that the ex-rights price would be $71. The current price is $76 per share, and there are 29 million shares outstanding. The r...

See Answer

Q: Nougat Corp. wants to raise $5.1 million via

Nougat Corp. wants to raise $5.1 million via a rights offering. The company currently has 530,000 shares of common stock outstanding that sell for $55 per share. Its underwriter has set a subscription...

See Answer

Q: The Clifford Corporation has announced a rights offer to raise $25

The Clifford Corporation has announced a rights offer to raise $25 million for a new journal, the Journal of Financial Excess. This journal will review potential articles after the author pays a nonre...

See Answer

Q: The Tennis Shoe Co. has concluded that additional equity financing will

The Tennis Shoe Co. has concluded that additional equity financing will be needed to expand operations and that the needed funds will be best obtained through a rights offering. It has correctly deter...

See Answer

Q: The Woods Co. and the Koepka Co. have both announced

The Woods Co. and the Koepka Co. have both announced IPOs at $40 per share. One of these is undervalued by $12, and the other is overvalued by $5, but you have no way of knowing which is which. You pl...

See Answer

Q: The Meadows Corporation needs to raise $75 million to finance its

The Meadows Corporation needs to raise $75 million to finance its expansion into new markets. The company will sell new shares of equity via a general cash offering to raise the needed funds. If the o...

See Answer

Q: A stock has a beta of 1.19 and an expected

A stock has a beta of 1.19 and an expected return of 12.4 percent. A risk-free asset currently earns 2.7 percent. a. What is the expected return on a portfolio that is equally invested in the two asse...

See Answer

Q: In Problem 5, if the SEC filing fee and associated administrative

In Problem 5, if the SEC filing fee and associated administrative expenses of the offering are $1.9 million, how many shares need to be sold? Problem 5: The Meadows Corporation needs to raise $75 mil...

See Answer

Q: The Telwar Co. has just gone public. Under a firm

The Telwar Co. has just gone public. Under a firm commitment agreement, the company received $25.11 for each of the 30 million shares sold. The initial offering price was $27 per share, and the stock...

See Answer

Q: Damron, Inc., has 250,000 shares of stock outstanding

Damron, Inc., has 250,000 shares of stock outstanding. Each share is worth $81, so the company’s market value of equity is $20,250,000. Suppose the firm issues 40,000 new shares at the following price...

See Answer

Q: Marker, Inc., wishes to expand its facilities. The company

Marker, Inc., wishes to expand its facilities. The company currently has 5 million shares outstanding and no debt. The stock sells for $64 per share, but the book value per share is $19. Net income is...

See Answer

Q: Fujita, Inc., has no debt outstanding and a total market

Fujita, Inc., has no debt outstanding and a total market value of $222,000. Earnings before interest and taxes, EBIT, are projected to be $18,000 if economic conditions are normal. If there is strong...

See Answer

Q: Sugar Skull Corp. uses no debt. The weighted average cost

Sugar Skull Corp. uses no debt. The weighted average cost of capital is 7.9 percent. If the current market value of the equity is $15.6 million and there are no taxes, what is EBIT?

See Answer

Q: In Problem 10, suppose the corporate tax rate is 22 percent

In Problem 10, suppose the corporate tax rate is 22 percent. What is EBIT in this case? What is the WACC? Explain. Problem 10: Sugar Skull Corp. uses no debt. The weighted average cost of capital is...

See Answer

Q: Solar Industries has a debt-equity ratio of 1.25

Solar Industries has a debt-equity ratio of 1.25. Its WACC is 7.8 percent, and its cost of debt is 4.7 percent. The corporate tax rate is 21 percent. a. What is the company’s cost of equity capital? b...

See Answer

Q: Navarro Corp. has no debt but can borrow at 5.

Navarro Corp. has no debt but can borrow at 5.9 percent. The firm’s WACC is currently 9.2 percent, and the tax rate is 21 percent. a. What is the company’s cost of equity? b. If the firm converts to 2...

See Answer

Q: Fields & Co. expects its EBIT to be $125,

Fields & Co. expects its EBIT to be $125,000 every year forever. The firm can borrow at 7 percent. The company currently has no debt, and its cost of equity is 12 percent. If the tax rate is 24 percen...

See Answer

Q: Cookies ’n Cream, Inc., recently issued new securities to finance

Cookies ’n Cream, Inc., recently issued new securities to finance a new TV show. The project cost $45 million, and the company paid $2.2 million in flotation costs. In addition, the equity issued had...

See Answer

Q: In Problem 14, what is the cost of equity after recapitalization

In Problem 14, what is the cost of equity after recapitalization? What is the WACC? What are the implications for the firm’s capital structure decision? Problem 14: Fields & Co. expects its EBIT to b...

See Answer

Q: Tool Manufacturing has an expected EBIT of $57,000 in

Tool Manufacturing has an expected EBIT of $57,000 in perpetuity and a tax rate of 21 percent. The firm has $134,000 in outstanding debt at an interest rate of 5.35 percent, and its unlevered cost of...

See Answer

Q: Tempest Corporation expects an EBIT of $37,700 every year

Tempest Corporation expects an EBIT of $37,700 every year forever. The company currently has no debt, and its cost of equity is 11 percent. The tax rate is 22 percent. a. What is the current value of...

See Answer

Q: The Day Company and the Knight Company are identical in every respect

The Day Company and the Knight Company are identical in every respect except that Day is not levered. Financial information for the two firms appears in the following table. All earnings streams are p...

See Answer

Q: Repeat parts (a) and (b) in Problem 1

Repeat parts (a) and (b) in Problem 1 assuming the company has a tax rate of 21 percent, a market-to-book ratio of 1.0 before recapitalization, and the stock price changes according to M&M. Problem 1...

See Answer

Q: Repeat parts (a) and (b) in Problem 1

Repeat parts (a) and (b) in Problem 1 assuming the company has a tax rate of 21 percent, a market-to-book ratio of 1.0, and the stock price remains constant. Problem 1: Fujita, Inc., has no debt outs...

See Answer

Q: In Problem 3, suppose instead that you assume the company has

In Problem 3, suppose instead that you assume the company has a market-to-book ratio of 1.0 before recapitalization and the stock price changes according to M&M. How would this affect your answer? Pr...

See Answer

Q: Suppose the company in Problem 1 has a market-tobook ratio

Suppose the company in Problem 1 has a market-tobook ratio of 1.0 and the stock price remains constant. a. Calculate return on equity (ROE) under each of the three economic scenarios before any debt i...

See Answer

Q: Foundation, Inc., is comparing two different capital structures: an

Foundation, Inc., is comparing two different capital structures: an all-equity plan (Plan I) and a levered plan (Plan II). Under Plan I, the company would have 145,000 shares of stock outstanding. Und...

See Answer

Q: In Problem 4, use M&M Proposition I to find

In Problem 4, use M&M Proposition I to find the price per share of equity under each of the two proposed plans. What is the value of the firm? Problem 4: Foundation, Inc., is comparing two different...

See Answer

Q: Ying Import has several bond issues outstanding, each making semiannual interest

Ying Import has several bond issues outstanding, each making semiannual interest payments. The bonds are listed in the following table. If the corporate tax rate is 22 percent, what is the aftertax co...

See Answer

Q: Dickson Corp. is comparing two different capital structures. Plan I

Dickson Corp. is comparing two different capital structures. Plan I would result in 12,700 shares of stock and $100,050 in debt. Plan II would result in 9,800 shares of stock and $226,200 in debt. The...

See Answer

Q: Ignoring taxes in Problem 6, what is the price per share

Ignoring taxes in Problem 6, what is the price per share of equity under Plan I? Plan II? What principle is illustrated by your answers? Problem 6: Dickson Corp. is comparing two different capital st...

See Answer

Q: Finch, Inc., is debating whether to convert its allequity capital

Finch, Inc., is debating whether to convert its allequity capital structure to one that is 30 percent debt. Currently, there are 6,500 shares outstanding, and the price per share is $51. EBIT is expec...

See Answer

Q: ABC Co. and XYZ Co. are identical firms in all

ABC Co. and XYZ Co. are identical firms in all respects except for their capital structure. ABC is all-equity financed with $680,000 in stock. XYZ uses both stock and perpetual debt; its stock is wort...

See Answer

Q: Gatto, Inc., has declared a $5.85 per

Gatto, Inc., has declared a $5.85 per share dividend. Suppose capital gains are not taxed, but dividends are taxed at 15 percent. New IRS regulations require that taxes be withheld at the time the div...

See Answer

Q: You own 1,000 shares of stock in Avondale Corporation.

You own 1,000 shares of stock in Avondale Corporation. You will receive a $3.45 per share dividend in one year. In two years, the company will pay a liquidating dividend of $62 per share. The required...

See Answer

Q: In Problem 10, suppose you want only $1,500

In Problem 10, suppose you want only $1,500 total in dividends the first year. What will your homemade dividend be in two years? Problem 10: You own 1,000 shares of stock in Avondale Corporation. You...

See Answer

Q: Erna Corporation is evaluating an extra dividend versus a share repurchase.

Erna Corporation is evaluating an extra dividend versus a share repurchase. In either case, $53,500 would be spent. Current earnings are $1.79 per share, and the stock currently sells for $64 per shar...

See Answer

Q: The Gecko Company and the Gordon Company are two firms that have

The Gecko Company and the Gordon Company are two firms that have the same business risk but different dividend policies. Gecko pays no dividend, whereas Gordon has an expected dividend yield of 2.9 pe...

See Answer

Q: As discussed in the text, in the absence of market imperfections

As discussed in the text, in the absence of market imperfections and tax effects, we would expect the share price to decline by the amount of the dividend payment when the stock goes ex dividend. Once...

See Answer

Q: Consider the following information about three stocks: /

Consider the following information about three stocks: a. If your portfolio is invested 40 percent each in A and B and 20 percent in C, what is the portfolio expected return? The variance? The stand...

See Answer

Q: National Business Machine Co. (NBM) has $5 million

National Business Machine Co. (NBM) has $5 million of extra cash after taxes have been paid. NBM has two choices to make use of this cash. One alternative is to invest the cash in financial assets. Th...

See Answer

Q: After completing its capital spending for the year, Carlson Manufacturing has

After completing its capital spending for the year, Carlson Manufacturing has $1,000 extra cash. Carlson’s managers must choose between investing the cash in Treasury bonds that yield 3 percent or pay...

See Answer

Q: The owners’ equity accounts for Vulcano International are shown here:

The owners’ equity accounts for Vulcano International are shown here: Common stock ($.50 par value) …………….. $ 20,000 Capital surplus …………………………………….. 210,000 Retained earnings ………………………………… 587,300 T...

See Answer

Q: For the company in Problem 2, show how the equity accounts

For the company in Problem 2, show how the equity accounts will change if: a. The company declares a 4-for-1 stock split. How many shares are outstanding now? What is the new par value per share? b. T...

See Answer

Q: Stockton Mineral Operations, Inc. (SMO), currently has 540

Stockton Mineral Operations, Inc. (SMO), currently has 540,000 shares of stock outstanding that sell for $83 per share. Assuming no market imperfections or tax effects exist, what will the share price...

See Answer

Q: The balance sheet for Quinn Corp. is shown here in market

The balance sheet for Quinn Corp. is shown here in market value terms. There are 12,000 shares of stock outstanding. The company has declared a dividend of $1.45 per share. The stock goes ex dividen...

See Answer

Q: In Problem 5, suppose the company has announced it is going

In Problem 5, suppose the company has announced it is going to repurchase $17,400 worth of stock. What effect will this transaction have on the equity of the firm? How many shares will be outstanding?...

See Answer

Q: The market value balance sheet for Murray Manufacturing is shown here.

The market value balance sheet for Murray Manufacturing is shown here. The company has declared a 25 percent stock dividend. The stock goes ex dividend tomorrow (the chronology for a stock dividend is...

See Answer

Q: The company with the common equity accounts shown here has declared a

The company with the common equity accounts shown here has declared a 15 percent stock dividend when the market value of its stock is $64 per share. What effects will the distribution of the stock div...

See Answer

Q: In Problem 8, suppose the company instead decides on a 4

In Problem 8, suppose the company instead decides on a 4-for-1 stock split. The firm’s 75-cent per-share cash dividend on the new (postsplit) shares represents an increase of 10 percent over last year...

See Answer

Q: You want to create a portfolio equally as risky as the market

You want to create a portfolio equally as risky as the market, and you have $1,000,000 to invest. Given this information, fill in the rest of the following table:

See Answer

Q: The following is the sales budget for Lemonis, Inc., for

The following is the sales budget for Lemonis, Inc., for the first quarter of 2021: Credit sales are collected as follows: 65 percent in the month of the sale. 20 percent in the month after the sale...

See Answer

Q: Here are some important figures from the budget of Nashville Nougats,

Here are some important figures from the budget of Nashville Nougats, Inc., for the second quarter of 2021: The company predicts that 5 percent of its credit sales will never be collected, 35 percen...

See Answer

Q: Below are the most recent balance sheets for Country Kettles, Inc

Below are the most recent balance sheets for Country Kettles, Inc. Excluding accumulated depreciation, determine whether each item is a source or a use of cash and the amount:

See Answer

Q: You’ve worked out a line of credit arrangement that allows you to

You’ve worked out a line of credit arrangement that allows you to borrow up to $50 million at any time. The interest rate is .37 percent per month. In addition, 4 percent of the amount that you borrow...

See Answer

Q: A bank offers your firm a revolving credit arrangement for up to

A bank offers your firm a revolving credit arrangement for up to $75 million at an interest rate of 1.75 percent per quarter. The bank also requires you to maintain a compensating balance of 5 percent...

See Answer

Q: In exchange for a $300 million fixed commitment line of credit

In exchange for a $300 million fixed commitment line of credit, your firm has agreed to do the following: 1. Pay 1.93 percent per quarter on any funds actually borrowed. 2. Maintain a 4.5 percent comp...

See Answer

Q: Cheap Money Bank offers your firm a discount interest loan at 9

Cheap Money Bank offers your firm a discount interest loan at 9.85 percent for up to $25 million and, in addition, requires you to maintain a 4.5 percent compensating balance against the amount borrow...

See Answer

Q: Cori’s Corp. has an equity value of $14,735

Cori’s Corp. has an equity value of $14,735. Long-term debt is $8,300. Net working capital, other than cash, is $2,850. Fixed assets are $18,440. How much cash does the company have? If current liabil...

See Answer

Q: The Morning Jolt Coffee Company has projected the following quarterly sales amounts

The Morning Jolt Coffee Company has projected the following quarterly sales amounts for the coming year: a. Accounts receivable at the beginning of the year are $335. The company has a 45-day collec...

See Answer

Q: Consider the following financial statement information for the Newk Corporation:

Consider the following financial statement information for the Newk Corporation: Assume all sales are on credit. Calculate the operating and cash cycles. How do you interpret your answer?

See Answer

Q: Your company is deciding whether to invest in a new machine.

Your company is deciding whether to invest in a new machine. The new machine will increase cash flow by $485,000 per year. You believe the technology used in the machine has a 10-year life; in other w...

See Answer

Q: You chair the compensation committee of the board of directors of Androscoggin

You chair the compensation committee of the board of directors of Androscoggin Copper. A consultant suggests two stock-option packages for the CEO: a. A conventional stock-option plan, with the exerci...

See Answer

Q: On the London Metals Exchange, the price for copper to be

On the London Metals Exchange, the price for copper to be delivered in one year is $5,500 a ton. (Note: Payment is made when the copper is delivered.) The risk-free interest rate is 2% and the expecte...

See Answer

Q: To finance the Madison County project (see Problem 10), Wishing

To finance the Madison County project (see Problem 10), Wishing Well needs to arrange an additional $80 million of long-term debt and make a $20 million equity issue. Underwriting fees, spreads, and o...

See Answer

Q: Consider another perpetual project like the crusher described in Section 19-

Consider another perpetual project like the crusher described in Section 19-1. Its initial investment is $1,000,000, and the expected cash inflow is $95,000 a year in perpetuity. The opportunity cost...

See Answer

Q: Note Figure 20.12 below. Match each diagram, (

Note Figure 20.12 below. Match each diagram, (a) and (b), with one of the following positions: 1) Call buyer 2) Call seller 3) Put buyer 4) Put seller

See Answer

Q: Three six-month call options are traded on Hogswill stock:

Three six-month call options are traded on Hogswill stock: How would you make money by trading in Hogswill options? (Hint: Draw a graph with the option price on the vertical axis and the ratio of stoc...

See Answer

Q: Dr. Livingstone I. Presume holds £600,000 in

Dr. Livingstone I. Presume holds £600,000 in East African gold stocks. Bullish as he is on gold mining, he requires absolute assurance that at least £500,000 will be availabl...

See Answer

Q: Option traders often refer to “straddles” and “butterflies.”

Option traders often refer to “straddles” and “butterflies.” Here is an example of each: 1) Straddle: Buy one call with exercise price of $100 and simultaneously buy one put with exercise price of $10...

See Answer

Q: Ms. Higden has been offered yet another incentive scheme (see

Ms. Higden has been offered yet another incentive scheme (see Section 20-2). She will receive a bonus of $500,000 if the stock price at the end of the year is $120 or more; otherwise she will receive...

See Answer

Q: In April 2017, Facebook’s stock price was about $145.

In April 2017, Facebook’s stock price was about $145. An eight-month call on the stock, with an exercise price of $145, sold for $10.18. The risk-free interest rate was 1% a year. How much would you b...

See Answer

Q: How does the price of a call option respond to the following

How does the price of a call option respond to the following changes, other things equal? Does the call price go up or down? a. Stock price increases. b. Exercise price is increased. c. Risk-free rate...

See Answer

Q: Respond to the following statements. a. “I’m a

Respond to the following statements. a. “I’m a conservative investor. I’d much rather hold a call option on a safe stock like Exxon Mobil than a volatile stock like Amazon.” b. “I bought an American...

See Answer

Q: Photographic laboratories recover and recycle the silver used in photographic film.

Photographic laboratories recover and recycle the silver used in photographic film. Stikine River Photo is considering purchase of improved equipment for their laboratory at Telegraph Creek. Here is t...

See Answer

Q: Look at actual trading prices of call options on stocks to check

Look at actual trading prices of call options on stocks to check whether they behave as the theory presented in this chapter predicts. For example, a. Follow several options as they approach maturity....

See Answer

Q: Problem 21 considered an arbitrage opportunity involving an American option. Suppose

Problem 21 considered an arbitrage opportunity involving an American option. Suppose that this option was a European call. Show that there is a similar possible arbitrage profit.

See Answer

Q: The stock price of Heavy Metal (HM) changes only once

The stock price of Heavy Metal (HM) changes only once a month: Either it goes up by 20% or it falls by 16.7%. Its price now is $40. The interest rate is 1% per month. a. What is the value of a one-mon...

See Answer

Q: Take another look at our two-step binomial trees for Amazon

Take another look at our two-step binomial trees for Amazon in Figure 21.2. Use the risk-neutral method to value six-month call and put options with an exercise price of $750. Assume the Amazon stock...

See Answer

Q: a. Use the Black-Scholes formula to find the value

a. Use the Black-Scholes formula to find the value of the following call option. i. Time to expiration 1 year. ii. Standard deviation 40% per year. iii. Exercise price $50. iv. Stock price $50. v. Int...

See Answer

Q: Use the Black–Scholes program to estimate how much you should

Use the Black–Scholes program to estimate how much you should be prepared to pay to insure the value of your pension fund portfolio for the coming year. Make reasonable assumptions about the volatilit...

See Answer

Q: For which of the following options might it be rational to exercise

For which of the following options might it be rational to exercise before maturity? Explain briefly why or why not. a. American put on a non-dividend-paying stock. b. American call—the dividend payme...

See Answer

Q: Calculate and compare the risk (betas) of the following investments

Calculate and compare the risk (betas) of the following investments: (a) a share of Amazon stock; (b) a one-year call option on Amazon; (c) a one-year put option; (d) a portfolio consisting of a share...

See Answer

Q: Respond to the following comments. a. “You don’t

Respond to the following comments. a. “You don’t need option pricing theories to value flexibility. Just use a decision tree. Discount the cash flows in the tree at the company cost of capital.” b. “...

See Answer

Q: Alert financial managers can create real options. Give three or four

Alert financial managers can create real options. Give three or four possible examples.

See Answer

Q: True or false? a. A firm that earns the

True or false? a. A firm that earns the opportunity cost of capital is earning economic rents. b. A firm that invests in positive-NPV ventures expects to earn economic rents. c. Financial managers sho...

See Answer

Q: Describe each of the following situations in the language of options:

Describe each of the following situations in the language of options: a. Drilling rights to undeveloped heavy crude oil in Northern Alberta. Development and production of the oil is a negative-NPV end...

See Answer

Q: Look back at the Malted Herring option in Section 22-2

Look back at the Malted Herring option in Section 22-2. How did the company’s analysts estimate the present value of the project? It turns out that they assumed that the probability of low demand was...

See Answer

Q: Flip back to Tables 6.2 and 6.3,

Flip back to Tables 6.2 and 6.3, where we assumed an economic life of seven years for IM&C’s guano plant. What’s wrong with that assumption? How would you undertake a more complete analysis?

See Answer

Q: Take another look at the perpetual crusher example in Section 22-

Take another look at the perpetual crusher example in Section 22-3. Construct a sensitivity analysis showing how the value of the abandonment put changes depending on the standard deviation of the pro...

See Answer

Q: Construct a sensitivity analysis of the value of the pharmaceutical R&

Construct a sensitivity analysis of the value of the pharmaceutical R&D project described in Figure 22.8. What input assumptions are most critical for the NPV of the project? Be sure to check the inpu...

See Answer

Q: In February 2018, Aaa bonds yielded 3.38%, Baa

In February 2018, Aaa bonds yielded 3.38%, Baa bonds yielded 4.51%, and comparable Treasuries yielded 2.86%. a. What was the credit spread on Aaa bonds? b. What was the spread on Baa bonds? c. What do...

See Answer

Q: It is 2030 and the yields on corporate bonds are as follows

It is 2030 and the yields on corporate bonds are as follows: Tau Corp wishes to raise $10 million by an issue of 9% 10-year bonds. What will be the likely issue price (as a percent of face value) if T...

See Answer

Q: Other things equal, would you expect the difference between the price

Other things equal, would you expect the difference between the price of a Treasury bond and a corporate bond to increase or decrease with a. The company’s business risk? b. The degree of leverage? c...

See Answer

Q: Square File’s assets are worth $100. It has $80

Square File’s assets are worth $100. It has $80 of zero-coupon debt outstanding that is due to be repaid at the end of two years. The risk-free interest rate is 5%, and the standard deviation of the r...

See Answer

Q: A friend has mentioned that she has read somewhere that the following

A friend has mentioned that she has read somewhere that the following variables can be used to predict bankruptcy: (a) the company debt ratio; (b) the interest coverage; (c) the amount of cash relativ...

See Answer

Q: We characterized the interstate rail lines owned by major U.S

We characterized the interstate rail lines owned by major U.S. railroads as “strategic assets” that could generate increased profits. In what conditions would you expect these assets to generate econo...

See Answer

Q: Look back at Section 23-4. Suppose that the standard

Look back at Section 23-4. Suppose that the standard deviation of the return on Upsilon’s assets is 50%. Recalculate the probability that the company will default.

See Answer

Q: Look at Table 24.1: a. The AMAT

Look at Table 24.1: a. The AMAT bond was issued on June 8, 2011, at 99.592%. How much would you have to pay to buy one bond delivered on June 15? Don’t forget to include accrued interest. b. When is t...

See Answer

Q: Suppose that you are a banker responsible for approving corporate loans.

Suppose that you are a banker responsible for approving corporate loans. Nine firms are seeking secured loans. They offer the following assets as collateral: a. Firm A, a heating oil distributor, offe...

See Answer

Q: Complete the passage below by selecting the most appropriate terms from the

Complete the passage below by selecting the most appropriate terms from the following list: floating lien, revolving credit, medium-term note, warehouse receipt, unsecured, commitment fee, commercial...

See Answer

Q: Term loans usually require firms to pay a fluctuating interest rate.

Term loans usually require firms to pay a fluctuating interest rate. For example, the interest rate may be set at 1% over LIBOR. LIBOR can sometimes vary by several percentage points within a single...

See Answer

Q: Find the terms and conditions of a recent bond issue and compare

Find the terms and conditions of a recent bond issue and compare them with those of the AMAT issue.

See Answer

Q: Select the most appropriate term from within the parentheses: a

Select the most appropriate term from within the parentheses: a. (High-grade bonds/Low-grade bonds) generally have only light sinking-fund requirements. b. Equipment trust certificates are usually iss...

See Answer

Q: Suppose that the AMAT bond was issued at face value and that

Suppose that the AMAT bond was issued at face value and that investors continue to demand a yield of 5.85%. Sketch what you think would happen to the bond price as the first interest payment date appr...

See Answer

Q: a. Look at Table 24.1. Suppose that AMAT

a. Look at Table 24.1. Suppose that AMAT decides to call the bond one year before it is due to expire. The interest rate on one-year Treasury bonds is 2%. What price must AMAT pay to call the bonds? b...

See Answer

Q: Maple Aircraft has issued a 4¾% convertible subordinated debenture due 2023

Maple Aircraft has issued a 4¾% convertible subordinated debenture due 2023. The conversion price is $47.00 and the debenture is callable at 102.75% of face value. The market price of the convertible...

See Answer

Q: Thanks to acquisition of a key patent, your company now has

Thanks to acquisition of a key patent, your company now has exclusive production rights for barkelgassers (BGs) in North America. Production facilities for 200,000 BGs per year will require a $25 mill...

See Answer

Q: Sweeney Pies has issued a zero-coupon 10-year bond

Sweeney Pies has issued a zero-coupon 10-year bond that can be converted into 10 Sweeney shares. Comparable straight bonds are yielding 8%. Sweeney stock is priced at $50 a share. a. Suppose that you...

See Answer

Q: Zenco Inc. is financed by 3 million shares of common stock

Zenco Inc. is financed by 3 million shares of common stock and by $5 million face value of 8% convertible debt maturing in 2029. Each bond has a face value of $1,000 and a conversion ratio of 200. Wha...

See Answer

Q: Match each of the following terms with one of the definitions below

Match each of the following terms with one of the definitions below: A. Revolving credit B. Bridge loan C. Term loan D. Syndicated loan E. Commitment fee F. Maintenance covenant a. Requirement that bo...

See Answer

Q: Some of the following reasons for leasing are rational. Others are

Some of the following reasons for leasing are rational. Others are irrational or assume imperfect or inefficient capital markets. Which of the following reasons are the rational ones? a. The lessee’s...

See Answer

Q: Lenders to leveraged leases hold nonrecourse debt. What does “nonrecourse

Lenders to leveraged leases hold nonrecourse debt. What does “nonrecourse” mean? What are the benefits and costs of nonrecourse debt to the equity investors in the lease?

See Answer

Q: True or false? a. Lease payments are usually made

True or false? a. Lease payments are usually made at the start of each period. Thus, the first payment is usually made as soon as the lease contract is signed. b. A sensible motive for financial lease...

See Answer

Q: Acme has branched out to rentals of office furniture to start-

Acme has branched out to rentals of office furniture to start-up companies. Consider a $3,000 desk. Desks last for six years and can be depreciated immediately. What is the break-even operating lease...

See Answer

Q: Look again at Problem 7. Suppose a blue-chip company

Look again at Problem 7. Suppose a blue-chip company requests a six-year financial lease for a $3,000 desk. The company has just issued five-year notes at an interest rate of 6% per year. What is the...

See Answer

Q: Suppose that National Waferonics has before it a proposal for a four

Suppose that National Waferonics has before it a proposal for a four-year financial lease of a Waferooney machine. The firm constructs a table like Table 25.2. The bottom line of its table shows the l...

See Answer

Q: Look again at the National Waferonics lease in Problem 11. Suppose

Look again at the National Waferonics lease in Problem 11. Suppose that National Waferonics is highly levered and is unable to deduct further interest payments for tax. a. Does this make a lease more...

See Answer

Q: Demand for concave utility meters is expanding rapidly, but the industry

Demand for concave utility meters is expanding rapidly, but the industry is highly competitive. A utility meter plant costs $50 million to set up, and it has an annual capacity of 500,000 meters. The...

See Answer

Q: Look again at the bus lease described in Table 25.2

Look again at the bus lease described in Table 25.2. a. What is the value of the lease if Greymare’s marginal tax rate is Tc = .30? b. What would the lease value be if the tax rate is 21%, but for tax...

See Answer

Q: In Section 25-5, we showed that the lease offered

In Section 25-5, we showed that the lease offered to Greymare Bus Lines had a positive NPV of $660 if Greymare paid no tax and a +$4,930 NPV to a lessor paying 21% tax. What is the minimum lease payme...

See Answer

Q: The Safety Razor Company has a large tax-loss carry-

The Safety Razor Company has a large tax-loss carry-forward and does not expect to pay taxes for another 10 years. The company is therefore proposing to lease $100,000 of new machinery. The lease term...

See Answer

Q: Yesterday, you sold six-month futures on the German DAX

Yesterday, you sold six-month futures on the German DAX stock market index at a price of 13,200. Today, the DAX closed at 13,150 and DAX futures closed at 13,250. You get a call from your broker, who...

See Answer

Q: Calculate the value of a six-month futures contract on a

Calculate the value of a six-month futures contract on a Treasury bond. You have the following information: 1. Six-month interest rate: 10% per year, or 4.9% for six months. 2. Spot price of bond: 95....

See Answer

Q: In December 2017, six-month futures on the Australian S

In December 2017, six-month futures on the Australian S&P/ASX 200 Index traded at 5,947. Spot was 6,001. The interest rate was 1.8% a year, and the dividend yield was about 4.4% a year. Were the futur...

See Answer

Q: Calculate convenience yield for magnoosium scrap from the following information:

Calculate convenience yield for magnoosium scrap from the following information: 1. Spot price: $2,550 per ton. 2. Futures price: $2,408 for a one-year contract. 3. Interest rate: 12%. 4. Storage cost...

See Answer

Q: In March 2018, six-month bitcoin futures were priced at

In March 2018, six-month bitcoin futures were priced at $7,925. The spot price was $7,946. The six-month interest rate was 1.92%. a. What was the convenience yield? b. Is your answer to part (a) consi...

See Answer

Q: In September 2020, swap dealers were quoting a rate for five

In September 2020, swap dealers were quoting a rate for five-year euro interest-rate swaps of 4.5% against Euribor (the short-term interest rate for euro loans). Euribor at the time was 4.1%. Suppose...

See Answer

Q: Look at Table 27.1. a. How many

Look at Table 27.1. a. How many Japanese yen do you get for your dollar? b. What is the three-month forward rate for yen? c. Is the yen at a forward discount or premium on the dollar? d. Use the one-y...

See Answer

Q: New-model commercial airplanes are much more fuel-efficient than

New-model commercial airplanes are much more fuel-efficient than older models. How is it possible for airlines flying older models to make money when its competitors are flying newer planes? Explain b...

See Answer

Q: In March 2017, the exchange rate for the Narnianleo was L2

In March 2017, the exchange rate for the Narnianleo was L2,419 = $1. Inflation in the year to March 2018 was about 30% in Narnia and 2% in the United States. a. If purchasing power parity held, what...

See Answer

Q: In December 2017, an American investor buys 1,000 shares

In December 2017, an American investor buys 1,000 shares in a Mexican company at a price of 500 pesos each. The share does not pay any dividend. A year later she sells the shares for 550 pesos each. T...

See Answer

Q: Keller Cosmetics maintains an operating profit margin of 8% and a

Keller Cosmetics maintains an operating profit margin of 8% and a sales-to-assets ratio of 3. It has assets of $500,000 and equity of $300,000. Assume that interest payments are $30,000 and the tax ra...

See Answer

Q: Table 28.8 gives abbreviated balance sheets and income statements for

Table 28.8 gives abbreviated balance sheets and income statements for Walmart. At the end of fiscal 2017, Walmart had 2,960 million shares outstanding with a share price of $106. The companyâ...

See Answer

Q: Look again at Table 28.8, which gives abbreviated balance

Look again at Table 28.8, which gives abbreviated balance sheets and income statements for Walmart. Assume Walmart had a 35% marginal corporate tax rate in 2017. Calculate the following using balance-...

See Answer

Q: Sara Togas sells all its output to Federal Stores. The following

Sara Togas sells all its output to Federal Stores. The following table shows selected 2017 financial data, in millions, for the two firms: The company’s tax rate is 35%. Calculate th...

See Answer

Q: a. If a firm’s assets of $10,000 represent

a. If a firm’s assets of $10,000 represent 200 days’ sales, what is its annual sales? b. What is its asset turnover ratio?

See Answer

Q: Look up some firms that have been in trouble. Plot the

Look up some firms that have been in trouble. Plot the changes over the preceding years in the principal financial ratios. Are there any patterns?

See Answer

Q: State whether each of the following events is a source or use

State whether each of the following events is a source or use of cash, or neither. a. An automobile manufacturer increases production in response to a forecasted increase in demand. Unfortunately, the...

See Answer

Q: a. What is the internal growth rate of Eagle Sport (

a. What is the internal growth rate of Eagle Sport (see Problem 25) if the dividend payout ratio is fixed at 50% and the equity-to-asset ratio is fixed at two-thirds? b. What is the sustainable growth...

See Answer

Q: Here are alphas and betas for Estée Lauder and Caterpillar Tractor for

Here are alphas and betas for Estée Lauder and Caterpillar Tractor for the 60 months ending June 2017. Alpha is expressed as a percent per month. Explain how these estimates would be used...

See Answer

Q: Table 29.18 shows the 2019 financial statements for the Executive

Table 29.18 shows the 2019 financial statements for the Executive Cheese Company. Annual depreciation is 10% of fixed assets at the beginning of the year, plus 10% of new investments. The company plan...

See Answer

Q: Table 29.11 shows Dynamic Mattress’s year-end 2016 balance

Table 29.11 shows Dynamic Mattress’s year-end 2016 balance sheet, and Table 29.12 shows its income statement for 2017. Work out the statement of cash flows for 2017. Group these item...

See Answer

Q: In fiscal 2017 and 2016, Estée Lauder’s financial statements included the

In fiscal 2017 and 2016, Estée Lauder’s financial statements included the following items. What was its cash cycle?

See Answer

Q: a. Paymore places orders for goods equal to 75% of

a. Paymore places orders for goods equal to 75% of its sales forecast for the next quarter. What will orders be in each quarter of the coming year if the sales in the current quarter are expected to b...

See Answer

Q: Each of the following events affects one or more tables in Sections

Each of the following events affects one or more tables in Sections 29-2 and 29-3. Show the effects of each event by adjusting the tables listed in parentheses: a. Dynamic repays only $10 million of s...

See Answer

Q: Construct a new model for Dynamic Mattress based on your answer to

Construct a new model for Dynamic Mattress based on your answer to Problem 19. Does your model generate a feasible financial plan for 2019? (Hint: If it doesn’t, you may have to allow the firm to issu...

See Answer

Q: Table 29.15 summarizes the 2019 income statement and end year

Table 29.15 summarizes the 2019 income statement and end year balance sheet of Drake’s Bowling Alleys. Drake’s financial manager forecasts a 10% increase in sales and costs in 2020. The ratio of sales...

See Answer

Q: Abbreviated financial statements for Archimedes Levers are shown in Table 29.

Abbreviated financial statements for Archimedes Levers are shown in Table 29.16. If sales increase by 10% in 2018 and all other items, including debt, increase correspondingly, what must be the balanc...

See Answer

Q: The financial statements of Eagle Sport Supply are shown in Table 29

The financial statements of Eagle Sport Supply are shown in Table 29.17. For simplicity, “Costs” include interest. Assume that Eagle’s assets are proportional to its sales. a. Find Eagle’s required ex...

See Answer

Q: Take a look at Figure 30.1. Why do food

Take a look at Figure 30.1. Why do food stores hold large inventories? Why do railroads hold small inventories? Why do you think that pharmaceutical companies hold so much cash and securities? Answer...

See Answer

Q: Explain how incentive and agency problems might contribute to mispricing of securities

Explain how incentive and agency problems might contribute to mispricing of securities or to bubbles. Give examples.

See Answer

Q: Look up current interest rates offered by short-term investment alternatives

Look up current interest rates offered by short-term investment alternatives. Suppose that your firm has $1 million excess cash to invest for the next two months. How would you invest this cash? How w...

See Answer

Q: What is a territorial corporate income tax system? How does it

What is a territorial corporate income tax system? How does it differ from the U.S. tax system that was in place before 2018? Explain why the U.S. system in 2017 and earlier forced U.S companies to le...

See Answer

Q: Uppose you are a wealthy individual paying 37% tax on interest

Uppose you are a wealthy individual paying 37% tax on interest income, 20% on dividends, and zero tax on municipal notes. What is the expected after-tax yield on each of the following investments? a....

See Answer

Q: Look again at Problem 7, which asked you to assume that

Look again at Problem 7, which asked you to assume that the 6.5% interest rate was the opportunity cost of capital. Was that a reasonable assumption? What should the opportunity cost of capital for in...

See Answer

Q: True or false? a. Companies with negative net working

True or false? a. Companies with negative net working capital are usually in financial trouble. b. Principal payments on long-term debt are shown as current liabilities if due within the next 12 month...

See Answer

Q: Central banks pushed short-term interest rates down to extremely low

Central banks pushed short-term interest rates down to extremely low levels in the financial crisis that started in 2008. Some Treasury bill rates in Europe were negative. Other things equal, how woul...

See Answer

Q: Take another look at Example 30.1. Suppose a rise

Take another look at Example 30.1. Suppose a rise in interest rates increases carrying cost per ton from $55 to $75. What is the effect on economic order quantity?

See Answer

Q: Polar Express Railroad keeps a $5 million inventory of spare parts

Polar Express Railroad keeps a $5 million inventory of spare parts on hand for repairing unexpected breakdowns and equipment failures. The inventory is held in one centralized warehouse at a storage c...

See Answer

Q: True or false? a. “Money market” refers

True or false? a. “Money market” refers to the system of electronic cash transfers between corporations and within the banking industry. b. The eurodollar market is a market for exchanging dollars for...

See Answer

Q: Anne Teak, the financial manager of a furniture manufacturer, is

Anne Teak, the financial manager of a furniture manufacturer, is considering operating a lockbox system. She forecasts that 300 payments a day will be made to lockboxes with an average payment size of...

See Answer

Q: True or false? a. Most managers tend to be

True or false? a. Most managers tend to be overconfident. b. Psychologists have found that, once people have suffered a loss, they are more relaxed about the possibility of incurring further losses. c...

See Answer

Q: The financial manager of JAC Cosmetics is considering opening a lockbox in

The financial manager of JAC Cosmetics is considering opening a lockbox in Pittsburgh. Checks cleared through the lockbox will amount to $10,000 per day. The lockbox will make cash available to the co...

See Answer

Q: In Section 30-5, we described a three-month

In Section 30-5, we described a three-month bill that was issued on an annually compounded yield of 5%. Suppose that one month has passed and the investment still offers the same annually compounded r...

See Answer

Q: True or false? a. Under purchase accounting any difference

True or false? a. Under purchase accounting any difference between the amounts paid for the target’s assets and their book value is shown as goodwill in the merged company’s balance sheet. b. In a tax...

See Answer

Q: Examine a hostile acquisition and discuss the tactics employed by both the

Examine a hostile acquisition and discuss the tactics employed by both the predator and the target companies. Do you think that the management of the target firm was trying to defeat the bid or to sec...

See Answer

Q: How do you think mergers should be regulated? For example,

How do you think mergers should be regulated? For example, what defenses should target companies be allowed to employ? Should managers of target firms be compelled to seek out the highest bids? Should...

See Answer

Q: Examine several recent mergers and suggest the principal motives for merging in

Examine several recent mergers and suggest the principal motives for merging in each case.

See Answer

Q: As financial manager of Corton Inc., you are investigating a possible

As financial manager of Corton Inc., you are investigating a possible acquisition of Denham. You have the basic data given in the following table. You estimate that investors expect a steady growth of...

See Answer

Q: Gobi Desserts is bidding to take over Universal Puddings. Gobi has

Gobi Desserts is bidding to take over Universal Puddings. Gobi has 3,000 shares outstanding, selling at $50 per share. Universal has 2,000 shares outstanding, selling at $17.50 a share. Gobi estimates...

See Answer

Q: Winterbourne is considering a takeover of Monkton Inc. Winterbourne has 10

Winterbourne is considering a takeover of Monkton Inc. Winterbourne has 10 million shares outstanding, which sell for $40 each. Monkton has 5 million shares outstanding, which sell for $20 each. If th...

See Answer

Q: If Winterbourne from Problem 12 has a price-earnings ratio of

If Winterbourne from Problem 12 has a price-earnings ratio of 12 and Monkton has a P/E ratio of 8, what should be the P/E ratio of the merged firm? Assume in this case that the merger is financed by a...

See Answer

Q: In recent years, several large banks have paid management bonuses partly

In recent years, several large banks have paid management bonuses partly in bonds and partly in stock. What do you think is the reason for this? Do you think it is a good idea?

See Answer

Q: Many commentators have blamed the subprime crisis on “irrational exuberance.”

Many commentators have blamed the subprime crisis on “irrational exuberance.” What is your view? Explain briefly.

See Answer

Q: Examine a recent merger in which at least part of the payment

Examine a recent merger in which at least part of the payment made to the seller was in the form of stock. Use stock market prices to obtain an estimate of the gain from the merger and the cost of the...

See Answer

Q: Connect each term to its correct definition or description. a

Connect each term to its correct definition or description. a. poison pill b. tender offer c. shark repellent d. merger e. tax inversion f. proxy contest A. Changes in the corporate charter that are d...

See Answer

Q: In Section 31-5, we described how Valeant and its

In Section 31-5, we described how Valeant and its ally, Pershing Square, lost the battle to acquire Allergan. Sometimes, the losers in a takeover battle can also win if they own a toehold stake in the...

See Answer

Q: True or false? a. One of the first tasks

True or false? a. One of the first tasks of an LBO’s financial manager is to pay down debt. b. Once an LBO or MBO goes private, it almost always stays private. c. Many early MBOs were arranged for unw...

See Answer

Q: List the disadvantages of traditional U.S. conglomerates. Can

List the disadvantages of traditional U.S. conglomerates. Can private-equity firms overcome these disadvantages?

See Answer

Q: True or false? a. Carve-out or spin

True or false? a. Carve-out or spin-off of a division improves incentives for the division’s managers. b. The announcement of a spin-off is generally followed by a sharp fall in the stock price. c. Pr...

See Answer

Q: Examine some recent examples of divestitures. What do you think were

Examine some recent examples of divestitures. What do you think were the underlying reasons for them? How did investors react to the news?

See Answer

Q: True/ false? a. Direct holdings of equity by

True/ false? a. Direct holdings of equity by households are larger in Japan than in the United States. b. Bank financing of corporations is relatively important in Japan and continental Europe. c. Int...

See Answer

Q: What is a keiretsu? Give a brief description. What are

What is a keiretsu? Give a brief description. What are some of their advantages and disadvantages?

See Answer

Q: Generally in most Anglo-Saxon economies, managers have an obligation

Generally in most Anglo-Saxon economies, managers have an obligation to act in the interests of shareholders. In other countries, they may have wider obligations to society as a whole. What are the pr...

See Answer

Q: “The strong form of the efficient-market hypothesis is nonsense

“The strong form of the efficient-market hypothesis is nonsense. Look at mutual fund X; it has had superior performance for each of the last 10 years.” Does the speaker have a point? Suppose that ther...

See Answer

Q: True/false? a. In China, networks of

True/false? a. In China, networks of companies are known as keiretsus. b. Dual-class equity prevents any one group of shareholders from maintaining control. c. Large firms in Germany have two boards o...

See Answer

Q: It is suggested that family control allows the firm to focus on

It is suggested that family control allows the firm to focus on the long-term interests of the firm. What do you think are the offsetting disadvantages of family ownership?

See Answer

Q: Do large block holders increase company value? Does your answer depend

Do large block holders increase company value? Does your answer depend on the structure of the financial system?

See Answer

Q: Old Time Savings Bank pays 4% interest on its savings account

Old Time Savings Bank pays 4% interest on its savings account. If you deposit $1,000 in the bank and leave it there: a. How much interest will you earn in the first year? b. How much interest will you...

See Answer

Q: a. The cost of a new automobile is $10,

a. The cost of a new automobile is $10,000. If the interest rate is 5%, how much would you have to set aside now to provide this sum in five years? b. You have to pay $12,000 a year in school fees at...

See Answer

Q: You are contemplating membership in the St. Swithin’s and Ancient Golf

You are contemplating membership in the St. Swithin’s and Ancient Golf Club. The annual membership fee for the coming year is $5,000, but you can make a single payment today of $12,750, which will pro...

See Answer

Q: In 2017, Leonardo da Vinci’s painting Salvator Mundi sold for a

In 2017, Leonardo da Vinci’s painting Salvator Mundi sold for a record $450.3 million. In 1958, it sold for $125, equivalent in purchasing power to about $1,060 at 2017 prices. The painting was origin...

See Answer

Q: Compute the future value of a $100 investment for the following

Compute the future value of a $100 investment for the following combinations of rates and times. a. r = 6%, t = 10 years b. r = 6%, t = 20 years c. r = 4%, t = 10 years d. r = 4%, t = 20 years

See Answer

Q: In the five years preceding the end of 2016, the price

In the five years preceding the end of 2016, the price of Amazon shares rose by 34% a year. If you had invested $100 in Amazon at the beginning of this period, how much would you have by the end of th...

See Answer

Q: a. If the present value of $139 is $125

a. If the present value of $139 is $125, what is the discount factor? b. If that $139 is received in year 5, what is the interest rate?

See Answer

Q: Some extreme bubbles are obvious with hindsight, after they burst.

Some extreme bubbles are obvious with hindsight, after they burst. But how would you define a bubble? There are many examples of good news and rising stock prices, followed by bad news and falling sto...

See Answer

Q: Lofting Snodbury is considering investing in a new boring machine. It

Lofting Snodbury is considering investing in a new boring machine. It costs $380,000 and is expected to produce the following cash flows: If the cost of capital is 12%, what is the machineâ€...

See Answer

Q: The $40 million lottery prize that you have just won actually

The $40 million lottery prize that you have just won actually pays out $2 million a year for 20 years. The interest rate is 8%. a. If the first payment comes after 1 year, what is the present value of...

See Answer

Q: Annuities due A store offer two payment plans. Under the installment

Annuities due A store offer two payment plans. Under the installment plan, you pay 25% down and 25% of the purchase price in each of the next 3 years. If you pay the entire bill immediately, you can t...

See Answer

Q: A bank loan requires you to pay $70,000 at

A bank loan requires you to pay $70,000 at the end of each of the next eight years. The interest rate is 8%. a. What is the present value of these payments? b. Calculate for each year the loan balance...

See Answer

Q: A 10-year bond is issued with a face value of

A 10-year bond is issued with a face value of $1,000, paying interest of $60 a year. If interest rates increase shortly after the bond is issued, what happens to the bonds: a. Coupon rate? b. Price? c...

See Answer

Q: The following statements are true. Explain why. a.

The following statements are true. Explain why. a. If a bond’s coupon rate is higher than its yield to maturity, then the bond will sell for more than face value. b. If a bond’s coupon rate is lower t...

See Answer

Q: Bond prices and yields In November 2017, Treasury 4¾s

Bond prices and yields In November 2017, Treasury 4¾s of 2041 offered a semiannually compounded yield to maturity of 2.6%. Recognizing that coupons are paid semiannually, calculate the bond’s price.

See Answer

Q: Choose 10 U.S. Treasury bonds with different coupons and

Choose 10 U.S. Treasury bonds with different coupons and different maturities. Calculate how their prices would change if their yields to maturity increased by 1 percentage point. Are long- or short-t...

See Answer

Q: Look again at the bonds in part (b) of Problem

Look again at the bonds in part (b) of Problem 24. a. Explain intuitively why the yield to maturity on the 10% bond is less than that on the 5% bond. b. What should be the yield to maturity on a five-...

See Answer

Q: The following table shows the prices of a sample of Narnian Treasury

The following table shows the prices of a sample of Narnian Treasury strips in December 2018. Each strip makes a single payment of $1,000 at maturity. a. Calculate the annually compounded, spot intere...

See Answer

Q: True or false? The efficient-market hypothesis assumes that:

True or false? The efficient-market hypothesis assumes that: a. There are no taxes. b. There is perfect foresight. c. Successive price changes are independent. d. Investors are irrational. e. There ar...

See Answer

Q: A bond’s credit rating provides a guide to its price. In

A bond’s credit rating provides a guide to its price. In the fall of 2017 Aaa bonds yielded 3.6% and Baa bonds yielded 4.3%. If some bad news causes a 10% five-year bond to be unexpectedly downrated f...

See Answer

Q: Write a spreadsheet program to construct a series of bond tables that

Write a spreadsheet program to construct a series of bond tables that show the present value of a bond given the coupon rate, maturity, and yield to maturity. Assume that coupon payments are semiannua...

See Answer

Q: True or false? a. The bid price is always

True or false? a. The bid price is always greater than the ask price. b. An investor who wants to sell his stock immediately should enter a limit order. c. The sale of shares by a large investor usual...

See Answer

Q: Permian Partners (PP) produces from aging oil fields in west

Permian Partners (PP) produces from aging oil fields in west Texas. Production is 1.8 million barrels per year in 2018, but production is declining at 7% per year for the foreseeable future. Costs of...

See Answer

Q: Phoenix Corp. faltered in the recent recession but is recovering.

Phoenix Corp. faltered in the recent recession but is recovering. Free cash flow has grown rapidly. Forecasts made in 2019 are as follows: Phoenix’s recovery will be complete by 2024...

See Answer

Q: a. “I would like to sell 1000 shares of Walmart

a. “I would like to sell 1000 shares of Walmart at best.” b. “I would like to buy 500 shares of Hattersley at $50 or better.” Which of these is a limit order and which is a market order? If the price...

See Answer

Q: Here is a small part of the order book for Mesquite Foods

Here is a small part of the order book for Mesquite Foods: a. Georgina Sloberg submits a market order to sell 100 shares. What price will she receive? b. Norman Pilbarra submits a market order to buy...

See Answer

Q: Stock quotes Go to finance.yahoo.com and get trading

Stock quotes Go to finance.yahoo.com and get trading quotes for IBM. a. What is the latest IBM stock price and market cap? b. What is IBM’s dividend payment and dividend yield? c. What is IBM’s traili...

See Answer

Q: Look up P/E and P/B ratios for Entergy

Look up P/E and P/B ratios for Entergy (ticker symbol ETR), using Yahoo! Finance or another Internet source. Calculate the same ratios for the following potential comparables: American Electric Power...

See Answer

Q: True or false? a. All stocks in an equivalent

True or false? a. All stocks in an equivalent-risk class are priced to offer the same expected rate of return. b. The value of a share equals the PV of future dividends per share. c. The value of a sh...

See Answer

Q: Give two or three examples of research results or events that raise

Give two or three examples of research results or events that raise doubts about market efficiency. Briefly explain why.

See Answer

Q: Here are forecasts for next year for two stocks: /

Here are forecasts for next year for two stocks: a. What are the dividend payout ratios for each firm? b. What are the expected dividend growth rates for each stock? c. If investors require a return o...

See Answer

Q: Look up General Mills (GIS), Kellogg (K), Campbell

Look up General Mills (GIS), Kellogg (K), Campbell Soup (CPB), and Seneca Foods (SENEA). a. What are the current P/E and P/B ratios for these food companies? What are the dividend and dividend yield f...

See Answer

Q: Look up Intel (INTC), Oracle (ORCL), and HP

Look up Intel (INTC), Oracle (ORCL), and HP (HPQ) on finance.yahoo.com. Rank the companies’ forward P/E ratios from highest to lowest. What are the possible reasons for the different ratios? Which of...

See Answer

Q: Respond to the following comments: a. “I like

Respond to the following comments: a. “I like the IRR rule. I can use it to rank projects without having to specify a discount rate.” b. “I like the payback rule. As long as the minimum payback period...

See Answer

Q: Consider the following two mutually exclusive projects: / a

Consider the following two mutually exclusive projects: a. Calculate the NPV of each project for discount rates of 0%, 10%, and 20%. Plot these on a graph with NPV on the vertical axis and discount ra...

See Answer

Q: Plot the NPVs for the following projects for discount rates from 0

Plot the NPVs for the following projects for discount rates from 0% to 30%: a. Which one of these projects has no IRR? b. One of the projects has two IRRs. Which is this project and what are the IRRs?...

See Answer

Q: Consider the following two projects: / a. If

Consider the following two projects: a. If the opportunity cost of capital is 11%, which of these two projects would you accept (A, B, or both)? b. Suppose that you can choose only one of these two pr...

See Answer

Q: In the International Mulch and Compost example (Section 6-3

In the International Mulch and Compost example (Section 6-3), we assumed that early losses on the project could be used to offset taxable profits elsewhere in the corporation. Suppose that the losses...

See Answer

Q: Suppose that Sudbury Mechanical Drifters is proposing to invest $10 million

Suppose that Sudbury Mechanical Drifters is proposing to invest $10 million in a new factory. It can depreciate this investment straight-line over 10 years. The tax rate is 40%, and the discount rate...

See Answer

Q: In 2022, the California Air Resources Board (CARB) started

In 2022, the California Air Resources Board (CARB) started planning its “Phase 3” requirements for reformulated gasoline (RFG). RFG is gasoline blended to tight specifications designed to reduce pollu...

See Answer

Q: Here again are the five lessons of market efficiency. For each

Here again are the five lessons of market efficiency. For each lesson give an example showing the lesson’s relevance to financial managers. a. Markets have no memory. b. Trust market prices. c. Read t...

See Answer

Q: Deutsche Transport can lease a truck for four years at a cost

Deutsche Transport can lease a truck for four years at a cost of €30,000 annually. It can instead buy a truck at a cost of €80,000, with annual maintenance expenses of €10,000. The truck will be sold...

See Answer

Q: You can purchase an optical scanner today for $400. The

You can purchase an optical scanner today for $400. The scanner provides benefits worth $60 a year. The expected life of the scanner is 10 years. Scanners are expected to decrease in price by 20% per...

See Answer

Q: Economy-Cool air conditioners cost $300 to purchase, result

Economy-Cool air conditioners cost $300 to purchase, result in electricity bills of $150 per year, and last for five years. Luxury Air models cost $500, result in electricity bills of $100 per year, a...

See Answer

Q: Hayden Inc. has a number of copiers that were bought four

Hayden Inc. has a number of copiers that were bought four years ago for $20,000. Currently maintenance costs $2,000 a year, but the maintenance agreement expires at the end of two years, and thereafte...

See Answer

Q: You are operating an old machine that is expected to produce a

You are operating an old machine that is expected to produce a cash inflow of $5,000 in each of the next three years before it fails. You can replace it now with a new machine that costs $20,000 but i...

See Answer

Q: A forklift will last for only two more years. It costs

A forklift will last for only two more years. It costs $5,000 a year to maintain. For $20,000 you can buy a new forklift that can last for 10 years and should require maintenance costs of only $2,000...

See Answer

Q: Restate the net cash flows in Table 6.2 in real

Restate the net cash flows in Table 6.2 in real terms. Discount the restated cash flows at a real discount rate. Assume a 20% nominal rate and 10% expected inflation. NPV should be unchanged at +3,806...

See Answer

Q: Real and nominal flows Guandong Machinery are evaluating a new project to

Real and nominal flows Guandong Machinery are evaluating a new project to produce encapsulators. The initial investment in plant and equipment is RMB 500,000.15 Sales of encapsulators in year 1 are fo...

See Answer

Q: The following table tracks the main components of working capital over the

The following table tracks the main components of working capital over the life of a four-year project. Calculate net working capital and the cash inflows and outflows due to investment in working cap...

See Answer

Q: Better Mousetrap’s research laboratories have developed a new trap. The project

Better Mousetrap’s research laboratories have developed a new trap. The project requires an initial investment in plant and equipment of $6 million. This investment will be depreciat...

See Answer

Q: Two financial managers, Alpha and Beta, are contemplating a chart

Two financial managers, Alpha and Beta, are contemplating a chart showing the actual performance of the Standard and Poor’s Composite Index over a five-year period. Each manager’s company needs to iss...

See Answer

Q: Project NPV Marsha Jones has bought a used Mercedes horse transporter for

Project NPV Marsha Jones has bought a used Mercedes horse transporter for her Connecticut estate. It cost $35,000. The object is to save on horse transporter rentals. Marsha had been renting a transpo...

See Answer

Q: Project NPV Imperial Motors is considering producing its popular Rooster model in

Project NPV Imperial Motors is considering producing its popular Rooster model in China. This will involve an initial investment of RMB 4 billion. The plant will start production after one year. It is...

See Answer

Q: Project NPV and IRR A project requires an initial investment of $

Project NPV and IRR A project requires an initial investment of $100,000 and is expected to produce a cash inflow before tax of $26,000 per year for five years. Company A has substantial accumulated t...

See Answer

Q: Go to the Excel spreadsheet versions of Table 6.2 and

Go to the Excel spreadsheet versions of Table 6.2 and answer the following questions. a. New engineering estimates raise the possibility that capital investment will be more than $12 million, perhaps...

See Answer

Q: Rate of return The level of the Syldavia market index is 21

Rate of return The level of the Syldavia market index is 21,000 at the start of the year and 25,500 at the end. The dividend yield on the index is 4.2%. a. What is the return on the index over the yea...

See Answer

Q: Here are some historical data on the risk characteristics of Ford and

Here are some historical data on the risk characteristics of Ford and Harley Davidson: Assume the standard deviation of the return on the market was 9.5%. a. The correlation coefficient of Fordâ...

See Answer

Q: Real versus nominal returns The Costaguana stock market provided a rate of

Real versus nominal returns The Costaguana stock market provided a rate of return of 95%. The inflation rate in Costaguana during the year was 80%. In Ruritania the stock market return was 12%, but th...

See Answer

Q: Arithmetic average and compound returns* Integrated Potato Chips (IPC)

Arithmetic average and compound returns* Integrated Potato Chips (IPC) does not pay a dividend. Its current stock price is $150 and there is an equal probability that the return over the coming year w...

See Answer

Q: Here are inflation rates and U.S. stock market and

Here are inflation rates and U.S. stock market and Treasury bill returns between 1929 and 1933: a. What was the real return on the stock market in each year? b. What was the average real return? c. Wh...

See Answer

Q: Risk Premium Suppose that in year 2030, investors become much more

Risk Premium Suppose that in year 2030, investors become much more willing than before to bear risk. As a result, they require a return of 8% to invest in common stocks rather than the 10% that they h...

See Answer

Q: What does the efficient-market hypothesis have to say about these

What does the efficient-market hypothesis have to say about these two statements? a. “I notice that short-term interest rates are about 1% below long-term rates. We should borrow short-term.” b. “I no...

See Answer

Q: Here are the percentage returns on two stocks. a.

Here are the percentage returns on two stocks. a. Calculate the monthly variance and standard deviation of each stock. Which stock is the riskier if held on its own? b. Now calculate the variance and...

See Answer

Q: True or false? a. Investors prefer diversified companies because

True or false? a. Investors prefer diversified companies because they are less risky. b. If stocks were perfectly positively correlated, diversification would not reduce risk. c. Diversification over...

See Answer

Q: Portfolio risk Suppose that the standard deviation of returns from a typical

Portfolio risk Suppose that the standard deviation of returns from a typical share is about .40 (or 40%) a year. The correlation between the returns of each pair of shares is about .3. a. Calculate th...

See Answer

Q: What is the beta of each of the stocks shown in Table

What is the beta of each of the stocks shown in Table 7.9?

See Answer

Q: For each of the following pairs of investments, state which would

For each of the following pairs of investments, state which would always be preferred by a rational investor (assuming that these are the only investments available to the investor): a. Portfolio A, r...

See Answer

Q: Look back at Problem 9 in Chapter 7. The risk-

Look back at Problem 9 in Chapter 7. The risk-free interest rate in each of these years was as follows: a. Calculate the average return and standard deviation of returns for Ms. Saurosâ€...

See Answer

Q: Portfolio beta Refer to Table 7.5. a.

Portfolio beta Refer to Table 7.5. a. What is the beta of a portfolio that has 40% invested in ExxonMobil and 60% in Newmont? b. Would you invest in this portfolio if you had no superior information a...

See Answer

Q: True or false? Explain or qualify as necessary. a

True or false? Explain or qualify as necessary. a. Investors demand higher expected rates of return on stocks with more variable rates of return. b. The CAPM predicts that a security with a beta of 0...

See Answer

Q: True or false? a. The CAPM implies that if

True or false? a. The CAPM implies that if you could find an investment with a negative beta, its expected return would be less than the interest rate. b. The expected return on an investment with a b...

See Answer

Q: Suppose that the Treasury bill rate is 6% rather than 2

Suppose that the Treasury bill rate is 6% rather than 2%. Assume that the expected return on the market stays at 9%. Use the betas in Table 8.2. a. Calculate the expected return from Johnson & Johnson...

See Answer

Q: True or false? a. If markets are efficient,

True or false? a. If markets are efficient, shareholders should expect to receive only the risk-free interest rate on their investment. b. If markets are efficient, investment in the stock market is a...

See Answer

Q: The Treasury bill rate is 4%, and the expected return on

The Treasury bill rate is 4%, and the expected return on the market portfolio is 12%. Using the capital asset pricing model: a. Draw a graph similar to Figure 8.6 showing how the expected return varie...

See Answer

Q: Epsilon Corp. is evaluating an expansion of its business. The

Epsilon Corp. is evaluating an expansion of its business. The cash-flow forecasts for the project are as follows: The firm’s existing assets have a beta of 1.4. The risk-free interes...

See Answer

Q: APT Consider a three-factor APT model. The factors and

APT Consider a three-factor APT model. The factors and associated risk premiums are Calculate expected rates of return on the following stocks. The risk-free interest rate is 7%. a. A stock whose retu...

See Answer

Q: Some true or false questions about the APT: a.

Some true or false questions about the APT: a. The APT factors cannot reflect diversifiable risks. b. The market rate of return cannot be an APT factor. c. There is no theory that specifically identif...

See Answer

Q: Consider the following simplified APT model: / Calculate the

Consider the following simplified APT model: Calculate the expected return for the following stocks. Assume rf = 5%.

See Answer

Q: Figure 8.11 purports to show the range of attainable combinations

Figure 8.11 purports to show the range of attainable combinations of expected return and standard deviation. a. Which diagram is incorrectly drawn and why? b. Which is the efficient set of portfolios?...

See Answer

Q: APT Look again at Problem 19. Consider a portfolio with equal

APT Look again at Problem 19. Consider a portfolio with equal investments in stocks P, P2, and P3. a. What are the factor risk exposures for the portfolio? b. What is the portfolio’s expected return?...

See Answer

Q: The following table shows the sensitivity of four stocks to the three

The following table shows the sensitivity of four stocks to the three Fama–French factors. Estimate the expected return on each stock assuming that the interest rate is 2%, the expec...

See Answer

Q: Between 2008 and 2017, the returns on Microfund averaged 10%

Between 2008 and 2017, the returns on Microfund averaged 10% a year. In his 2017 discussion of performance, the fund president noted that this was 2.5% a year better than the return on the U.S. market...

See Answer

Q: In footnote 4, we noted that the minimum-risk portfolio

In footnote 4, we noted that the minimum-risk portfolio contained an investment of 53% in Amazon and 47% in Southwest Airlines. Prove it. (Hint: You need a little calculus to do so.)

See Answer

Q: Analysis of 60 monthly rates of return on United Futon common stock

Analysis of 60 monthly rates of return on United Futon common stock indicates a beta of 1.45 and an alpha of –.2% per month. A month later, the market is up by 5%, and United Futon is up by 6%. What i...

See Answer

Q: Look again at the set of the three efficient portfolios that we

Look again at the set of the three efficient portfolios that we calculated in Section 8-1. a. If the interest rate is 5%, which of the three efficient portfolios should you hold? b. How would your ans...

See Answer

Q: The following question illustrates the APT. Imagine that there are only

The following question illustrates the APT. Imagine that there are only two pervasive macroeconomic factors. Investments X, Y, and Z have the following sensitivities to these two factors: We assume th...

See Answer

Q: a. Plot the following risky portfolios on a graph:

a. Plot the following risky portfolios on a graph: b. Five of these portfolios are efficient, and three are not. Which are inefficient ones? c. Suppose you can also borrow and lend at an interest rate...

See Answer

Q: Look back at the calculation for Southwest Airlines and Amazon in Section

Look back at the calculation for Southwest Airlines and Amazon in Section 8-1. a. Recalculate the expected portfolio return and standard deviation for different values of x1 and x2, assuming the corre...

See Answer

Q: Mark Harrywitz proposes to invest in two shares, X and Y

Mark Harrywitz proposes to invest in two shares, X and Y. He expects a return of 12% from X and 8% from Y. The standard deviation of returns is 8% for X and 5% for Y. The correlation coefficient betwe...

See Answer

Q: Ebenezer Scrooge has invested 60% of his money in share A

Ebenezer Scrooge has invested 60% of his money in share A and the remainder in share B. He assesses their prospects as follows: a. What are the expected return and standard deviation of returns on his...

See Answer

Q: Here are returns and standard deviations for four investments. /

Here are returns and standard deviations for four investments. Calculate the standard deviations of the following portfolios. a. 50% in Treasury bills, 50% in stock P. b. 50% each in Q and R, assuming...

See Answer

Q: Percival Hygiene has $10 million invested in long-term corporate

Percival Hygiene has $10 million invested in long-term corporate bonds. This bond portfolio’s expected annual rate of return is 9%, and the annual standard deviation is 10%. Amanda Reckonwith, Perciva...

See Answer

Q: Use the long-term data on security returns in Sections 7

Use the long-term data on security returns in Sections 7-1 and 7-2 to calculate the historical level of the Sharpe ratio for the market portfolio.

See Answer

Q: Define the following terms: a. Cost of debt.

Define the following terms: a. Cost of debt. b. Cost of equity. c. After-tax WACC. d. Equity beta. e. Asset beta. f. Pure-play comparable. g. Certainty equivalent.

See Answer

Q: Most managers have no difficulty avoiding blatantly dishonest actions. But sometimes

Most managers have no difficulty avoiding blatantly dishonest actions. But sometimes there are gray areas, where it is debatable whether an action is unethical and unacceptable. Suggest an important e...

See Answer

Q: The second column in Table 13.1 shows the monthly return

The second column in Table 13.1 shows the monthly return on the British FTSE 100 index from January 2015 through July 2017. The remaining columns show returns on the stocks of two firmsâ€&#...

See Answer

Q: Figure 9.4 shows plots of monthly rates of return on

Figure 9.4 shows plots of monthly rates of return on three stocks versus those of the market index. The beta and standard deviation of each stock is given beside the plot. a. Which stock is safest for...

See Answer

Q: The following table shows estimates of the risk of two well-

The following table shows estimates of the risk of two well-known Canadian stocks: a. What proportion of each stock’s risk was market risk, and what proportion was specific risk? b....

See Answer

Q: Look again at Table 9.1. This time we will

Look again at Table 9.1. This time we will concentrate on Union Pacific. a. Calculate Union Pacific’s cost of equity from the CAPM using its own beta estimate and the industry beta estimate. How diffe...

See Answer

Q: Which of these projects is likely to have the higher asset beta

Which of these projects is likely to have the higher asset beta, other things equal? Why? a. The sales force for project A is paid a fixed annual salary. Project B’s sales force is paid by commissions...

See Answer

Q: EZCUBE Corp. is 50% financed with long-term bonds

EZCUBE Corp. is 50% financed with long-term bonds and 50% with common equity. The debt securities have a beta of .15. The company’s equity beta is 1.25. What is EZCUBE’s asset beta?

See Answer

Q: What types of firms need to estimate industry asset betas? How

What types of firms need to estimate industry asset betas? How would such a firm make the estimate? Describe the process step by step.

See Answer

Q: You run a perpetual encabulator machine, which generates revenues averaging $

You run a perpetual encabulator machine, which generates revenues averaging $20 million per year. Raw material costs are 50% of revenues. These costs are variable—they are always proportional to reven...

See Answer

Q: Many investment projects are exposed to diversifiable risks. What does “

Many investment projects are exposed to diversifiable risks. What does “diversifiable” mean in this context? How should diversifiable risks be accounted for in project valuation? Should they be ignore...

See Answer

Q: John Barleycorn estimates his firm’s after-tax WACC at only 8

John Barleycorn estimates his firm’s after-tax WACC at only 8%. Nevertheless, he sets a 15% companywide discount rate to offset the optimistic biases of project sponsors and to impose “discipline” on...

See Answer

Q: Fudge factors Mom and Pop Groceries has just dispatched a year’s supply

Fudge factors Mom and Pop Groceries has just dispatched a year’s supply of groceries to the government of the Central Antarctic Republic. Payment of $250,000 will be made one year hence after the ship...

See Answer

Q: True or false? a. Financing decisions are less easily

True or false? a. Financing decisions are less easily reversed than investment decisions. b. Tests have shown that there is almost perfect negative correlation between successive price changes. c. The...

See Answer

Q: True or false? a. The company cost of capital

True or false? a. The company cost of capital is the correct discount rate for all projects because the high risks of some projects are offset by the low risk of other projects. b. Distant cash flows...

See Answer

Q: An oil company is drilling a series of new wells on the

An oil company is drilling a series of new wells on the perimeter of a producing oil field. About 20% of the new wells will be dry holes. Even if a new well strikes oil, there is still uncertainty abo...

See Answer

Q: A project has a forecasted cash flow of $110 in year

A project has a forecasted cash flow of $110 in year 1 and $121 in year 2. The interest rate is 5%, the estimated risk premium on the market is 10%, and the project has a beta of .5. If you use a cons...

See Answer

Q: A project has the following forecasted cash flows: /

A project has the following forecasted cash flows: The estimated project beta is 1.5. The market return rm is 16%, and the risk-free rate rf is 7%. a. Estimate the opportunity cost of capital and the...

See Answer

Q: The McGregor Whisky Company is proposing to market diet scotch. The

The McGregor Whisky Company is proposing to market diet scotch. The product will first be test-marketed for two years in southern California at an initial cost of $500,000. This test launch is not exp...

See Answer

Q: Suppose that you are valuing a future stream of high-risk

Suppose that you are valuing a future stream of high-risk (high-beta) cash outflows. High risk means a high discount rate. But the higher the discount rate, the less the present value. This seems to s...

See Answer

Q: An oil company executive is considering investing $10 million in one

An oil company executive is considering investing $10 million in one or both of two wells: Well 1 is expected to produce oil worth $3 million a year for 10 years; well 2 is expected to produce $2 mill...

See Answer

Q: Company cost of capital Quark Productions (“Give your loved one a

Company cost of capital Quark Productions (“Give your loved one a quark today.”) uses its company cost of capital to evaluate all projects. Will it underestimate or overestimate the value of high-risk...

See Answer

Q: The total market value of the common stock of the Okefenokee Real

The total market value of the common stock of the Okefenokee Real Estate Company is $6 million, and the total value of its debt is $4 million. The treasurer estimates that the beta of the stock is cur...

See Answer

Q: You are given the following information for Golden Fleece Financial:

You are given the following information for Golden Fleece Financial: Calculate Golden Fleece’s company cost of capital. Ignore taxes.

See Answer

Q: Which (if any) of these statements are true? Stock

Which (if any) of these statements are true? Stock prices appear to behave as though successive values (a) Are random numbers. (b) Follow regular cycles. (c) Differ by a random number.

See Answer

Q: Company cost of capital Nero Violins has the following capital structure:

Company cost of capital Nero Violins has the following capital structure: a. What is the firm’s asset beta? (Hint: What is the beta of a portfolio of all the firm’s...

See Answer

Q: A company is 40% financed by risk-free debt.

A company is 40% financed by risk-free debt. The interest rate is 10%, the expected market risk premium is 8%, and the beta of the company’s common stock is .5. What is the after-tax WACC, assuming th...

See Answer

Q: Binomial Tree Farm’s financing includes $5 million of bank loans.

Binomial Tree Farm’s financing includes $5 million of bank loans. Its common equity is shown in Binomial’s Annual Report at $6.67 million. It has 500,000 shares of common stock outstanding, which trad...

See Answer

Q: Refer to the top-right panel of Figure 9.2

Refer to the top-right panel of Figure 9.2. What proportion of U.S. Steel’s returns was explained by market movements? What proportion of risk was diversifiable? How does the diversifiable risk show u...

See Answer

Q: Match each of the following terms to one of the definitions or

Match each of the following terms to one of the definitions or descriptions listed below: sensitivity analysis, scenario analysis, break-even analysis, operating leverage, Monte Carlo simulation, deci...

See Answer

Q: Break-even analysis Break-even calculations are most often concerned

Break-even analysis Break-even calculations are most often concerned with the effect of a Short fall in sales, but they could equally well focus on any other component of cash flow. Dog Days is consid...

See Answer

Q: Dime a Dozen Diamonds makes synthetic diamonds by treating carbon. Each

Dime a Dozen Diamonds makes synthetic diamonds by treating carbon. Each diamond can be sold for $100. The materials cost for a synthetic diamond is $40. The fixed costs incurred each year for factory...

See Answer

Q: Modern Artifacts can produce keepsakes that will be sold for $80

Modern Artifacts can produce keepsakes that will be sold for $80 each. Non depreciation fixed costs are $1,000 per year, and variable costs are $60 per unit. The initial investment of $3,000 will be d...

See Answer

Q: Break-even analysis Define the cash-flow break-even

Break-even analysis Define the cash-flow break-even point as the sales volume (in dollars) at which cash flow equals zero. a. Is the cash-flow break-even level of sales higher or lower than the zero-p...

See Answer

Q: Break-even analysis A financial analyst has computed both accounting and

Break-even analysis A financial analyst has computed both accounting and NPV breakeven sales levels for a project using straight-line depreciation over a six-year period. The project manager wants to...

See Answer

Q: Supply the missing words: “There are three forms of the

Supply the missing words: “There are three forms of the efficient-market hypothesis. Tests of randomness in stock returns provide evidence for the form of the hypothesis. Tests of stock price reaction...

See Answer

Q: Fixed and variable costs In a slow year, Deutsche Burgers will

Fixed and variable costs In a slow year, Deutsche Burgers will produce 2 million hamburgers at a total cost of $3.5 million. In a good year, it can produce 4 million hamburgers at a total cost of $4.5...

See Answer

Q: Operating leverage You estimate that your cattle farm will generate $1

Operating leverage You estimate that your cattle farm will generate $1 million of profits on sales of $4 million under normal economic conditions and that the degree of operating leverage is 8. a. Wha...

See Answer

Q: Operating leverage Look again at Modern Artifacts in Problem 12.

Operating leverage Look again at Modern Artifacts in Problem 12. a. What is the degree of operating leverage of Modern Artifacts when sales are $7,000? b. What is the degree of operating leverage when...

See Answer

Q: Operating leverage What is the lowest possible value for the degree of

Operating leverage What is the lowest possible value for the degree of operating leverage for a profitable firm? Show with a numerical example that if Modern Artifacts (see Problem 12) has zero fixed...

See Answer

Q: A project has fixed costs of $1,000 per year

A project has fixed costs of $1,000 per year, depreciation charges of $500 a year, annual revenue of $6,000, and variable costs equal to two-thirds of revenues. a. If sales increase by 10%, what will...

See Answer

Q: True or false? a. Sensitivity analysis is unnecessary for

True or false? a. Sensitivity analysis is unnecessary for projects with asset betas that are equal to zero. b. Sensitivity analysis can be used to identify the variables most crucial to a project’s su...

See Answer

Q: Monte Carlo simulation Suppose a manager has already estimated a project’s cash

Monte Carlo simulation Suppose a manager has already estimated a project’s cash flows, calculated its NPV, and done a sensitivity analysis like the one shown in Table 10.2. List the additional steps r...

See Answer

Q: Real options Explain why options to expand or contract production are most

Real options Explain why options to expand or contract production are most valuable when forecasts about future business conditions are most uncertain.

See Answer

Q: Real options describe the real option in each of the following cases

Real options describe the real option in each of the following cases: a. Moda di Milano postpones a major investment. The expansion has positive NPV on a discounted cash-flow basis, but top management...

See Answer

Q: Real options True or false? a. Decision trees can

Real options True or false? a. Decision trees can help identify and describe real options. b. The option to expand increases PV. c. High abandonment value decreases PV. d. If a project has positive NP...

See Answer

Q: How would you respond to the following comments? a.

How would you respond to the following comments? a. “Efficient market, my eye! I know lots of investors who do crazy things.” b. “Efficient market? Balderdash! I know at least a dozen people who have...

See Answer

Q: A silver mine can yield 10,000 ounces of silver at

A silver mine can yield 10,000 ounces of silver at a variable cost of $32 per ounce. The fixed costs of owning the mine are $40,000 per year regardless of whether the mine is open or closed. In half t...

See Answer

Q: Real options An auto plant that costs $100 million to build

Real options An auto plant that costs $100 million to build can produce a line of flex-fuel cars. The investment will produce cash flows with a present value of $140 million if the line is successful...

See Answer

Q: Decision trees Look back at the Vegetron electric mop project in Section

Decision trees Look back at the Vegetron electric mop project in Section 9-4. Assume that if tests fail and Vegetron continues to go ahead with the project, the $1 million investment would generate on...

See Answer

Q: Decision trees* Your midrange guess as to the amount of oil

Decision trees* Your midrange guess as to the amount of oil in a prospective field is 10 million barrels, but there is a 50% chance that the amount of oil is 15 million barrels and a 50% chance of 5 m...

See Answer

Q: Decision trees Look again at the decision tree in Figure 10.

Decision trees Look again at the decision tree in Figure 10.4. Expand the possible outcomes as follows: 1. Blockbuster: PV = $1.5 billion with 5% probability. 2. Above average: PV = $700 million with...

See Answer

Q: Decision trees look again at the example in Figure 10.4

Decision trees look again at the example in Figure 10.4. The R&D team has put forward a proposal to invest an extra $20 million in expanded phase II trials. The object is to prove that the drug can be...

See Answer

Q: Otobai’s staff (see Section 10-1) has come up

Otobai’s staff (see Section 10-1) has come up with the following revised estimates for the electric scooter project: Conduct a sensitivity analysis using the spreadsheets (available...

See Answer

Q: Project analysis New Energy is evaluating a new bio fuel facility.

Project analysis New Energy is evaluating a new bio fuel facility. The plant would cost $4,000 million to build and has the potential to produce up to 40 million barrels of synthetic oil a year. The p...

See Answer

Q: Monte Carlo simulation Look back at the guano project in Section 6

Monte Carlo simulation Look back at the guano project in Section 6-3. Use the Crystal Ball™ software to simulate how uncertainty about inflation could affect the project’s cash flows.

See Answer

Q: Decision trees Magna Charter is a new corporation formed by Agnes Magna

Decision trees Magna Charter is a new corporation formed by Agnes Magna to provide an executive flying service for the southeastern United States. The founder thinks there will be a ready demand from...

See Answer

Q: Respond to the following comments: a. “The random

Respond to the following comments: a. “The random-walk theory, with its implication that investing in stocks is like playing roulette, is a powerful indictment of our capital markets.” b. “If everyone...

See Answer

Q: The Rustic Welt Company is proposing to replace its old welt-

The Rustic Welt Company is proposing to replace its old welt-making machinery with more modern equipment. The new equipment costs $9 million (the existing equipment has zero salvage value). The attrac...

See Answer

Q: Use the spreadsheet for the guano project in Chapter 6 to undertake

Use the spreadsheet for the guano project in Chapter 6 to undertake a sensitivity analysis of the project. Make whatever assumptions seem reasonable to you. What are the critical variables? What shoul...

See Answer

Q: Emperor’s Clothes Fashions can invest $5 million in a new plant

Emperor’s Clothes Fashions can invest $5 million in a new plant for producing invisible makeup. The plant has an expected life of five years, and expected sales are 6 million jars of makeup a year. Fi...

See Answer

Q: A project currently generates sales of $10 million, variable costs

A project currently generates sales of $10 million, variable costs equal 50% of sales, and fixed costs are $2 million. The firm’s tax rate is 21%. What are the effects of the following changes on cash...

See Answer

Q: What is the NPV of the electric scooter project under the following

What is the NPV of the electric scooter project under the following scenario? Unit sales are 20% below expectations. Unit price is 10% below expectations. Unit variable cost remains at 50% of revenue....

See Answer

Q: You are considering a proposal to produce and market a new sluffing

You are considering a proposal to produce and market a new sluffing machine. The most likely outcomes for the project are as follows: Expected sales: 30,000 units per year Unit price: $50 Variable cos...

See Answer

Q: True or false? a. The approval of a firm’s

True or false? a. The approval of a firm’s capital budget allows managers to go ahead with any project included in the budget. b. Capital budgets and project authorizations are mostly developed “botto...

See Answer

Q: Explain how each of the following actions or problems can distort or

Explain how each of the following actions or problems can distort or disrupt the capital budgeting process. a. Overoptimism by project sponsors. b. Inconsistent forecasts of industry and macroeconomic...

See Answer

Q: Draw up an outline or flowchart tracing the capital budgeting process from

Draw up an outline or flowchart tracing the capital budgeting process from the initial idea for a new investment project to the completion of the project and the start of operations. Assume the idea f...

See Answer

Q: Explain why setting a higher discount rate is not a cure for

Explain why setting a higher discount rate is not a cure for upward-biased cash-flow forecasts.

See Answer

Q: “If the efficient-market hypothesis is true, the pension

“If the efficient-market hypothesis is true, the pension fund manager might as well select a portfolio with a pin.” Explain why this is not so.

See Answer

Q: Reliable Electric, a major Ruritanian producer of electrical products, is

Reliable Electric, a major Ruritanian producer of electrical products, is considering a proposal to manufacture a new type of industrial electric motor that would replace most of its existing product...

See Answer

Q: Fama and French show that average stock returns on firms with small

Fama and French show that average stock returns on firms with small market capitalizations have been significantly higher than average returns for “large-cap” firms. What are the possible explanations...

See Answer

Q: Which of the following observations appear to indicate market inefficiency? Explain

Which of the following observations appear to indicate market inefficiency? Explain whether the observation appears to contradict the weak, semistrong, or strong form of the efficient-market hypothesi...

See Answer

Q: Fill in the blanks, using the following terms: floating rate

Fill in the blanks, using the following terms: floating rate, common stock, convertible, subordinated, preferred stock, senior, warrant. a. If a lender ranks behind the firm’s general creditors in th...

See Answer

Q: The Finance in Practice box in Section 1-2 describes three

The Finance in Practice box in Section 1-2 describes three corporate practices that have been criticized as unethical. Select one of these and discuss at what point (if any) the practice slide into un...

See Answer

Q: True or false? a. Financing for public corporations must

True or false? a. Financing for public corporations must flow through financial markets. b. Financing for private corporations must flow through financial intermediaries. c. Almost all foreign exchang...

See Answer

Q: Which of the following are financial markets? a. NASDAQ

Which of the following are financial markets? a. NASDAQ. b. Vanguard Explorer Fund. c. JP Morgan Chase. d. Chicago Mercantile Exchange.

See Answer

Q: True or false? a. Exchange traded funds are hedge

True or false? a. Exchange traded funds are hedge funds that can be bought and sold on the stock exchange. b. Hedge funds provide small investors with low-cost diversification. c. The sale of insuranc...

See Answer

Q: Financial markets and intermediaries channel savings from investors to corporate investment.

Financial markets and intermediaries channel savings from investors to corporate investment. The savings make this journey by many different routes. Give a specific example for each of the following r...

See Answer

Q: Explain briefly how each of the following allow individuals or companies to

Explain briefly how each of the following allow individuals or companies to spread their risk: a. An exchange traded fund. b. Commodity markets. c. A life insurance company.

See Answer

Q: Some individuals are eager to spend income before it arrives; others

Some individuals are eager to spend income before it arrives; others want to postpone consumption. Give some examples of intermediaries that provide services to these individuals.

See Answer

Q: Construct a timeline of the important events in the financial crisis that

Construct a timeline of the important events in the financial crisis that started in the summer of 2007. When do you think the crisis ended? You will probably want to review some of the entries under...

See Answer

Q: We mention several causes of the financial crisis. What other causes

We mention several causes of the financial crisis. What other causes can you identify? You will probably want to review some of the entries under Further Reading before you answer.

See Answer

Q: True or false? a. In the United States,

True or false? a. In the United States, most common shares are owned by individual investors. b. An insurance company is a financial intermediary. c. Investments in partnerships cannot be publicly tra...

See Answer

Q: True or false? a. Net stock issues by U

True or false? a. Net stock issues by U.S. nonfinancial corporations in most years are small but positive. b. Most capital investment by U.S. companies is funded by retained earnings and reinvested de...

See Answer

Q: Look back to the cash flows for projects F and G in

Look back to the cash flows for projects F and G in Section 5-3. The cost of capital was assumed to be 10%. Assume that the forecasted cash flows for projects of this type are overstated by 8% on aver...

See Answer

Q: True or false? a. Banks are huge investors in

True or false? a. Banks are huge investors in corporate equity. b. Insurance companies are huge investors in corporate debt. c. Rather than investing directly in corporate equities, most households pr...

See Answer

Q: What do we mean when we say that stockholders have control rights

What do we mean when we say that stockholders have control rights and residual cash flow rights? How in practice do they exercise their control rights?

See Answer

Q: Saga City has a declassified board with nine directors. a

Saga City has a declassified board with nine directors. a. How many directors come up for election each year? b. Would Saga be more or less vulnerable to a hostile takeover if it had a classified boar...

See Answer

Q: Suppose that East Corporation has issued voting and nonvoting stock. Investors

Suppose that East Corporation has issued voting and nonvoting stock. Investors hope that holders of the voting stock will use their power to vote out the company’s incompetent management. Would you ex...

See Answer

Q: In 2018, Beta Corporation earned gross profits of $760,

In 2018, Beta Corporation earned gross profits of $760,000. a. Suppose that Beta was financed by a combination of common stock and $1 million of debt. The interest rate on the debt was 10%, and the co...

See Answer

Q: Which of the following features would increase the value of a corporate

Which of the following features would increase the value of a corporate bond? Which would reduce its value? a. The bond is convertible into shares. b. The bond is secured by a mortgage on real estate....

See Answer

Q: Each of the following terms is associated with one of the events

Each of the following terms is associated with one of the events beneath. Can you match them up? a. Best efforts b. Bookbuilding c. Shelf registration d. Rule 144A Events: A. Investors indicate to th...

See Answer

Q: Having heard about IPO underpricing, I put in an order to

Having heard about IPO underpricing, I put in an order to my broker for 1,000 shares of every IPO he can get for me. After three months, my investment record is as follows: a. What is the average unde...

See Answer

Q: Fishwick Enterprises has 200,000 shares outstanding, half of which

Fishwick Enterprises has 200,000 shares outstanding, half of which are owned by Jennifer Fishwick and half by her cousin. The two cousins have decided to sell 100,000 shares in an IPO. Half of these s...

See Answer

Q: Spike Equino is the CEO of a private medical equipment company that

Spike Equino is the CEO of a private medical equipment company that is proposing to sell 100,000 shares of its stock in an open auction. Suppose the company receives the bids in the following table. a...

See Answer

Q: Your brother-in-law wants you to join him in

Your brother-in-law wants you to join him in purchasing a building on the outskirts of town. You and he would then develop and run a Taco Palace restaurant. Both of you are extremely optimistic about...

See Answer

Q: Problem 23 contains details of a rights offering by Pandora Box.

Problem 23 contains details of a rights offering by Pandora Box. Suppose that the company had decided to issue new stock at €4. How many new shares would it have needed to sell to raise the same sum o...

See Answer

Q: True or false? a. Venture capitalists typically provide first

True or false? a. Venture capitalists typically provide first-stage financing sufficient to cover all development expenses. Second-stage financing is provided by stock issued in an IPO. b. Underpricin...

See Answer

Q: Ethelbert.com is a young software company owned by two entrepreneurs

Ethelbert.com is a young software company owned by two entrepreneurs. It currently needs to raise $400,000 to support its expansion plans. A venture capitalist is prepared to provide the cash in retu...

See Answer

Q: Look at Marvin’s first attempt to raise financing. (Refer to

Look at Marvin’s first attempt to raise financing. (Refer to Section 15.1 and the Marvin Prospectus in the appendix at the end of this chapter.) Suppose that First Meriam decides that Marvin’s shares...

See Answer

Q: True or false? a. Venture capital companies know that

True or false? a. Venture capital companies know that managers are more likely to work hard if they can be assured of a good steady salary. b. Venture capital companies generally advance the money in...

See Answer

Q: Complete the passage using the following terms: limited partners, venture

Complete the passage using the following terms: limited partners, venture capital, private, underwriters, general partners, private equity, corporate ventures, partnerships, private, angel investors....

See Answer

Q: Refer to Section 15.1 and the Marvin Prospectus Appendix at

Refer to Section 15.1 and the Marvin Prospectus Appendix at the end of this chapter to answer the following questions. a. If there is unexpectedly heavy demand for the issue, how many extra shares can...

See Answer

Q: Find the prospectus for a recent IPO. How do the issue

Find the prospectus for a recent IPO. How do the issue costs compare with, a. Those of the Marvin issue? b. Those shown in Table 15.3? Can you suggest reasons for the differences?

See Answer

Q: In April 2019, Van Dyck Exponents offered 100 shares for sale

In April 2019, Van Dyck Exponents offered 100 shares for sale in an IPO. Half of the shares were sold by the company and the other half by existing shareholders, each of whom sold exactly half of thei...

See Answer

Q: In 2017, Entergy paid a regular quarterly dividend of $.89

In 2017, Entergy paid a regular quarterly dividend of $.89 per share. a. Match each of the following dates. b. On one of these dates, the stock price fell by about $.89. Which date? Why? c. Entergy&ac...

See Answer

Q: Suppose that you are considering investing in an asset for which there

Suppose that you are considering investing in an asset for which there is a reasonably good secondary market. Specifically, your company is Delta Airlines, and the asset is a Boeing 757—a widely used...

See Answer

Q: Go to the Apple website or to a financial source such as

Go to the Apple website or to a financial source such as Yahoo! Finance. a. Has Apple’s dividend increased from the initial quarterly rate of $2.65? b. What was the announcement date of the most recen...

See Answer

Q: Go back to the first Rational Demiconductor balance sheet. Now assume

Go back to the first Rational Demiconductor balance sheet. Now assume that Rational wins a lawsuit and is paid $1 million in cash. Its market capitalization rises by that amount. It decides to pay out...

See Answer

Q: Here are key financial data for House of Herring Inc.:

Here are key financial data for House of Herring Inc.: House of Herring plans to pay the entire dividend early in January 2026. All corporate and personal taxes were repealed in 2024. a. Other things...

See Answer

Q: Reliable Gearing currently is all-equity-financed. It has

Reliable Gearing currently is all-equity-financed. It has 10,000 shares of equity outstanding, selling at $100 a share. The firm is considering a capital restructuring. The low-debt plan calls for a d...

See Answer

Q: Suppose that new security designs could be patented.14The patent holder

Suppose that new security designs could be patented.14The patent holder could restrict use of the new design or charge other firms royalties for using it. What effect would such patents have on MM’s c...

See Answer

Q: What is wrong with the following arguments? a. As

What is wrong with the following arguments? a. As the firm borrows more and debt becomes risky, both stock- and bondholders demand higher rates of return. Thus, by reducing the debt ratio, we can redu...

See Answer

Q: “MM totally ignore the fact that as you borrow more,

“MM totally ignore the fact that as you borrow more, you have to pay higher rates of interest.” Explain carefully whether this is a valid objection.

See Answer

Q: Look back to Section 17-1. Suppose that Ms.

Look back to Section 17-1. Suppose that Ms. Macbeth’s investment bankers have informed her that since the new issue of debt is risky, debtholders will demand a return of 12.5%, which is 2.5% above the...

See Answer

Q: Hubbard’s Pet Foods is financed 80% by common stock and 20

Hubbard’s Pet Foods is financed 80% by common stock and 20% by bonds. The expected return on the common stock is 12% and the rate of interest on the bonds is 6%. Assuming that the bonds are default-ri...

See Answer

Q: Omega Corporation has 10 million shares outstanding, now trading at $

Omega Corporation has 10 million shares outstanding, now trading at $55 per share. The firm has estimated the expected rate of return to shareholders at about 12%. It has also issued long-term bonds a...

See Answer

Q: There is an active, competitive leasing (i.e.,

There is an active, competitive leasing (i.e., rental) market for most standard types of commercial jets. Many of the planes flown by the major domestic and international airlines are not owned by the...

See Answer

Q: Gamma Airlines has an asset beta of 1.5. The

Gamma Airlines has an asset beta of 1.5. The risk-free interest rate is 6%, and the market risk premium is 8%. Assume the capital asset pricing model is correct. Gamma pays taxes at a marginal rate of...

See Answer

Q: Consider the following three tickets: Ticket A pays $10 if

Consider the following three tickets: Ticket A pays $10 if is elected as president, ticket B pays $10 if is elected, and ticket C pays $10 if neither is elected. (Fill in the blanks yourself.) Could t...

See Answer

Q: People often convey the idea behind MM’s proposition 1 by various supermarket

People often convey the idea behind MM’s proposition 1 by various supermarket analogies, for example, “The value of a pie should not depend on how it is sliced,” or, “The cost of a whole chicken shoul...

See Answer

Q: Look back at the Johnson & Johnson example in Section 18-

Look back at the Johnson & Johnson example in Section 18-1. Suppose Johnson & Johnson increases its long-term debt to $45 billion. It uses the additional debt to repurchase shares. Reconstruct Table 1...

See Answer

Q: What is the relative tax advantage of corporate debt if the corporate

What is the relative tax advantage of corporate debt if the corporate tax rate is Tc = .21, the personal tax rate is Tp = .37, but all equity income is received as capital gains and escapes tax entire...

See Answer

Q: On February 29, 2019, when PDQ Computers announced bankruptcy,

On February 29, 2019, when PDQ Computers announced bankruptcy, its share price fell from $3.00 to $.50 per share. There were 10 million shares outstanding. Does that imply bankruptcy costs of 10 × (3....

See Answer

Q: Look at some real companies with different types of assets. What

Look at some real companies with different types of assets. What operating problems would each encounter in the event of financial distress? How well would the assets keep their value?

See Answer

Q: This question tests your understanding of financial distress. a.

This question tests your understanding of financial distress. a. What are the costs of going bankrupt? Define these costs carefully. b. “A company can incur costs of financial distress without ever go...

See Answer

Q: True or false? a. If the probability of default

True or false? a. If the probability of default is high, managers and stockholders will be tempted to take on excessively risky projects. b. If the probability of default is high, stockholders may ref...

See Answer

Q: The traditional theory of optimal capital structure states that firms trade off

The traditional theory of optimal capital structure states that firms trade off corporate interest tax shields against the possible costs of financial distress due to borrowing. What does this theory...

See Answer

Q: Suppose the current price of gold is $1,200 an

Suppose the current price of gold is $1,200 an ounce. Hotshot Consultants advises you that gold prices will increase at an average rate of 12% for the next two years. After that the growth rate will...

See Answer

Q: True or false? a. Financial slack means having cash

True or false? a. Financial slack means having cash in the bank or ready access to the debt markets. b. Financial slack is most valuable to firms with few investment opportunities and poor prospects....

See Answer

Q: Calculate the weighted-average cost of capital (WACC) for

Calculate the weighted-average cost of capital (WACC) for Federated Junkyards of America, using the following information: 1) Debt: $75,000,000 book value outstanding. The debt is trading at 90% of b...

See Answer

Q: Take another look at the APV calculation for the perpetual crusher project

Take another look at the APV calculation for the perpetual crusher project in Section 19-4. This time assume that the corporation investing in the project has hit the 30% constraint on interest deduct...

See Answer

Q: Take another look at the valuations of Rio in Tables 19.

Take another look at the valuations of Rio in Tables 19.1 and 19.2. Now use the live spreadsheets in Connect to show how Rio’s value depends on: a. The forecasted long-term growth rate. b. The require...

See Answer

Q: Whispering Pines Inc. is all-equity-financed. The

Whispering Pines Inc. is all-equity-financed. The expected rate of return on the company’s shares is 12%. a. What is the opportunity cost of capital for an average-risk Whispering Pines investment? b....

See Answer

Q: Table 19.3 shows a book balance sheet for the Wishing

Table 19.3 shows a book balance sheet for the Wishing Well Motel chain. The company’s long-term debt is secured by its real estate assets, but it also uses short-term bank loans as a...

See Answer

Q: Table 19.4 shows a simplified balance sheet for the Dutch

Table 19.4 shows a simplified balance sheet for the Dutch manufacturer Rensselaer Felt. Calculate this company’s weighted-average cost of capital. The debt has just been refinanced a...

See Answer

Q: Nevada Hydro is 40% debt-financed and has a weighted

Nevada Hydro is 40% debt-financed and has a weighted-average cost of capital of 10.2%: WACC = (1 − Tc)rD D/V + rE E/V = (1 − .21)(.085)(.40) + .125(.60) = .102 Goldensacks Company is advising Nevada H...

See Answer

Q: Suppose Wishing Well (see Problem 6) is evaluating a new

Suppose Wishing Well (see Problem 6) is evaluating a new motel and resort on a romantic site in Madison County, Wisconsin. Explain how you would forecast the after-tax cash flows for this project. (Hi...

See Answer

Q: Consider a project lasting one year only. The initial outlay is

Consider a project lasting one year only. The initial outlay is $1,000, and the expected inflow is $1,200. The opportunity cost of capital is r = .20. The borrowing rate is rD = .10, and the tax shiel...

See Answer

Q: This is your second interview with a prestigious brokerage firm for a

This is your second interview with a prestigious brokerage firm for a job as an equity analyst. You survived the morning interviews with the department manager and the Vice President of Equity. Everyt...

See Answer

Q: You work in Walt Disney Company’s corporate finance and treasury department and

You work in Walt Disney Company’s corporate finance and treasury department and have just been assigned to the team estimating Disney’s WACC. You must estimate this WACC in preparation for a team meet...

See Answer

Q: See Table 2.5 showing financial statement data and stock price

See Table 2.5 showing financial statement data and stock price data for Mydeco Corp. a. Compute Mydeco’s PE ratio each year from 2015 to 2019. In which year was it the highest? b. Wh...

See Answer

Q: In early 2018, United Airlines (UAL) had a market

In early 2018, United Airlines (UAL) had a market capitalization of $20.2 billion, debt of $14.4 billion, and cash of $3.8 billion. United also had annual revenues of $37.7 billion. Southwest Airlines...

See Answer

Q: See Table 2.5 showing financial statement data and stock price

See Table 2.5 showing financial statement data and stock price data for Mydeco Corp. a. Compute Mydeco’s ROE each year from 2015 to 2019. b. Compute Mydeco’s ROA ea...

See Answer

Q: In Problem 20, assume the risk-free rate is 3

In Problem 20, assume the risk-free rate is 3% and the market risk premium is 7%. a. What does the CAPM predict the expected return for each stock should be? b. Clearly, the CAPM predictions are not e...

See Answer

Q: In Problem 20, assume the risk-free rate is 3

In Problem 20, assume the risk-free rate is 3% and the market risk premium is 7%. a. What does the CAPM predict the expected return for each stock should be? b. Clearly, the CAPM predictions are not e...

See Answer

Q: In Problem 20, assume the risk-free rate is 3

In Problem 20, assume the risk-free rate is 3% and the market risk premium is 7%. a. What does the CAPM predict the expected return for each stock should be? b. Clearly, the CAPM predictions are not e...

See Answer

Q: In early 2018 Qualcomm Inc. had $21 billion in debt

In early 2018 Qualcomm Inc. had $21 billion in debt, total equity capitalization of $78 billion, and an equity beta of 1.49 (as reported on Yahoo! Finance). Included in Qualcomm’s assets was $37 billi...

See Answer

Q: Find online the annual 10-K report for Costco Wholesale Corporation

Find online the annual 10-K report for Costco Wholesale Corporation (COST) for fiscal year 2017 (filed in October 2017). a. Which auditing firm certified these financial statements? b. Which officers...

See Answer

Q: Rumolt Motors has 30 million shares outstanding with a price of $

Rumolt Motors has 30 million shares outstanding with a price of $15 per share. In addition, Rumolt has issued bonds with a total current market value of $150 million. Suppose Rumolt’s equity cost of c...

See Answer

Q: Summit Builders has a market debt-equity ratio of 0.

Summit Builders has a market debt-equity ratio of 0.65 and a corporate tax rate of 25%, and it pays 7% interest on its debt. The interest tax shield from its debt lowers Summit’s WACC by what amount?...

See Answer

Q: You work in the corporate finance division of The Home Depot and

You work in the corporate finance division of The Home Depot and your boss has asked you to review the firm’s capital structure. Specifically, your boss is considering changing the firm’s debt level....

See Answer

Q: NatNah, a builder of acoustic accessories, has no debt and

NatNah, a builder of acoustic accessories, has no debt and an equity cost of capital of 15%. Suppose NatNah decides to increase its leverage and maintain a market debt-to-value ratio of 0.5. Suppose i...

See Answer

Q: Acme Storage has a market capitalization of $100 million and debt

Acme Storage has a market capitalization of $100 million and debt outstanding of $40 million. Acme plans to maintain this same debt-equity ratio in the future. The firm pays an interest rate of 7.5% o...

See Answer

Q: Milton Industries expects free cash flow of $5 million each year

Milton Industries expects free cash flow of $5 million each year. Milton’s corporate tax rate is 21%, and its unlevered cost of capital is 15%. The firm also has outstanding debt of $19.05 million, an...

See Answer

Q: Suppose Microsoft has 8.75 billion shares outstanding and pays a

Suppose Microsoft has 8.75 billion shares outstanding and pays a marginal corporate tax rate of 21%. If Microsoft announces that it will pay out $50 billion in cash to investors through a combination...

See Answer

Q: Kurz Manufacturing is currently an all-equity firm with 20 million

Kurz Manufacturing is currently an all-equity firm with 20 million shares outstanding and a stock price of $7.50 per share. Although investors currently expect Kurz to remain an all-equity firm, Kurz...

See Answer

Q: Rally, Inc., is an all-equity firm with assets

Rally, Inc., is an all-equity firm with assets worth $25 billion and 10 billion shares outstanding. Rally plans to borrow $10 billion and use these funds to repurchase shares. The firm’s corporate tax...

See Answer

Q: Suppose the corporate tax rate is 25%, and investors pay a

Suppose the corporate tax rate is 25%, and investors pay a tax rate of 20% on income from dividends or capital gains and a tax rate of 33% on interest income. Your firm decides to add debt so it will...

See Answer

Q: Markum Enterprises is considering permanently adding $100 million of debt to

Markum Enterprises is considering permanently adding $100 million of debt to its capital structure. Markum’s corporate tax rate is 25%. a. Absent personal taxes, what is the value of the interest tax...

See Answer

Q: With its current leverage, Impi Corporation will have net income next

With its current leverage, Impi Corporation will have net income next year of $4.5 million. If Impi’s corporate tax rate is 21% and it pays 8% interest on its debt, how much additional debt can Impi i...

See Answer

Q: Colt Systems will have EBIT this coming year of $15 million

Colt Systems will have EBIT this coming year of $15 million. It will also spend $6 million on total capital expenditures and increases in net working capital, and have $3 million in depreciation expen...

See Answer

Q: Your boss was impressed with your presentation regarding the irrelevance of capital

Your boss was impressed with your presentation regarding the irrelevance of capital structure but, as expected, has realized that market imperfections like taxes must be accounted for. You have now be...

See Answer

Q: Grommit Engineering expects to have net income next year of $20

Grommit Engineering expects to have net income next year of $20.75 million and free cash flow of $22.15 million. Grommit’s marginal corporate tax rate is 20%. a. If Grommit increases leverage so that...

See Answer

Q: Suppose the corporate tax rate is 25%. Consider a firm that

Suppose the corporate tax rate is 25%. Consider a firm that earns $1000 before interest and taxes each year with no risk. The firm’s capital expenditures equal its depreciation expenses each year, and...

See Answer

Q: Consider the following potential events that might have taken place at Global

Consider the following potential events that might have taken place at Global Conglomerate on December 30, 2018. For each one, indicate which line items in Global’s balance sheet would be affected and...

See Answer

Q: What was the change in Global Conglomerate’s book value of equity from

What was the change in Global Conglomerate’s book value of equity from 2017 to 2018 according to Table 2.1 ? Does this imply that the market price of Global’s share...

See Answer

Q: Use EDGAR to find Qualcomm’s 10-K filing for 2017.

Use EDGAR to find Qualcomm’s 10-K filing for 2017. From the balance sheet, answer the following questions: a. How much did Qualcomm have in cash, cash equivalents, and marketable securities (short-and...

See Answer

Q: In your role as a consultant at a wealth management firm,

In your role as a consultant at a wealth management firm, you have been assigned a very powerful client who holds one million shares of Cisco Systems, Inc. purchased on February 28, 2003. In researchi...

See Answer

Q: Toyota Motor Company is expanding the production of their gas-electric

Toyota Motor Company is expanding the production of their gas-electric hybrid drive systems and plans to shift production in the United States. To enable the expansion, they are contemplating investin...

See Answer

Q: Repeat the requirements in E11-13 assuming that Kurtis Koal Company

Repeat the requirements in E11-13 assuming that Kurtis Koal Company, Inc. acquired the asset on August 1 of the current year. Data from E11-13: Prepare the depreciation schedules for the machine assu...

See Answer

Q: Repeat the requirements in E11-14 assuming that Kurtis Koal Company

Repeat the requirements in E11-14 assuming that Kurtis Koal Company, Inc. acquired the asset on August 1 of the current year. Use partial-year depreciation without adopting any of the acceptable conve...

See Answer

Q: Ace Manufacturing, Inc. purchased a new piece of manufacturing equipment

Ace Manufacturing, Inc. purchased a new piece of manufacturing equipment at a total acquisition cost of $3,000,000 on January 4 of the current year. The firm estimates that the equipment has a useful...

See Answer

Q: Repeat the requirements in E11-17 assuming that Ace is an

Repeat the requirements in E11-17 assuming that Ace is an IFRS reporter and the manufacturing equipment has two components: computer controls and engine. The amount allocated to the computer controls...

See Answer

Q: Repeat the requirements in E11-17 assuming that Ace acquired the

Repeat the requirements in E11-17 assuming that Ace acquired the asset on July 14 of the current year. Use partial-year depreciation assuming that the manufacturing equipment was acquired at the begin...

See Answer

Q: You have been hired by a well-funded hedge fund to

You have been hired by a well-funded hedge fund to assess whether there are any arbitrage opportunities created by differences in prices in bitcoin across different markets. For the purposes of this e...

See Answer

Q: IFRS. Repeat the requirements in E11-18 assuming that Ace

IFRS. Repeat the requirements in E11-18 assuming that Ace acquired the asset on July 14 of the current year. Use partial-year depreciation assuming that the manufacturing equipment was acquired at the...

See Answer

Q: The Gemini Group sold one of its plant assets on August 1

The Gemini Group sold one of its plant assets on August 1 of the current year for $200,000. The asset had an original cost of $500,000 and an estimated residual value of $80,000. The firm used the str...

See Answer

Q: The Aries Group sold one of its plant assets on April 1

The Aries Group sold one of its plant assets on April 1 of the current year for $250,000. The asset had an original cost of $500,000 and an estimated residual value of $80,000. Aries used the straight...

See Answer

Q: Use the information in E11-13, part (a)

Use the information in E11-13, part (a) to prepare the required footnote disclosure for Kurtis Koal Company, Inc.’s property, plant, and equipment for Years 1 and 2, including a sta...

See Answer

Q: IFRS. Use the information in E11-14, part (

IFRS. Use the information in E11-14, part (a) to prepare the required footnote disclosure under IFRS for Kurtis Koal Company, Inc.’s property, plant, and equipment for Years 1 and 2, including a state...

See Answer

Q: On January 2, 2019, Bubba and Company paid $5

On January 2, 2019, Bubba and Company paid $5,000,000 in cash to acquire 100% of the Cire Company’s voting common stock. Cire’s balance sheet on that date showed th...

See Answer

Q: During the current year, Carlson Industries, Inc. conducted significant

During the current year, Carlson Industries, Inc. conducted significant research activities related to the development of a new computer chip. Carlson had the following costs Prepare all journal entri...

See Answer

Q: IFRS. Repeat the requirements in E11-26 assuming that Carlson

IFRS. Repeat the requirements in E11-26 assuming that Carlson reports under IFRS. Assume that all the conditions to capitalize development costs have been met and the project is completed on January 1...

See Answer

Q: Alto Devices acquires Medifast, a small start-up company,

Alto Devices acquires Medifast, a small start-up company, by paying $2,170,000 in cash on January 2. Following are the book values and fair values of Medifast on the date of acquisition. Required: a...

See Answer

Q: On January 2, 2018, Temptations Corporation paid $31,

On January 2, 2018, Temptations Corporation paid $31,500 in cash and exchanged a chocolate mixing machine, which had a fair value of $437,500 and a book value of $500,000 ($1,135,000 historical cost −...

See Answer

Q: Assume that Rascals Candy, Inc. reports under IFRS. Repeat

Assume that Rascals Candy, Inc. reports under IFRS. Repeat the requirement of E11-29, part (c). Data from E11-29: a. Record the journal entry on the books of Temptations Corporation to record the exch...

See Answer

Q: Assume that Sebastian Company from E11-38 reports under IFRS.

Assume that Sebastian Company from E11-38 reports under IFRS. Prepare the journal entry to record the exchange on the books of Sebastian Company. Data from E11-38: Brown Company contracts with Sebast...

See Answer

Q: You are an intern with First Bank, and have been asked

You are an intern with First Bank, and have been asked to develop estimates for yield spreads that would be appropriate for corporate bonds or loans with different maturities and credit ratings. To ge...

See Answer

Q: Ferro Fuel Company (FFC) acquired a tract of land to

Ferro Fuel Company (FFC) acquired a tract of land to be used for oil and gas exploration at the beginning of the current year. FFC paid $500,000 to acquire the land, paid $325,000 in development costs...

See Answer

Q: Repeat the requirements in E11-40 assuming that FFC uses the

Repeat the requirements in E11-40 assuming that FFC uses the successful-efforts method. Data from E11-40: a. Determine the total cost of the natural resource under the full-cost method. b. Prepare t...

See Answer

Q: Spill Oil Corporation drilled 10 oil wells at the beginning of the

Spill Oil Corporation drilled 10 oil wells at the beginning of the current year. The total exploration costs associated with this oil and gas activity amounted to $8,500,000. Only six of the wells wer...

See Answer

Q: Rolling Blackout Power Company constructed a new power plant to supply energy

Rolling Blackout Power Company constructed a new power plant to supply energy to the Northeast Electrical Grid. The construction began on January 2 and ended on December 31 of the current year. On the...

See Answer

Q: How would the solution to E11-6 change if Rolling Blackout

How would the solution to E11-6 change if Rolling Blackout Power Company was an IFRS reporter and earned $11,000 interest income on investing the excess funds from the construction loan during the yea...

See Answer

Q: Yawyag Corporation engaged Sir Peter, Inc. to design and construct

Yawyag Corporation engaged Sir Peter, Inc. to design and construct a manufacturing facility. Construction began on January 2 and was completed on December 31 of the current year. The following payment...

See Answer

Q: Assume that the project in E11-8 was not completed in

Assume that the project in E11-8 was not completed in Year 1. Yawyag was required to make two additional payments to the contractor in Year 2: $900,000 on April 1 and $1,800,000 on August 1 of Year 2....

See Answer

Q: IFRS. Assume that the Yawyag Corporation in E11-8 is

IFRS. Assume that the Yawyag Corporation in E11-8 is an IFRS reporter and complete the following: Required: a. Compute the weighted-average accumulated expenditures for the current year. b. Comput...

See Answer

Q: You have been asked to account for a plant asset exchange on

You have been asked to account for a plant asset exchange on the books of the Ecara Video Game Company. On January 1, 2013, Ecara acquired a plastic extruding machine at a cost of $260,000. This machi...

See Answer

Q: Clave Building Products, Inc. conducts regularly scheduled maintenance of its

Clave Building Products, Inc. conducts regularly scheduled maintenance of its machinery and equipment every Friday afternoon. The cost of maintenance for the current year amounted to $345,000. The reg...

See Answer

Q: IFRS. Avery Air, Plc, a UK company, conducts

IFRS. Avery Air, Plc, a UK company, conducts regularly scheduled maintenance and improvement of its fleet of airplanes. During the year, it replaced engines at a cost of £2,900,000, upgraded airplane...

See Answer

Q: Your success in business thus far has put you in a position

Your success in business thus far has put you in a position to purchase a home for $500,000 located close to the university you attend. You plan to pay a 20% down payment of $100,000 and borrow the re...

See Answer

Q: Kurtis Koal Company, Inc. purchased a new mining machine at

Kurtis Koal Company, Inc. purchased a new mining machine at a total cost of $900,000 on the first day of its fiscal year. The firm estimates that the machine has a useful life of 6 years o...

See Answer

Q: IFRS. Repeat the requirements in E11-13 assuming that Kurtis

IFRS. Repeat the requirements in E11-13 assuming that Kurtis Koal Company, Inc. is an IFRS reporter and the mining machine has two components: casing and engine. The amount allocated to the engine is...

See Answer

Q: St. Charles Flooring Company recently purchased a new tile-cutting

St. Charles Flooring Company recently purchased a new tile-cutting machine with an invoice price of $215,500. The cost of delivery was $2,000, and installation amounted to $3,550. To test the machine,...

See Answer

Q: On January 2 of the current year, Vaughn, Inc.

On January 2 of the current year, Vaughn, Inc. acquired land for $2,000,000 to be used to construct a new service and repair center. The closing costs amounted to $110,000, and Vaughn paid $20,000 for...

See Answer

Q: Sonata Manufacturing Corporation decided to expand its operations and open a new

Sonata Manufacturing Corporation decided to expand its operations and open a new facility in Illinois. Rather than constructing a new plant, Sonata negotiated a contract to purchase an existing facili...

See Answer

Q: On December 31, 2018, the Clearwater Corporation acquired a custom

On December 31, 2018, the Clearwater Corporation acquired a custom-made plant asset by issuing a promissory note with a face value of $750,000, a due date of December 31, 2023, and a stated (coupon) r...

See Answer

Q: Quartech Enterprises manufactures and distributes thermostats for major kitchen appliances. At

Quartech Enterprises manufactures and distributes thermostats for major kitchen appliances. At the beginning of the current year, Quartech decided to expand its operations by acquiring a metal solderi...

See Answer

Q: Assume that the Ecara Video Game Company reports under IFRS. Repeat

Assume that the Ecara Video Game Company reports under IFRS. Repeat the requirement of E11-31, part (c). Data from E11-31: Prepare the journal entry for Ecara to record the exchange as if the exchange...

See Answer

Q: Braxton Enterprises currently has debt outstanding of $35 million and an

Braxton Enterprises currently has debt outstanding of $35 million and an interest rate of 8%. Braxton plans to reduce its debt by repaying $7 million in principle at the end of each year for the next...

See Answer

Q: Your firm currently has $100 million in debt outstanding with a

Your firm currently has $100 million in debt outstanding with a 10% interest rate. The terms of the loan require the firm to repay $25 million of the balance each year. Suppose that the marginal corpo...

See Answer

Q: Arnell Industries has just issued $10 million in debt (at

Arnell Industries has just issued $10 million in debt (at par). The firm will pay interest only on this debt. Arnell’s marginal tax rate is expected to be 21% for the foreseeable future. a. Suppose Ar...

See Answer

Q: You have just been hired by Internal Business Machines Corporation (IBM

You have just been hired by Internal Business Machines Corporation (IBM) in their capital budgeting division. Your first assignment is to determine the free cash flows and NPV of a proposed new type o...

See Answer

Q: Ten years have passed since Arnell issued $10 million in perpetual

Ten years have passed since Arnell issued $10 million in perpetual interest only debt with a 6% annual coupon, as in Problem 6 . Tax rates have remained the same at 21% but interest rates have dropped...

See Answer

Q: Bay Transport Systems (BTS) currently has $30 million in

Bay Transport Systems (BTS) currently has $30 million in debt outstanding. In addition to 6.5% interest, it plans to repay 5% of the remaining balance each year. If BTS has a marginal corporate tax ra...

See Answer

Q: Safeco Inc. has no debt, and maintains a policy of

Safeco Inc. has no debt, and maintains a policy of holding $10 million in excess cash reserves, invested in risk-free Treasury securities. If Safeco pays a corporate tax rate of 21%, what is the cost...

See Answer

Q: Kohwe Corporation plans to issue equity to raise $50 million to

Kohwe Corporation plans to issue equity to raise $50 million to finance a new investment. After making the investment, Kohwe expects to earn free cash flows of $10 million each year. Kohwe currently h...

See Answer

Q: Suppose B&E Press paid dividends at the end of each

Suppose B&E Press paid dividends at the end of each year according to the schedule below. It also reduced its share count by repurchasing 5 million shares at the end of each year at the ex-dividen...

See Answer

Q: Using Table 17.2 , for each of the following years

Using Table 17.2 , for each of the following years, state whether dividends were tax disadvantaged or not for individual investors with a one-year investment horizon: a. 1985 b. 1989 c. 1995 d. 1999 e...

See Answer

Q: Mercurial Company traded its cutting equipment for the newer air-cooled

Mercurial Company traded its cutting equipment for the newer air-cooled equipment manufactured by Broad Street Corporation. The air-cooled equipment will increase Mercurial’s productivity. The old equ...

See Answer

Q: In year 1, AMC will earn $1600 before interest and

In year 1, AMC will earn $1600 before interest and taxes. The market expects these earnings to grow at a rate of 3% per year. The firm will make no net investments (i.e., capital expenditures will equ...

See Answer

Q: Prokter and Gramble (PKGR) has historically maintained a debt-

Prokter and Gramble (PKGR) has historically maintained a debt-equity ratio of approximately 0.20. Its current stock price is $50 per share, with 2.5 billion shares outstanding. The firm enjoys very st...

See Answer

Q: Revtek, Inc., has an equity cost of capital of 12

Revtek, Inc., has an equity cost of capital of 12% and a debt cost of capital of 6%. Revtek maintains a constant debt-equity ratio of 0.5, and its tax rate is 25%. a. What is Revtek’s WACC given its c...

See Answer

Q: In 2018, Mastercard Incorporated had a market capitalization of $200

In 2018, Mastercard Incorporated had a market capitalization of $200 billion, debt of $6.5 billion, cash of $8.2 billion, and EBIT of over $7 billion. If Mastercard were to increase its debt by $1 bil...

See Answer

Q: As a new analyst for a large brokerage firm, you are

As a new analyst for a large brokerage firm, you are anxious to demonstrate the skills you learned in your MBA program and prove that you are worth your attractive salary. Your first assignment is to...

See Answer

Q: Repeat Problem 6 assuming the corporation is an S corporation.

Repeat Problem 6 assuming the corporation is an S corporation. Data from Problem 6: You are a shareholder in a C corporation. The corporation earns $2 per share before taxes. Once it has paid taxes i...

See Answer

Q: In early 2015, Ford Motor (F) had a book

In early 2015, Ford Motor (F) had a book value of equity of $24.8 billion, 4.0 billion shares outstanding, and a market price of $16 per share. Ford also had cash of $21.7 billion, and total debt of $...

See Answer

Q: In early 2018, Abercrombie & Fitch (ANF) had a

In early 2018, Abercrombie & Fitch (ANF) had a book equity of $1250 million, a price per share of $22.48, and 68.4 million shares outstanding. At the same time, The Gap (GPS) had a book equity of $314...

See Answer

Q: See Table 2.5 showing financial statement data and stock price

See Table 2.5 showing financial statement data and stock price data for Mydeco Corp. a. What is Mydeco’s market capitalization at the end of each year? b. What is Mydecoâ€...

See Answer

Q: Suppose that in 2019, Global launches an aggressive marketing campaign that

Suppose that in 2019, Global launches an aggressive marketing campaign that boosts sales by 15%. However, their operating margin falls from 5.57% to 4.50%. Suppose that they have no other income, inte...

See Answer

Q: Doris Company traded a tract of land to Rick’s Real Estate for

Doris Company traded a tract of land to Rick’s Real Estate for a similar tract of land with no significant effect on future cash flows. The old land had a carrying value of $6,500,000. The land was ap...

See Answer

Q: Find online the annual 10-K report for Costco Wholesale Corporation

Find online the annual 10-K report for Costco Wholesale Corporation (COST) for fiscal year 2017 (filed in October 2017). Answer the following questions from their income statement: a. What were Costco...

See Answer

Q: See Table 2.5 showing financial statement data and stock price

See Table 2.5 showing financial statement data and stock price data for Mydeco Corp. a. By what percentage did Mydeco’s revenues grow each year from 2016–2019? b. B...

See Answer

Q: See Table 2.5 showing financial statement data and stock price

See Table 2.5 showing financial statement data and stock price data for Mydeco Corp. Suppose Mydeco repurchases 2 million shares each year from 2016 to 2019. What would its earnings per share be in ye...

See Answer

Q: See Table 2.5 showing financial statement data and stock price

See Table 2.5 showing financial statement data and stock price data for Mydeco Corp. Suppose Mydeco had purchased additional equipment for $12 million at the end of 2016, and this equipment was deprec...

See Answer

Q: See Table 2.5 showing financial statement data and stock price

See Table 2.5 showing financial statement data and stock price data for Mydeco Corp. Suppose Mydeco’s costs and expenses had been the same fraction of revenues in 2016â€&#...

See Answer

Q: Today is March 30, 2018, and you have just started

Today is March 30, 2018, and you have just started your new job with a financial planning firm. In addition to studying for all your license exams, you have been asked to review a portion of a client’...

See Answer

Q: Suppose a firm’s tax rate is 25%. a. What

Suppose a firm’s tax rate is 25%. a. What effect would a $10 million operating expense have on this year’s earnings? What effect would it have on next year’s earnings? b. What effect would a $10 milli...

See Answer

Q: Find online the annual 10-K report for Costco Wholesale Corporation

Find online the annual 10-K report for Costco Wholesale Corporation (COST) for fiscal year 2017 (filed in October 2017). Answer the following questions from their balance sheet: a. How much cash did C...

See Answer

Q: Find online the annual 10-K report for Costco Wholesale Corporation

Find online the annual 10-K report for Costco Wholesale Corporation (COST) for fiscal year 2017 (filed in October 2017). Answer the following questions from their cash flow statement: a. How much cash...

See Answer

Q: Why has finance historically been an early adopter of new technology?

Why has finance historically been an early adopter of new technology?

See Answer

Q: Clayton Company exchanged a used machine with a book value of $

Clayton Company exchanged a used machine with a book value of $26,000 (cost $54,000 less $28,000 accumulated depreciation) and cash of $8,000 for a delivery truck. The machine is estimated to have a f...

See Answer

Q: How are the following companies utilizing new technology in the provision of

How are the following companies utilizing new technology in the provision of financial services? a. Wealthfront b. LendingTree c. Metromile d. Acorns

See Answer

Q: See Table 2.5 showing financial statement data and stock price

See Table 2.5 showing financial statement data and stock price data for Mydeco Corp. Use the data from the balance sheet and cash flow statement in 2015 to determine the following: a. How much cash di...

See Answer

Q: See Table 2.5 showing financial statement data and stock price

See Table 2.5 showing financial statement data and stock price data for Mydeco Corp. a. From 2015 to 2019, what was the total cash flow from operations that Mydeco generated? b. What fraction of the t...

See Answer

Q: See Table 2.5 showing financial statement data and stock price

See Table 2.5 showing financial statement data and stock price data for Mydeco Corp. a. In what year was Mydeco’s net income the lowest? b. In what year did Mydeco need to reduce its...

See Answer

Q: Use the incremental IRR rule to correctly choose between the investments in

Use the incremental IRR rule to correctly choose between the investments in Problem 23 when the cost of capital is 7%. At what cost of capital would your decision change? Data from Problem 23: You ar...

See Answer

Q: See Table 2.5 showing financial statement data and stock price

See Table 2.5 showing financial statement data and stock price data for Mydeco Corp. a. What were Mydeco’s retained earnings each year? b. Using the data from 2015, what was Mydeco&a...

See Answer

Q: Your manager was so impressed with your work analyzing the return and

Your manager was so impressed with your work analyzing the return and standard deviations of the 12 stocks that he would like you to continue your analysis. Specifically, he wants you to update the st...

See Answer

Q: Find online the annual 10-K report for Costco Wholesale Corporation

Find online the annual 10-K report for Costco Wholesale Corporation (COST) for fiscal year 2017 (filed in October 2017). Answer the following questions from the notes to their financial statements: a....

See Answer

Q: See Table 2.5 showing financial statement data and stock price

See Table 2.5 showing financial statement data and stock price data for Mydeco Corp. a. What were Mydeco’s gross margins each year? b. Comparing Mydeco’s gross marg...

See Answer

Q: At the end of 2017, Walmart Inc. (WMT)

At the end of 2017, Walmart Inc. (WMT) reported annual revenues of $500.34 billion, gross profit of $126.95 billion, and net income of $9.86 billion. Costco Wholesale Corporation (COST) had revenue of...

See Answer

Q: Clarke Company traded a used mixing machine for a new model.

Clarke Company traded a used mixing machine for a new model. The used machine has a book value of $11,000 (cost $32,000 less $21,000 accumulated depreciation) and a fair market value of $8,000. The ne...

See Answer

Q: At the end of 2017, Apple had cash and short-

At the end of 2017, Apple had cash and short-term investments of $74.18 billion, accounts receivable of $17.87 billion, current assets of $128.65 billion, and current liabilities of $100.81 billion. a...

See Answer

Q: See Table 2.5 showing financial statement data and stock price

See Table 2.5 showing financial statement data and stock price data for Mydeco Corp. a. How did Mydeco’s accounts receivable days change over this period? b. How did Mydecoâ...

See Answer

Q: See Table 2.5 showing financial statement data and stock price

See Table 2.5 showing financial statement data and stock price data for Mydeco Corp. a. Compare Mydeco’s accounts payable days in 2015 and 2019. b. Did this change in accounts payabl...

See Answer

Q: In September 2008, the IRS changed tax laws to allow banks

In September 2008, the IRS changed tax laws to allow banks to utilize the tax loss carry forwards of banks they acquire to shield up to 100% of their future income from taxes (prior law restricted the...

See Answer

Q: Procter and Gamble (PG) paid an annual dividend of $

Procter and Gamble (PG) paid an annual dividend of $2.87 in 2018. You expect PG to increase its dividends by 8% per year for the next five years (through 2023), and thereafter by 3% per year. If the a...

See Answer

Q: What if the last two and a half decades had been “

What if the last two and a half decades had been “normal”? Download the spreadsheet from MyLab Finance containing the data for Figure 10.1. a. Calculate the arithme...

See Answer

Q: Using the data in Table 10.2, a.

Using the data in Table 10.2, a. What was the average annual return of Microsoft stock from 2005–2017? b. What was the annual volatility for Microsoft stock from 2005â€...

See Answer

Q: Brown Company contracts with Sebastian Company to exchange refrigerated trucks. Brown

Brown Company contracts with Sebastian Company to exchange refrigerated trucks. Brown Company will trade three SMC trucks for four DROF trucks owned by Sebastian Company. The trucks are approximately...

See Answer

Q: Brown Company contracts with Sebastian Company to exchange refrigerated trucks. Brown

Brown Company contracts with Sebastian Company to exchange refrigerated trucks. Brown Company will trade three SMC trucks for four DROF trucks owned by Sebastian Company. The DROF refrigerated trucks...

See Answer

Q: You have been hired as a consultant by CARE (Conservatives Are

You have been hired as a consultant by CARE (Conservatives Are Responsible), a political action committee focusing on fiscal responsibility. The Florida chapter has become increasingly worried about t...

See Answer